Download as pdf or txt
Download as pdf or txt
You are on page 1of 189

INDEX

Unit -1 Number System 1 – 13

Unit -2 Surds, Indices & Simplifications 14 – 22

Unit -3 Ratio, Proportion & Partnership 23 – 35

Unit -4 Percentage and its Applications 36 – 43

Unit -5 Profit, Loss and Discount 44 – 52

Unit – 6 Simple and Compound Interest 53 – 63

Unit -7 Average, Mixtures and Allegation 64 – 75

Unit -8 Time and Work 76 – 86

Unit -9 Time, Speed and Distance 87 - 98

Unit -10 Algebra 99 – 111

Unit -11 Geometry 112 – 131

Unit -12 Mensuration 132 – 145

Unit -13 Trigonometry 146 – 158

Unit -14 Statistics & Probability 159 – 170

Unit -15 Data Interpretation 171 – 187


________________________________________________________
Mathematics for Competitive Exams (1)

1 NUMBER SYSTEM

NUMBER SYSTEM
Numbers are classified as:

Natural Numbers
The numbers that are used for counting are called natural numbers. Examples are 1, 2,
3...and so on.
The smallest natural number is one (1) and there is no largest natural number.

Whole Numbers
All natural numbers along with the number zero (0) form the system of whole numbers.
Hence, 0, 1, 2, 3… are all whole numbers.
The smallest whole number is 0 and there is no largest whole number.

Integers
An integer is a number that can be written without a fractional component. For example, 44,
897, -98 are integers, whereas 9.75, 5 1 2 are not. The set of integers consist of zero (0),
the positive natural numbers (1, 2, 3 …), and their additive inverses (the negative integers,
i.e.,
There is neither the smallest nor the largest integer number.

The Number Line


The number line is a straight line between negative infinity on the left to positive infinity on
the right and serves as an abstraction for real numbers.

Real Numbers
All numbers that can be represented on the number line are called real numbers. In other
words, real numbers are a combination of all rational and irrational numbers.

________________________________________________________
Subhash Institute of Competitive Exams (SICE) Vijayawada
Vijaya Ph: 9030924949
________________________________________________________
Mathematics for Competitive Exams (2)

Every twin prime pair except (3, 5) is of the form (6n


Rational Numbers
6n + 1) for some natural number n; that is, the number
p
A number that can be written in the form of , where p between the two primes is a multiple of 6.
q
and q are integers and q Coprime Numbers
Two integers x and y are said to be relatively prime,
Here, p is called the numerator and q is called the
coprime or mutually prime, if the only positive integer
denominator.
that divides both of them is 1. For example, 13 and 21
Every integer including 0 is a rational number. The are coprime numbers.
smallest and largest rational numbers cannot be
determined. Composite Numbers
A whole number that can be divided completely by
Irrational Numbers numbers other than 1 or itself is called a composite
A number which is not rational or which cannot be number. For example, 15 can be divided evenly by 3 and
p 5 (as well as 1 and 15) so, 15 is a composite number.
expressed in the form of , where p and q are integers
q Every composite number can be factorised into its prime
and q 0 is called an irrational number. Examples are , factors.

Points to remember
1 is neither a prime nor a composite number.
PLACE VALUE AND FACE VALUE
1 is an odd integer.
Face value is the actual value of the digit irrespective of 0 is neither positive nor negative.
where it is placed.
0 is an even integer.
On the other hand, every digit in a numeral has a value 2 is the only even number that is prime
depending on its place called the place value of the digit.
Smallest odd prime number is 3.
In the number system, each place has a value that's 10
times the value of the place immediately to its right. Even Numbers
Numbers which are divisible by 2 are called even
For example, in the number 456, "5" has a face value of
numbers. For example, 2, 4, and 6 are even numbers.
5 and the place value is 50.

Place value of a digit = (face value of the digit) × (value Odd Numbers
of the place) Numbers which are not divisible by 2 are called odd
numbers. For example, 1, 3, 5……… are odd numbers.
Prime Numbers
A number that is divisible only by one and itself is called Divisibility
a prime number. All prime numbers less than 100 are:
MULTIPLE
2, 3, 5, 7, 11, 13, 17, 19, 23, 29, 31, 37, 41, 43, 47,
53, 59, 61, 67, 71, 73,79, 83, 89, 97. Multiple of a number is defined as a number obtained by
the multiplication of that number by some other number.
Twin Prime Numbers
A pair of prime numbers are said to be twin prime when In other words, for quantities x and y, we say that y is a
their difference is 2. The first few twin prime pairs are: multiple of x if y = nx for some integer n, called the
(3, 5), (5, 7), (11, 13), (17, 19), (29, 31), (41, 43), (59, multiplier.
61), (71, 73), (101, 103), (107, 109), and (137, 139). For example, the list of multiples of 5 are 5, 10, 25,555,
etc.

________________________________________________________
470
Subhash Institute of Competitive Exams (SICE) Vijayawada
Vijaya Ph: 9030924949
(3)
________________________________________________________
Mathematics for Competitive Exams

FACTOR For example, 473312 is divisible by 7, because the


difference between 473 and 312 is 161, which is
Factor is a number or algebraic expression that divides divisible by 7.
another number or expression evenly without leaving a
remainder. For example, 2, 3 and 4 are factors of 12. OR

Double the last digits and subtract it from the remaining


Divisibility Test number. If the result is divisible by 7, then so is the
Divisibility by 2 original number. Apply this rule over and over again if the
A number is divisible by 2 if its unit digit is 0, 2, 4, 6, or number is quite large.
8.
For example, for 826, we double 6 to get 12. We have 82
For example, 58694 is divisible by 2, while 86945 is not. left. Now, subtracting, we get 82-12=70. This is divisible
by 7, so 826 is also divisible by 7.
Divisibility by 3
A number is divisible by 3 only if the sum of its digits is Divisibility by 8
divisible by 3. A number is divisible by 8 if the number formed by its
last three digits i.e. hundred’s, ten’s and unit’s digit is
For example, 695421, the sum of the digits of the
divisible by 8.
number 6, 9, 5, 4, 2, and 1 is equal to 27. Since, 27 is
divisible by 3, this implies that 695421 is also divisible For example, for 16789352, the number formed by last
by 3. On the other hand, the sum of the digits of 751246 3 digits is 352, which is divisible by 8. Hence, 16789352
is equal to 25, and hence it is not divisible by 3. is also divisible by 8.

Divisibility by 4 Divisibility by 9
A number is divisible by 4 if the number formed by the A number is divisible by 9 only if the sum of its digits is
ten’s and unit’s digit of the given number is divisible also divisible by 9.
by 4.
For example, sum of digits of the number 246591 is
For example, 6879376 is divisible by 4, since 76 is equal to 27, which is divisible by 9. Hence, 246591 is
divisible by 4 but 496138 is not divisible by 4 because divisible by 9 too.
38 is not divisible by 4.
Divisibility by 10
Divisibility by 5 A number is divisible by 10 only when its unit digit is 0.
A number is divisible by 5 only when its unit digit is 0 or 5.
For example, 7849320 is divisible by 10, since its unit
For example, each of the numbers 76895 and 68790 digit is 0 but 45845685 is not.
are divisible by 5.
Divisibility by 11
Divisibility by 6 A number is divisible by 11 if the difference between the
A number is divisible by 6 if it is simultaneously divisible sum of its digits at odd places and the sum of its digits at
by both 2 and 3. even places from extreme right is either 0 or a number
divisible by 11.
For example, 9390 is divisible by 6 because it is divisible
by both 2 and 3 simultaneously. For example, for 29435417, (Sum of its digits at odd
places from right) – (Sum of its digits at even places from
Divisibility by 7 right)
A number is divisible by 7 if and only if the difference of
the number of its thousands and the remaining part of = (7 + 4 + 3 + 9) – (1 + 5 + 4 + 2) = (23 – 12) = 11,
the given number is divisible by 7 respectively. which is divisible by 11. Hence, 29435417 is divisible
by 11.

________________________________________________________
471
Subhash Institute of Competitive Exams (SICE) Vijayawada
Vijaya Ph: 9030924949
(4)
________________________________________________________
Mathematics for Competitive Exams

Divisibility by 12: prime numbers before 12 as (12)2 > 137. Since, 137 is
Any number which is divisible by both 3 and 4, is also not divisible by any of the numbers 2,3,5,7, and 11,
divisible by 12. For example, 6584124 is divisible by therefore, 137 is a prime number.
both 3 and 4 and hence by 12.
Unit Digit
General rules of divisibility for all numbers: If the last digit or the digit at unit’s place of a number x is
If a number is divisible by another number, then it is 1, 5 or 6, then whatever be the value of xn, it will have
also divisible by all the factors of that number. the same digit at unit’s place, i.e.,
If two numbers are divisible by the same number, (….1)n = (…….1)
then their sum and difference is also divisible by this
(….5)n = (…….5)
number.
(….6)n = (…….6)
If a number is divisible by two coprime numbers,
then it is also divisible by the product of the two If the last digit or the digit at unit’s place of a number x is
coprime numbers. 2, 3, 5, 7 or 8, then the last digit of xn depends upon the
value of n, which follows a repeating pattern in terms of
If m and n both are divisible by d then (m + n) and
(m – n) is also divisible by d. 4 as given below:

Out of n consecutive whole numbers, one and only N Last digit Last digit Last digit Last digit
one number is divisible by n. For example, out of the of (...2)n of (...3)n of (...7)n of (...8)n
five consecutive whole numbers 10, 11, 12, 13, 14, 4x + 1 2 3 7 8
only one i.e., 10 is divisible by 5. 4x +2 4 9 9 4
4x +3 8 7 3 2
The square of an odd integer when divided by 8 will
4x 6 1 1 6
always leave a remainder of 1.
Difference between any number and the number If the last digit or the digit at unit’s place of a number x is
obtained by writing the digits in reverse order is either 4 or 9, then the last digit of xn depends upon the
divisible by 9. value of n, which follows repeating pattern in terms of 2
Any number with number of digits equal to multiples as given below:
of 6, is divisible by each of 7, 11 and 13 if all of its
N Last digit of (...4)n Last digit of (...9)n
digits are the same. For example, 666666, 888888,
2x 6 1
333333333333 are all divisible by 7, 11 and 13.
2x + 1 4 9

While finding the unit’s digit of the product of a number


x, such that x = a×b×c… and so on, the product of the
unit digits of all the numbers is the unit’s digit of the
actual product or x.

To test whether a given number is a prime number or For example,


not. To find the unit digit of 1818 × 2829 × 288283,
Let p be a number and let n be the smallest counting We have, unit digit of 1818 is 8.
number such that n2 p. Then, test whether p is divisible
Unit digit of 2829 is 9.
by any of the prime numbers less than or equal to n. If
yes, then p is not prime. Otherwise, p is a prime number. Unit digit of 288283 is 3.

Example: To check whether 137 is a prime number or Hence, unit digit of 1818 × 2829 × 288283 = 8 × 9 × 3
not, we just need to check if it is divisible by any of the = 216 i.e., 6

________________________________________________________
472
Subhash Institute of Competitive Exams (SICE) Vijayawada
Vijaya Ph: 9030924949
________________________________________________________
Mathematics for Competitive Exams (5)
Section III – Chapter 1: Number System

Remainder Theorem 2. Find the sum of squares of the first 15 natural


This theorem states that the remainder of the expression numbers
a×b × c Solution:
, is equal to the remainder of the expression
n n n+1 2n + 1
(12 + 22 + 32+ … + n2) =
ar × br × cr 6
, where,
n 15 16 31
= = 1240
6
ar is the remainder when a is divided by n separately.
3. Find the value of 163 + 173 + 183 + ……. + 303.
br is the remainder when b is divided by n separately.
Solution:
cr is the remainder when c is divided by n separately. 163 + 173 + 183 + ……. + 303 = (13 + 23 + 33 + …….
+ 303) (13 + 23 + 33 + ……. + 153)
For example,
2 2
30 30 1 15 15 1
To find the remainder of 15 × 17 × 19 when divided by 2 2
7, we must find the remainders for individual numbers
2 2
first. 15 31 15 8

Dividing 15 by 7, we get 1 as the remainder. = 4652 – 1202 = 216225 – 14400 = 201825

Dividing 17 by 7, we get 3 as the remainder.


HCF and LCM
Dividing 19 by 7, we get 5 as the remainder.

Hence, remainder of 15 × 17 × 19 when divided by


Highest Common Factor (HCF)
The number that completely divides two or more
7= remainder of 1 × 3 × 5 divided by 7
numbers is known as the highest common factor of
15 those numbers. It is also known as the Greatest
= =1
7 Common Divisor (GCD) or the Greatest Common Factor
Important points to keep in mind: (GCF). Since every number has factor 1, the Highest
(an + bn) is divisible by (a + b), if n is odd. Common Factor (HCF) of two or more numbers is at least
1.
(an - bn) is divisible by (a + b), if n is even.
(an - bn) is always divisible by (a - b), if n is an integer.

Some important results on series:


n n +1
(1 + 2 + 3+ … + n) =
2
The methods of finding HCF are discussed as follows:
n n+1 2n + 1
(12 + 22 + 32+ … + n2) = Prime Factorisation Method
6 In this method, we express each number as the product
2 of prime factors and then identify the common factors of
n2 n + 1
(13 + 23 + 33 + … + n3) = all the numbers. The product of the common factors is
4
the HCF.
Important Examples
Example: Find the HCF of 45, 75 and 120.
1. Find the sum of first 20 consecutive natural
numbers. Solution:
Solution: 45 = 3 × 3 × 5
nn 1 20 21
(1 + 2 + 3+ … + n) = = = 210 75 = 3 × 5 × 5
2 2

________________________________________________________
473
Subhash Institute of Competitive Exams (SICE) Vijayawada
Vijaya Ph: 9030924949
(6)
________________________________________________________
Mathematics for Competitive Exams

120 = 2 × 2 × 2 × 3 × 5 For example, HCF of 4, 8 and 12 is 4 as 8 and 12


are the multiples of 4.
The common factors of 45, 75 and 120 are 3 and 5.

Therefore, the HCF of 45, 75 and 120 = 3 × 5 = 15 Least Common Multiple (LCM)
The least number that is completely divisible by two or
Division Method more numbers is known as the least common multiple of
In this method, we divide the larger number by the those numbers.
smaller and get the remainder. Then, we divide the
The methods of finding LCM are explained as follows:
divisor by the remainder and repeat the process till the
remainder is zero and the last divisor is the required Prime Factorisation Method
HCF. In this method, we express each number as the product
Example: Find the HCF of 48, 84 and 132. of its prime factors. The product of the terms with
maximum powers of all the factors is the required LCM.
Solution:
Example: Find the LCM of 8, 18 and 36.

Solution:

8 = 2 × 2 × 2 = 23

18 = 2 × 3 × 3 = 2 × 3 2

36 = 2 × 2 × 3 × 3 = 2 2 × 32

Here, the prime factors that occur in given numbers are


2 and 3 and their highest powers are 23 and 32

So, the LCM of 8, 18 and 36 = 23 × 32 = 72

Therefore, the HCF of 132 and 84 is 12. Now, we Division Method


calculate the HCF of 12 and 48. In this method, we arrange all the given numbers in a
row separated by a comma. Then, we divide them by a
suitable prime number, which exactly divides at least two
numbers, and put the quotient directly under the
numbers in the next row and continue the process until
all co-prime numbers are left in the last row. The product
Hence, the HCF of 48, 132 and 84 is 12.
of all the prime numbers (by which we have divided the
Notes: numbers) and co-prime numbers (left in the last row) is
The HCF of two or more co-prime numbers is always the required LCM.
equal to 1 and its converse is also true.
Example: Find the LCM of 42, 63 and 81.
For example, HCF of 9 and 14 is 1.
Solution:
The HCF of any given numbers is always less than or
equal to the smallest number among them. 3 42, 63, 81

For example, HCF of 6, 8 and 10 is 2 which is always 3 14, 21, 27


less than 6 (the smallest number). 7 14, 7, 9
If numbers are the multiples of the smallest number, 2, 1, 9
then the smallest number will be the HCF.
So, the LCM of 42, 63 and 81 = 3 × 3 × 7 × 2 × 9 =
1134

________________________________________________________
474
Subhash Institute of Competitive Exams (SICE) Vijayawada
Vijaya Ph: 9030924949
(7)
________________________________________________________
Mathematics for Competitive Exams Section III – Chapter 1: Number System

Notes: 12 15 HCF of Numerators 3


HCF of = =
The LCM of any set of given numbers cannot be 19 38 LCM of Denominators 38
smaller than the largest among them.
Example: The LCM of two numbers is 6 times of their
For example, the LCM of 14 and 21 is 42 (greater HCF. If LCM is 54, then what is the product of numbers?
than 21).
(1) 48 (2) 192
The LCM of co-prime numbers is always equal to (3) 96 (4) 486
their product. Solution:
For example, the LCM of 2 and 5 is 10 (2 × 5). LCM of numbers 54
HCF of numbers = 9
6 6
The LCM of any set of given numbers is equal to the Product of numbers = Product of LCM and HCF = 54 × 9
greatest number, if the greatest number is the = 486
multiple of all other numbers.
For example, the LCM of 6, 12 and 24 is 24 (12 × 2 Square and Square Roots
= 24, 6 × 4 = 24). When a number is multiplied by itself, the resultant
number is the square of the original number. For
Important Properties of H.C.F. and L.C.M. example, 42 = 16, 52 = 25.
The product of L.C.M. and H.C.F. of any two given
On the other hand, the square root of a number is a
natural numbers is equivalent to the product of the
value that, when multiplied by itself, gives the original
given numbers implying,
number. It is represented by the symbol is ‘
L.C.M. × H.C.F. = Product of the Numbers.
positive square root. For example,
For example, L.C.M. and H.C.F. of 9 and 12 is 3 and
L.C.M. is 36. In simple words, if x and y are two numbers such that, x 2
= y, we say that the square root of y is x and we write
Product of 9 and 12 is 108.
= ±x.
Product of L.C.M. and H.C.F. is 108
Hence, the property is stated. Note that 9 × 9 = ( × so is also a
square root of 81.
Since the H.C.F. of coprime numbers is 1, the
LCM of given coprime numbers is equal to the Cube and Cube Roots
product of the numbers. When a number is multiplied by itself three times, the
To find H.C.F. and L.C.M. of Fractions, use the resultant number is called the cube of the original
formula, number. For example, 63 = 216, 73 = 343.

LCM of numerators On the other hand, the cube root of a number is a value
L.C.M. of fractions =
HCF of denominators that, when multiplied by itself three times, gives the
HCF of numerators original number. It is represented by the symbol is
H.C.F. of fractions = ‘3 3
LCM of denominators
3
12 15
Example: Find the LCM and HCF of .
19 38 In simple words, if x and y are two numbers such that, x 3
= y, we say that the cube root of y is x and we write 3
Solution: LCM of numerators = 60 and HCF of
= x.
denominators = 19
Example: Find the remainder when 210 – 1 is divided
12 15 LCM of Numerators 60
LCM of = = by 3.
19 38 HCF of Denominators 19
Solution:
HCF of numerators = 3 and LCM of denominators = 38
an – bn is always divisible by a + b if n is even.

________________________________________________________
475
Subhash Institute of Competitive Exams (SICE) Vijayawada
Vijaya Ph: 9030924949
(8)
________________________________________________________
Mathematics for Competitive Exams

210 – 1 = 210 – 110 Hence, unit’s digit of 32003 will be 7.

As 10 is even, 210 – 110 is divisible by 2 + 1= 3 Example: If N = 4 4 34 52 , find the total number of


odd prime factors of N.
Example: LCM of two prime numbers x and y, (x > y), is
161. The value of 3y – x is Solution: Factorised form of N = 28 34 52
Solution: To find the number of odd factors, ignore the power of 2
We know that the LCM of two prime numbers is the and consider the powers of 3 and 5.
product of those numbers. Number of odd factors of N = (4 + 1)(2 + 1) = 5 × 3 = 15
161 is the product of the prime numbers 23 and 7. Example: The value of y for which the 4-digit number
Therefore, 161 is the LCM of 23 and 7. 51y3 is divisible by 9 is

We get x = 23 and y = 7 Solution:

Now, 3y – x = 3 × 7 – 23 = 21 – 23 = – 2 A number is divisible by 9 if the sum of its digits is also


divisible by 9.
Example: The smallest number by which 68600 must be
multiplied to get a perfect cube is Now, sum of digits of 51y3 = 5 + 1 + y + 3 = y + 9

So, y + 9 will be divisible by 9 if the value of y is either 0


Solution:
68600 = 2 × 2 × 2 × 5 × 5 × 7 × 7 × 7 or 9.

= (2 × 7)3 × 5 × 5 Example: The smallest value of ‘y’ in the number


9y8071 so that it is divisible by 11 is:
Therefore, it should be multiplied by 5 to make it a
perfect cube. Solution:

Example: Find the Unit's digit in 132003. A number is divisible by 11 if the difference of sum of
digits at even places and odd places is either 0 or
Solution: divisible by 11.
Unit’s digit 132003 will be the same as the unit’s digit of Here, the difference of sum of digits at even places and
32003. odd places in 9y8071
Now,
= (9 + 8 + 7) – (y + 0 + 1)
Unit’s digit of 34 will be 1.
= 24 – y – 1
Unit’s digit of all integral power of 34 will be 1.
Unit’s digit of (34)500 = 32000 will be 1. = 23 – y

And Unit’s digit of 33 will be 7. 23 – y will be divisible by 11 if the value of digit “y” is 1.
Unit’s digit of 3200 = Unit digit of 32000 × Unit digit 33
=1×7=7

________________________________________________________
476
Subhash Institute of Competitive Exams (SICE) Vijayawada
Vijaya Ph: 9030924949
________________________________________________________
Mathematics for Competitive Exams (9)
Section III – Chapter 1: Number System

PRACTICE QUESTIONS
Answer the following questions by selecting the most appropriate option.

1. Find the least number that must be added to 11. Find the least three-digit number which when
34126 so that the resulting number is exactly divided by 12, 16, 20 and 24 leaves 5 as
divisible by 12. remainder in each case.
(1) 4 (2) 5 (1) 240 (2) 245
(3) 8 (4) 2 (3) 250 (4) 485
2. What is the HCF of 24, 36 and 42? 12. Aditi, Govind and Pranav take 4 min, 5 min and 6
(1) 4 (2) 6 min, respectively to complete one round of a
(3) 12 (4) 36 park. If they start walking at 10:00 am together,
at what time will they meet again at the starting
3. Find the least number which must be added to
point?
32642 so that the resulting number is divisible
by 112. (1) 10:15 am
(1) 52 (2) 50 (2) 10:30 am
(3) 62 (4) 60 (3) 11:00 am
(4) 12:00 pm
4. The value of (11111)2 is
(1) 121212121 (2) 11344311 13. The unit digit in the product of (2467) 158 ×
(3) 12344321 (4) 123454321 (341)72 is
(1) 3 (2) 9
5. Find the greatest number that will divide 481 (3) 7 (4) 1
and 582 leaving remainders 9 and 10,
respectively? 14. The HCF of two numbers is 15 and their LCM is
(1) 8 (2) 4 840. If one of the numbers is 105, then what is
(3) 16 (4) 2 the second number?
(1) 8 (2) 110
6. What is the smallest of three consecutive odd (3) 120 (4) 150
integers if the sum of the second and the third
integers is 8 more than the difference of the first 15. The LCM of two numbers is 4 times of their HCF.
and the second integer? If LCM is 48, then what is the product of
(1) 0 (2) 3 numbers?
(3) 5 (4) 7 (1) 48 (2) 192
(3) 96 (4) 576
7. Fraction becomes 4 when 1 is added to the
numerator and becomes 5 when 1 is subtracted 16. A cuboid of dimension 80 cm × 60 cm × 50 cm
from the denominator. The numerator of the has to be cut into the minimum number of cubes
given fraction is: of same size without wasting any material. Find
(1) 15 (2) 11 the number of cubes.
(3) 13 (4) 17 (1) 48 (2) 96
(3) 240 (4) 100
8. 12 is divided into two parts such that 3 times the
first part added to 4 times the second part 17. The difference of HCF and LCM is 15. If the
makes 43. The difference between both parts is product of numbers is 54, then what is their
(1) 1 (2) 2 LCM?
(3) 3 (4) 7 (1) 27 (2) 24
(3) 18 (4) 15
9. When a number is divided by 243, the remainder
is 65. What will be the remainder when it is 18. The product of HCF and LCM is the square of 24.
divided by 27? If one number is 48, then what is the other
(1) 11 (2) 14 number?
(3) 20 (4) 8 (1) 12 (2) 24
(3) 18 (4) 36
10. The sum of digits of a two-digit number is 13.
When 45 is added to the number, the digits of 19. If the sum of LCM and HCF of two numbers is 66
the number are reversed. What is the actual and their difference is 44, then what is the
number? product of numbers?
(1) 31 (2) 85 (1) 2904 (2) 726
(3) 94 (4) 49 (3) 1210 (4) 605

________________________________________________________
477
Subhash Institute of Competitive Exams (SICE) Vijayawada
Vijaya Ph: 9030924949
( 10 )
________________________________________________________
Mathematics for Competitive Exams

20. Three cloths of 7 m, 7.5 m and 9.5 m, 30. Find the sum of two consecutive even numbers,
respectively have to be cut into same length difference of whose squares is 60.
pieces of maximum size. What will be the total (1) 34 (2) 46
number of pieces? (3) 58 (4) 70
(1) 24 (2) 48
31. n is a whole number which when divided by 6
(3) 96 (4) 120
gives 5 as remainder. What will be the remainder
21. The sum of two numbers is 23 and the difference when 2n is divided by 6?
is 5. What is the sum of their squares? (1) 0 (2) 4
(1) 277 (2) 291 (3) 2 (4) 5
(3) 349 (4) 554
32. If the product of two numbers is 1728 and the
22. Which one of the following is the least number of quotient, when the larger number is divided by
four digits exactly divisible by 24? the smaller is 3, then find the sum of the
(1) 1006 (2) 1008 numbers.
(3) 1012 (4) 1020 (1) 72 (2) 84
(3) 108 (4) 96
23. The least number which must be subtracted from
the greatest number of four digits so that it will 33. What will be the remainder when (3785 + 36) is
be divisible by 37 is divided by 38?
(1) 3 (2) 7 (1) 28 (2) 11
(3) 1 (4) 9 (3) 23 (4) 35
24. The HCF of two numbers "a" and "b" is 18. What 34. How many 2s are used in writing numbers from
can be the LCM of "a" and "b"? 200 to 300?
(1) 42 (2) 9 (1) 111 (2) 117
(3) 12 (4) None of these (3) 128 (4) 120
25. The unit's place digit of 129 121 + 7887 is 35. Three friends J, K and L jog around a circular
(1) 2 (2) 3 stadium and complete one round in 12, 18 and
(3) 4 (4) 1 20 seconds respectively. In how many minutes
will all the three meet again at the starting
26. When 3626 is divided by the square of a number
point?
and the answer so obtained is multiplied by 32,
(1) 5 (2) 8
the final answer is 2368. What is the number?
(3) 12 (4) 3
(1) 7 (2) 36
(3) 49 (4) 6 36. Let A be the greatest number that divides 1300,
4650 and 6325, leaving the same remainder in
27. How many pairs of 2-digit numbers exist such
each case. What is the sum of the digits in A?
that the difference between them and the
(1) 8 (2) 19
number obtained by interchanging their digits is
(3) 22 (4) 27
divisible by 45?
(1) 5 (2) 4 37. The product of three consecutive numbers is
(3) 3 (4) 2 4080. What is the least number?
(1) 17 (2) 14
28. The least positive integer is a perfect square and
(3) 13 (4) None of these
is also divisible by 27, 39 and 48. What is the
least positive integer? 38. How many pairs of numbers are there such that
(1) 8112 (2) 24336 their HCF and LCM are 12 and 924, respectively?
(3) 73008 (4) 219024 (1) 1 (2) 2
(3) 3 (4) 4
29. Three numbers are such that the first number is
10 less than twice the second and the second 39. What is the last digit of pqr2xy80?
number is 1 more than twice the third. Twice the (1) 6 (2) 2
second number is equal to the sum of the first (3) 8 (4) 4
and third numbers. Find the sum of first, second
40. A number when divided by 312 leaves 64 as
and third numbers.
remainder. When the same number is divided by
(1) 45 (2) 63
39, the remainder will be
(3) 81 (4) 99
(1) 25 (2) 29
(3) 31 (4) 35

________________________________________________________
478
Subhash Institute of Competitive Exams (SICE) Vijayawada
Vijaya Ph: 9030924949
________________________________________________________
Mathematics for Competitive Exams ( 11 )
Section III – Chapter 1: Number System

ANSWERS AND EXPLANATIONS


1. (4) Divide 34126 by 12, According to the question, = 0 + 11
After dividing, we get 10 as the x+1 = 11
=4
remainder. So, the least y
10. (4) Let the unit digit of the
number which must be added
x + 1 = 4y number be 13 – x and the tens
to 34126 to make it divisible
digit be x.
by 12 is (12 – 10) = 2. x = 4y – 1 …..(1)
Then, number = 10x + 13 – x =
2. (2) We have, Again, according to the 9x + 13
24 = 2 × 2 × 2 × 3 question, we have:
Reversed number = 10(13 – x)
36 = 2 × 2 × 3 × 3 x + x = 130 – 9x
=5
y–1
42 = 2 × 3 × 7 According to the question,
Therefore, HCF = 2 × 3 = 6 x = 5y – 5 …..(2) 9x + 13 + 45 = 130 – 9x
3. (3) After dividing 32642 by From Eq. (1) and (2), we get: 9x + 58 = 130 – 9x
112, we get 50 as remainder. 4y – 1 = 5y – 5
9x + 9x = 130 – 58
Therefore, the least number – 1 + 5 = 5y – 4y
that must be added to 32642 18x = 72
to make it divisible by 112 is y=4
x=4
(112 – 50) = 62 Putting the value of y in Eq. (1),
Therefore, the required number
4. (4) As we know (11)2 = 121 we get
= 9x + 13 = 9 × 4 + 13
(111)2 = 12321 x = 4 × 4 – 1 = 15
= 36 + 13 = 49
Similarly, (11111)2 8. (2) Let the first part be x, then
11. (2) We have,
= 123454321 the second part is 12 – x.
12 = 2 × 2 × 3
5. (2) The greatest number that According to the question,
will divide 481 and 582 leaving 16 = 2 × 2 × 2 × 2
3x + 4(12 – x) = 43
9, 10 as remainders 20 = 2 × 2 × 5
3x + 48 – 4x = 43
= HCF of (481 – 9) and (582 – 24 = 2 × 2 × 2 × 3
10) = HCF of (472, 572) = 4 – x = 43 – 48
Therefore, LCM of 12, 16, 20
6. (2) Let the three consecutive –x=–5 and 24 = 2 × 2 × 2 × 2 × 3 × 5
odd integers be x – 1, x and x + = 240
1. x=5
First number = 5 Required number = 240 + 5 =
According to the question, 245
x + x + 1 = 8 + (x – (x – 1)) Second number = 12 – 5 = 7
12. (3) LCM of 4 min, 5 min and 6
2x + 1 = 8 + 1 Difference = 7 – 5 = 2 min = 60 min
9. (1) Let the number be N. 60 min = 1 hour
2x = 8
According to the question, 1 hour after 10:00 am = 11:00
x=4
N = 243k + 65 am
x–1=4–1=3
where k = 0, 1, 2, 3,….. 13. (3) Unit digit of (2467)153 =
x+1=4+1=5 Unit digit of (2467)152× unit
Let R denote the remainder digit of 2467 = 1 × 7 = 7
Clearly, the smallest of the
when N is divided by 27,
three odd integers is 3. Unit digit of (341)72 = 1
N 243k 65
x R =R +R Therefore, the Unit digit of
7. (1) Let the fraction be . 27 27 27
y (2467)153× (341)72 = 7 × 1 = 7

________________________________________________________
479
Subhash Institute of Competitive Exams (SICE) Vijayawada
Vijaya Ph: 9030924949
________________________________________________________
Mathematics for Competitive Exams ( 11 )

14. (3) Product of two numbers = 19. (4) LCM + HCF = 66 ……(1) 23. (4) Four-digit greatest number
Product of their LCM and HCF = 9999
LCM – HCF = 44 …… (2)
First number × second number When we divide 9999 by 37
Adding Eq. (1) and (2), we get:
= LCM × HCF using long division method, we
2 × LCM = 110 get:
105 × second number =
840 × 15 LCM = 55 Quotient = 270
Second number Putting LCM = 55 in Eq. (1), we Remainder = 9
840 15 get
= = 120 i.e., the greatest number of
105 55 + HCF = 66 four digits divisible by 37
15. (4) HCF of numbers HCF = 11 = 9999 – 9 = 9990
LCM of numbers 48
= = Product = 55 × 11 = 605 Therefore, 9 must be
4 4 subtracted from the largest
20. (2) 7 m = 700 cm four-digit number so that it is
Product of numbers = Product
of LCM and HCF 7.5 m = 750 cm divisible by 37.
= 48 × 12 = 576 9.5 m = 950 cm 24. (4) HCF is always a factor of
LCM. Since, none of the given
16. (3) Sides of cuboid = HCF of 80 HCF of 700 cm, 750 cm and options is a multiple of 18, the
cm, 60 cm and 50 cm 950 cm = 50 cm correct option is "None of
= 10 cm Number of pieces these".

Number of cuboids 25. (4) The unit's place digit of


700 750 950
80 60 50 129121 + 7887
= 50 50 50
3
10 = 14 + 15 +19 = 48 = Unit's place digit of 129121 +
80 60 50 Unit's place digit of 7887
= = 240. 21. (1) Let the first number be x.
10 10 10 = Unit's place digit of 9121 +
Second number = 23 – x
17. (3) Let HCF be a and LCM be a Unit's place digit of 887
+ 15. According to the question,
= Unit's place digit of 91 +
Product of LCM and HCF = 54 x – (23 – x) = 5 Unit's place digit of 83 (since
the order of cycles of 9 and 8
a (a + 15) = 54 x – 23 + x = 5 are 2 and 4 respectively)
a2 + 15a = 54 2x = 28 = Unit's place digit of (9 + 2)
x = 14 =1
a2 + 15a – 54 = 0
First number = 14 26. (1) Let the required number be
(a – 3)(a + 18) = 0
x and answer obtained on
a = 3, –18 Second number = 9 dividing 3626 by x2 be a.
Since, HCF cannot be negative Sum of the squares of numbers According to the question, 32 ×
the HCF = a = 3 = 142 + 92 = 196 + 81 = 277 a = 2368
LCM = a + 15 = 18 22. (2) Least four-digit number = Therefore, a = 74
1000
18. (1) Product of numbers = x2 = 3626 ÷ 72 = 49
Product of LCM and HCF When we divide 1000 by 24
using long division method, we So, x = 7
First number × second number get 27. (2) Let the two-digit number be
= 242 10x + y and the number
Quotient = 41
48 × second number = 576 obtained by interchanging its
Remainder = 16 i.e., the least digits is 10y + x.
576 number of four digits divisible
Second number = by 24 According to the given
48 question,
= 12 = 1000 – 16 + 24 = 1008

________________________________________________________
480
Subhash Institute of Competitive Exams (SICE) Vijayawada
Vijaya Ph: 9030924949
________________________________________________________
Mathematics for Competitive Exams ( 13 )
Section III – Chapter 1: Number System

10x + y – x – 10y = 9x – 9y is So, the required sum = 28 + = HCF of (4650 – 1300),


divisible by 45. 30 = 58 (6325 – 4650) and (6325 –
1300) = HCF of 3350, 1675
Therefore, (x – y) should be a 31. (2) n = 6a + 5, therefore, 2n
and 5025 = 1675 = A
multiple of 5, i.e. it can be 5, = 12a + 10
10, 15 and so on. Since a and Therefore, the required sum of
= 6(2a + 1) + 4
b are single digits, their digits of A
difference cannot be a two-digit So, the remainder on dividing
= 1 + 6 + 7 + 5 = 19
number. 2n by 6 = 4
37. (4) Let the three consecutive
Hence x – y = 5 and the 32. (4) Let the two numbers be a
numbers be x – 1, x and x + 1.
possible values of x and y are and b.
(x = 9, y = 4), (x = 8, y = 3), (x = According to the question,
So, a × b = 1728 and a ÷ b = 3
7, y = 2) and (x = 6, y = 1), i.e.
b = 24 and a = 72 (x – 1) × x × (x + 1) = 4080
there are 4 pairs of two-digit
numbers. Therefore, the required sum x3 x2 x2 x 4080
= 24 + 72 = 96
28. (4) 27 = 3 × 3 × 3
x 3 x 4080 on solving, we
39 = 3 × 13 33. (4) If n is an odd number, then
get x = 16
(xn + 1) is always divisible by
48 = 2 × 2 × 2 × 2 × 3 (x + 1). Therefore, the least number
= 15
The least positive integer Therefore, (3735 + 1) will be
divisible by 27, 39 and 48 is divisible by 37 + 1 = 38 38. (2) Since HCF is a factor of
the LCM of these numbers = 2 both numbers, these numbers
× 2 × 2 × 2 × 3 × 3 × 3 × 13 (3735 + 36) = (3735 + 1) + 35,
must be 12a and 12b, where a
Therefore, the required and b are relatively prime.
= 5116
remainder is 35. According to the given
Therefore, the required square condition, 12ab = 924
34. (4) There are 101 numbers
number
from 200 to 300. Therefore, ab = 77
=2×2×2×2×3×3×3×
From 200 to 300, there are So, the required possible pairs
13 × 3 × 13 = 219024
hundred (100) 2s at hundreds' are (12, 924) and (84, 132).
29. (2) Let the three numbers be a, place + ten (10) 2s at tens'
39. (1) Last digit of pqr2xy80 = Last
b and c place + ten (10) 2s at ones'
digit of 2xy80
place
According to the given
Since the order of cycle of 2 is
question, Therefore, the total number of
4 and on dividing xy80 by 4,
2s used from 200 to 300
a + 10 = 2b, b – 1 = 2c and 2 the remainder is 0, the
=a+c = 100 + 10 + 10 = 120 required last digit = last digit of
24 = 6
a = 32, b = 21 and c = 10 35. (4) Time interval between two
successive meetings 40. (1) Number = 312a + 64
So, the required sum
= 39(8a) + 64
= LCM of (12, 18, 20) = 180
= 10 + 21 + 32 = 63
seconds or 3 minutes = 39(81 + 1) + 25
30. (3) Let the two consecutive
All three will meet again at the So, the required remainder =
even numbers be 2x and 2x +
starting point after 3 minutes. 25
2.
36. (2) The greatest number that
As given, (2x + 2)2 – (2x)2 = 60
divides 1300, 4650 and 6325,
x = 14 and the numbers are leaving the same remainder in
28 and 30 each case

________________________________________________________
481
Subhash Institute of Competitive Exams (SICE) Vijayawada
Vijaya Ph: 9030924949
( 14 )
________________________________________________________
Mathematics for Competitive Exams

2 SURDS, INDICES AND SIMPLIFICATION

EXPONENTS (INDICES)
The exponent of a number describes the number of times the number is multiplied by itself.
For example, in 75, 7 is called base and 5 is called exponent. It states that number 7 is
multiplied 5 times that is 7 × 7 × 7 × 7 × 7.

SURDS
Surds are irrational numbers that can be expressed with roots, such as 2 or 5 19 . In other
words, these are just numbers with fractional indices or exponential powers.
Any operation with indices can be applied to surds, and indices and surds are related through
a rule, where
m n m
x n x

Rules of Exponents
Exponents have several rules that are used for simplifying expressions. Rules given below are
applicable for any real number ‘a’ and rational exponents m and n.

Product rule
am × an = am+n
If numbers having the same base are multiplied together, the resultant number has the same
base and the exponent as the addition of exponents.
For example, 62 × 63 = 6(2+3) = 65

Quotient rule
am m-n
=a
an
If numbers having the same base are divided, the base of the resultant number remains the
same while the exponent is formed by subtracting the denominator exponent from the
numerator exponent.

38
For example, = 3(8–5) = 33
35

________________________________________________________
Subhash Institute of Competitive Exams (SICE) Vijayawada
Vijaya Ph: 9030924949
________________________________________________________
Mathematics for Competitive Exams ( 15 )
Section III – Chapter 2: Surds, Indices and Simplification

Power rule n
m
am = a n
(am)n = amn
If root of power is taken, then keep the same base and
The two exponents are multiplied when one exponent is
divide the power by the root.
raised to another.
6
3
For example, (92)4 = 92 × 4 = 98 For example, 86 83 82 64

Zero exponent rules Some important rules for exponents:


a0 = 1 If the base of a number is 0, the resultant number
The base (except 0) will always be 1 when the exponent will always be zero for any exponent except 0.
is zero for a non-zero base. If the base of a number is 1, the resultant number
For example, 20 = 1, 99999990 = 1 will always be 1 irrespective of the exponent. For
example, 18 = 131 = 1
Expanded power rule If the base of a number is –1, the resultant number
(ab)m = ambm will always be 1 when the exponent is even and –1
For example, (7 × 11)2 = 72 × 112 when the exponent is odd (–1)9 = –1 and (–1)16 = 1.
When the base is a fraction between zero and one,
Or 72 × 112 = (7 × 11)2
the resultant value decreases with an increase in the
a
m
am exponent. For example, (0.6)2 = 0.36 and 0.36 <
And
b bm 0.6

3
The exponent of 10 specifies the number of zeros to
2 23
For example, be written. For example, 105 = 100000 (five zeros).
5 53
In case of multiplying a decimal number, the decimal
Negative exponent rule point is moved in the right direction the number of
1 1 places equal to the power or exponent value of 10.
a m
m
and m am
a a For example, 0.00062 × 104 = 6.2
Sign of exponent changes if numerator or denominator is In case of dividing a decimal number, the decimal
moved across the fraction bar. point is moved in the left direction the number of
places equal to the power or exponent value of 10.
1
For example, 8–3 = For example, 0.27 ÷ 103 = 0.00027
83

1
53 VBODMAS
53
In mathematics, VBODMAS is a rule that specifies the
n n
a b order of operation in all calculations. VBODMAS is an
And
b a
acronym for:
3 3
3 5 V-V means Vinculum or Bar i.e. x
For example,
5 3
For example, 72 7 2 = 72 ÷ 9 = 8
1
Note: a
a B - Brackets
Parts of calculation inside the brackets are always done
1
For example, 8 first.
8

Taking root of a power

________________________________________________________
483
Subhash Institute of Competitive Exams (SICE) Vijayawada
Vijaya Ph: 9030924949
________________________________________________________
Mathematics for Competitive Exams ( 16 )

O - Orders Note: ‘Of’ is used as a multiplier and in simplification. For


Solve for orders if there is any, like powers, square roots example,
or cube roots.
2 2
of 85 + 8 = 85 8 = 2 ×17 + 8 = 34 + 8 = 42
DM – Divide or multiply before addition or subtraction. 5 5

AS – Do addition and subtraction in the last (from left to While performing simplification, always ensure that the
right). brackets are opened in the following order of priority:

It is useful for solving problems related to simplification i. vinculum ‘----’


as it helps in identifying the sequence of operations to be ii. Circular brackets ( )
carried out. iii. Curly brackets { }
For example iv. Square brackets [ ]
Let’s take an example to simplify a statement using the
72 2 7 2 3 2 3 2 5
BODMAS rule:

72 2 7 2 5 3 2 5 (vinculum) Simplify the following expression:

72 2 9 5 3 2 5 (bracket) 20 ÷ 8 of (45 + 9) + 15 =?

Solution:
72 2 4 3 2 5
We have 20 ÷ 8 of (45 + 9) + 15
72 8 3 2 5 (of)
20 54
= 20 ÷ 8 of 54 + 15 = 15
9 3 2 5 (divide) 8
= 135 + 15 = 150
9 6 5 (multiply)

15 5 (addition)

10 (substraction)

________________________________________________________
484
Subhash Institute of Competitive Exams (SICE) Vijayawada
Vijaya Ph: 9030924949
________________________________________________________
Mathematics for Competitive Exams ( 17 )
Section III – Chapter 2: Surds, Indices and Simplification

PRACTICE QUESTIONS
Answer the following questions by selecting the most appropriate option.
2 4 1 1
1. 96 ? of 2 3 100 . Find the missing 11. Find the value of the question mark (?) in the
3 3 2 3
4 18 8 75 9 2
number. following expression: ?
(1) 2 (2) 3 12 32 3
(3) 5 (4) 6 1 2
(1) (2)
6 6
2. Given that 7 = 2.64, what is the value of
4
36 7 – 4 7 3 7 (3) (4) 0
to one place of decimal? 6
5 7
(1) 0.1 (2) 0.8 12. Find the value of the question mark (?) in the
(3) 0.3 (4) 0.6 following expression:
3. The smallest number which is a perfect square 2 2
?×1 4 of 2142 30% of 2500 1369
and contains 1824 as a factor is: 3 3
(1) 207936 (2) 156816 (1) 312 (2) 295.88
(3) 324041 (4) 281269 (3) 312.78 (4) 383.78
4. If 1.7161 = 1.31, the value of 13. Find the value of the question mark (?) in the
is (Correct to2 following expression: (64)3 × (8)8 ÷ (512)2 = (4)?
decimal places) (1) 8 (2) 7
(1) 94.32 (2) 96.72 (3) 12 (4) 10
(3) 96.45 (4) 96.14 14. Find the value of the question mark (?) in the
1 following expression: (9)3 × (27)3 × (81)3 = (?)27
3
1 1 4
(1) 9 (2) 27
5. Find the value of 5 8 3 27 3 . (3) 16 (4) 3

(1) 5 (2) 10 15. Find the value of the question mark (?) in the
(3) 6 (4) 15 following expression: 62% of 350 – ?% of 140 =
154
1
9 12 (1) 35 (2) 55
6. The value of 3 is: (3) 45 (4) 30
1
9 12
3 16. Find the value of the question mark (?) in the
(1) 81/65 (2) 13/75 3 1
(3) 65/81 (4) 31/30 following expression: of of 512 = ?
4 2
7. Simplify 1 ÷ [1 + 1 ÷ {1 + 1 ÷ (1 ÷ 1 ÷ 1)}] (1) 148 (2) 132
(1) 2/3 (2) 2 (3) 768 (4) 192
(3) 1/3 (4) 8
17. If a = 64 and b = 289, then the value of
0.000729 1
2
8. 3
a b b a is
0.085184
44 27 (1) 21/2 (2) 2
(1) (2) (3) 4 (4) –2
9 42
27 9 62 72 82 92 10 2
(3) (4) 18. is equal to
44 44
7 4 3 4 2 3
9. 279.004 + 3.959 + 22.875 =? (1) 355 (2) 330
(1) 305 (2) 307 (3) 366 (4) 305
(3) 306 (4) 300
19. The value of
10. Find the approximate value of the question mark
(0.67 × 0.67 × 0.67) (0.33 × 0.33 × 0.33)
(?) in the following expression: ? = 110.002 ÷
103 × (9.99)2. (0.67 × 0.67) + (0.67 × 0.33) + (0.33 × 0.33)
(1) 11 (2) 22 (1) 3.4 (2) 0.34
(3) 100 (4) 1000 (3) 11 (4) 1.1

________________________________________________________
485
Subhash Institute of Competitive Exams (SICE) Vijayawada
Vijaya Ph: 9030924949
( 18 )
________________________________________________________
Mathematics for Competitive Exams

20. Find the value of the question mark (?) in the 31. 10000 ÷ 100 ÷ 10 = ?
following expression: ? 484 625 (1) 1 (2) 10
(1) 47 (2) 53 (3) 100 (4) 1000
(3) 472 (4) 53 32. 22 × 44 ÷ 16 = 4?
21. If x = 3 and y = 1, find the value of xy + yx (1) 1 (2) 5
(3) 4 (4) 3
(1) 1/3 (2) 1/3
(3) 2/3 (4) 2/3 33. 8+ 2+ 4 =?
22. The simplification of (1) 3 2 (2) 2 2
0.3 0.3 0.3 0.2 0.2 0.2 (3) 2 3 2 (4) 3 2
gives:
(0.3 0.2)
34. Find the approximate value: 4004 ÷ (399 ÷ 4.98)
(1) 0.07 (2) 0.007
× 3.0109 = ?
(3) 0.7 (4) 7
(1) 150 (2) 6
(3) 250 (4) 60
23. Find the value of 5 5 5 5 5 5 .... 35. 5 × ? = 9584 ÷ 4
(1) 2 (2) 4 (1) 7667.2 (2) 958.4
(3) 3 (4) 5 (3) 497.2 (4) 479.2

1 1 36. 7 3
is equal to:
24. If 1 of a number is , then what is the value
3 27 7– 3
4 5 21 6 35
of of the same number? (1) (2)
5 2 10
(1) 2/45 (2) 1/45 6 22 5 35
(3) 3/45 (4) 1 (3) (4)
12 12
0.04 0.0025 0.16 81 1 7 13
25. =? 37. 1 × 3 ÷1 =?
0.125 0.256 0.09 5 5 15 25
(1) 0.3 (2) 0.5 1 14
(3) 0.220 (4) 2 (1) 2 (2) 2
17 19
1
26. The least number by which 16595 should be (3) 2 1 (4) 2
divided to make it a perfect square is: 13 19
(1) 9 (2) 6 38. Find the approximate value of the question mark
(3) 4 (4) 7 (?) in the following expression:
2 2
27. 82 × 64–1 = ? ? = 6.003 ÷ 2.02 + 365.125
(1) 64 (2) 128 (1) 310 (2) 360
(3) 4096 (4) 1 (3) 375 (4) 395
28. The value of 39. Find the value of the question mark (?) in the
1 1 1 1 1 following expression:
is
7 6 6 5 5 2 8 7 3 8 57 9.5 63 11.5 1408 ?
(1) 5 (2) 1 (1) 15 (2) 35
(3) 7 (4) 0 (3) 25 (4) 45

29. If x = a 3 b a 3 b ........... , the value of x is


40. What is the value of
5
ab 3 5
a3b 4 + 131 + 154 + 211 + 196 ?
(1) (2)
3 3 (1) 16 (2) 4
(3) a5b (4) a3b
(3) 12 (4) 8
30. 500 ÷ 50 × 5 = ?
(1) 2 (2) 5 41. Evaluate: 0.16 0.16 2 0.16 0.25 0.25 0.25
(0.16 0.25)
(3) 50 (4) 10
(1) 0.30 (2) 3.6
(3) 0.41 (4) 0.50

________________________________________________________
486
Subhash Institute of Competitive Exams (SICE) Vijayawada
Vijaya Ph: 9030924949
( 19 )
________________________________________________________
Mathematics for Competitive Exams Section III – Chapter 2: Surds, Indices and Simplification

ANSWERS AND EXPLANATIONS


1. (4) Let the missing number be = 133.62 – 39.3 than .5 ones are counted as
x. Then, the next integer.
= 94.32
2 4 1 1 So, 279.004 + 3.959 + 22.875
96 x of 2 3 100 5. (1) The given expression is
3 3 2 3 279 + 4 + 23 = 306
1
3 10. (1)
2x 4 5 10 1 1 4
96 100
3 3 2 3 110
5 83 27 3
2
? 10
2x 10 10 103
96 100
3 3 3 110
?
2x 10
96 100 1
3 1 1 3 4 ? 11
3 3
2x 5 2 3 3 3
4 11. (4)
3
1
4 3 3 8 3 5 5
x 6 5(2 3)3 4
? 4
2 2 2 2 2 2 2
1
3
2. (2) The given expression is 5 5 4 3 3 2
3
36 7 4 7 3 7 1
54 4
3 8 5 3
5 7 2 2
5 2 4 2
36(2.64) 12 7 3 3 2
6. (1) The given expression is:
5(2.64)
1
9 12 2 5 3
95.04 84 3 2 2 3 3 6
1 2
13.2 9 12
3
2 6 5 2 3 3 6
11.04 3 12 15
5 6 5 6
13.2 1 1 1
12 12 0
= 0.83636 3 9 27
15 27 405 81 12. (2)
1 324 325 65
2 2
3. (1) 1824 = 25 × 3 × 19 7. (1) The given expression is: ?×1 4 of 2142 30%
3 3
In order to have a perfect 1 ÷ [1 + 1 ÷ {1 + 1 ÷
of 2500 1369
square, we must have even (1 ÷ 1 ÷ 1)}]
powers for all the prime 5 14
=1 ÷ [1 + 1 ÷{1 + 1 ÷ (1)}] ?× 2142
factors. Therefore, we have to 3 3
multiply the given number by = 1 ÷ [1 + 1 ÷ {1 + 1}] 750 37
2 × 3 × 19, i.e. 114. 5
= 1 ÷ [1 + 1 ÷ 2]
?× 9996 750 37
The required number = 3
1
1824 × 114 = 207936 1 1 5 9996
2 ?× 37
3 750
4. (1) 1.7161 = 1.31 3 2 2
1 1 9996 3
2 3 3 ? 37 295.88
Value of 750 5
8. (4) 93 729 Therefore the only
13. (3)
17161–3 171.61 possible option is option (4).
(4)? = (64)3 × (8)8 ÷ (512)2
9. (3) In approximation, less
than.5 terms are counted as = (43)3 × (4 × 2)8 ÷ (44 × 21)2
= 131 3(13.1) + 2(1.31) the previous integer and more
= (4)9 × (4)8 × (2)8 ÷ (44 × 21)2
= 131 – 39.3 +2.62

________________________________________________________
487
Subhash Institute of Competitive Exams (SICE) Vijayawada
Vijaya Ph: 9030924949
________________________________________________________
Mathematics for Competitive Exams ( 20 )

= (4)9 × (4)8 × (2)8 ÷ (48 × 22) 330 = 0.09 0.06 + 0.04


= = 330
= (4)9 × (2)6 2 3 1 3 = 0.13 – 0.06
= (4)9 × (22)3 19. (2) = 0.07
0.673 0.33 3
= (4)9 × (4)3
0.67 0.67 0.33 0.33 2
2
23. (4) Let 5 5 5 5 5 5.... = x
= (4)9+3 = (4)12
0.672 0.67
Therefore, ? = 12 0.67 0.33 5x = x
0.33 0.332
14. (4) The expression can also be 0.672 0.67 0.33 0.332 5x = x2
written as:
= 0.67 – 0.33 = 0.34 x=5
(?)27 = (32)3 × (33)3 × (34)3 24. (2) Let the number be x
20. (3)
6 9 12
= (3) × (3) × (3) According to the question,
? 484 625
= (3)6+9+12 = (3)27 1 1
? 222 252 1 x
Since, (?) 27
= (3) 27 3 27
? 22 25
Therefore, ? = 3 4x 1
? 47
15. (3) 3 27
? 472
62 ? 3
21. (4) The given expression is xy x=
× 350 – × 140 27 4
100 100 +z yx
= 154 1
When x = 3 and y = 1 x=
62 × 350–?×140 36
1 3
= 154 100 xy +y x = 3 1
4 4 1 1
×x= =
21700 – 15400 = ? × 140 1 5 5 36 45
= ( 1)
6300 3 25. (1) The given expression is
=?
140
1 0.04 0.0025 0.16 81
? = 45 = 1
3 0.125 0.256 0.09 5
16. (4)
2 4 25 16 81
3 1 =
?= × × 512 3 125 256 9 5
4 2
22. (1) The given expression is
3 9
= × 256 0.3 0.3 0.3 0.2 0.2 0.2 =
4 100
(0.3 0.2)
= 192 3
0.3 0.2
3 3
=
17. (1) = 10
(0.3 0.2)
12 = 0.3
a b b a 2
0.3 0.3 26. (2) 16595 = 25 × 3 ×132
12 0.3 0.2 2
64 289 289 64 0.2 0.2 In order to get even power of
8 17 17 8
12
(0.3 0.2) prime factors, we have to
12 divide the given number by 2 ×
5 3
1 /2
a3 b3 3, i.e. 6.
2
(a b)(a2 ab b2 ) 16595 should be divided by
36 49 64 81 100 6 to make it a perfect square.
18. (2) 2 2 2 2
2 3 1 3 0.3 0.3 0.2 0.2

________________________________________________________
488
Subhash Institute of Competitive Exams (SICE) Vijayawada
Vijaya Ph: 9030924949
________________________________________________________
Mathematics for Competitive Exams ( 21 )
Section III – Chapter 2: Surds, Indices and Simplification

27. (4) 82 × 64–1 = ? = 10 × 5 = 2396 ÷ 5


1 = 50 = 479.2
As, a–m = , we get:
am 31. (2) If the expression includes 36. (1) The given expression is
? = 8 × 8 ÷ 64 1 more than one similar 7 3
operation, solve from the left
= 64 ÷ 64 hand side first, and then the 7 3
=1 other. Therefore, Rationalising the function, we
? =10000 ÷ 100 ÷ 10 get
28. (1)
7 3 7 3
= 100 ÷ 10 = ×
1 1 7 6 7 3 7 3
7 6 7 6 7 6 = 10 2
7 3
7 6 32. (4) According to the rules, when = 2 2
two numbers are multiplied 7 3
Similarly, having the same base, their
1 exponents are added. On the [ (a b) (a + b) = (a2 - b2 )]
= 6 5, other hand, when a number is
6 5
divided by another number 7 3 2 21
1
5 2, having same base, the 7 3
5 2
exponent value of the
1 denominator is subtracted 10 2 21
8 7,
8 7 from the exponent of the 4
1 numerator.
3 8
3 8 2(5 21 )
So, 41 × 44 ÷ 42 = 41 × 4(4 – 2)
4
The given expression = 41 + 2
5 21
=( –( = 43
2) – ( 2
Since 43 = 4? 37. (2)
=
On comparing, ? = 3 1 7 13
? 1 3 1
29. (2) 33. (3) After factorization, we get 5 15 25
6 52 38
? 2 2 2 2 2 2 =
Let x = a 3 b a 3 b ................... 5 15 25
2 2 2 2 6 52 25
On squaring both sides 5 15 38
3 2 2
52 14
3 3
x2 = a b a b ................... 2
=2+3 2 19 19
Again on cubing both sides 34. (1) We need to find the 38. (3)
approximate value of the given
2 2
(x2)3 = a3 b a 3 b ................... expression. The above ? 6 2 365
expression can also be written ? 36 4 365
x6 = a3bx in the simple form as:
? 9 365
x5 = a3 b ? = 4000 ÷ (400 ÷ 5) × 3 ? 374 375
x= 5 3
ab = 4000 ÷ 80 × 3 39. (2)
30. (3) According to the VBODMAS = 50 × 3
rule, division needs to be 57 9.5 724.5 1408 ?
= 150
performed before 541.5 724.5 1408 ?
multiplication, therefore; 35. (4) The above expression can
1949.5 724.5 ?
also be written as:
? = 500 ÷ 50 × 5
? 1225 35
? = 9584 ÷ 4 ÷ 5

________________________________________________________
489
Subhash Institute of Competitive Exams (SICE) Vijayawada
Vijaya Ph: 9030924949
( 22 )
________________________________________________________
Mathematics for Competitive Exams

40. (2) 41. (3) The given expression is: (0.16 0.25)2 (0.41)2
0.16 0.16 2 0.16 0.25 0.25 0.25 0.41 0.41
4 131 154 211 14
(0.16 0.25) (0.1681
4 131 154 15 0.41
(0.16)2 2(0.16)(0.25) (0.25)2 0.41
4 131 13 (0.16 0.25)

4 12 4 [ from (a+b)2 = a2 +2ab +b2]

________________________________________________________
490
Subhash Institute of Competitive Exams (SICE) Vijayawada
Vijaya Ph: 9030924949
________________________________________________________
Mathematics for Competitive Exams ( 23 )

3 RATIO, PROPORTION AND PARTNERSHIP

RATIO, PROPORTION AND PARTNERSHIP


Ratio, proportion and partnership are important from the perspective of RRB NTPC
examination. The questions based on comparison of one quantity with another are based on
the application of ratio, proportion and partnership. Let us study these concepts in detail.

RATIO
Ratio refers to a quantitative relation between two numbers or amounts or quantities. It
shows the number of times one value contains the other value or is contained within the
x
other value. If we have two quantities, say x and y, then the ratio of x to y is calculated as
y
and is written as x : y.
The numerator of the ratio is called the antecedent and the denominator is called the
consequent.

Important Characteristics and Formulae Related to Ratios


1. If both the antecedent and the consequent are multiplied or divided by the same number
(except 0), the ratio will remain the same, i.e.
a ka a a/k
= or that, =
b kb b b/k

a 5
Example: If = and we multiply both antecedent and the consequent by 7, the ratio
b 8
will remain the same as shown below:
35 5
=
56 8

________________________________________________________
Subhash Institute of Competitive Exams (SICE) Vijayawada
Vijaya Ph: 9030924949
( 24 )
________________________________________________________
Mathematics for Competitive Exams

2. Duplicate ratio refers to the ratio of the squares of Some Important Examples:
the antecedent and the consequent. Duplicate ratio 1. Sweets were distributed among 50 students. If
of x : y = x2 : y2. each boy gets 3 sweets and each girl gets 2
3. Triplicate ratio refers to the ratio of the cubes of the sweets, find out the ratio of the number of girls
and boys in the class.
antecedent and the consequent. Triplicate ratio of
x : y = x3 : y3 . Solution: Let the number of girls and boys in the
class be a and b, respectively.
4. Sub-duplicate ratio refers to the ratio of the square
roots of the antecedent and the consequent. Sub- So, a + b = 50
duplicate ratio of x : y = x : . and 2a + 3b = 120

5. Sub-triplicate ratio refers to the ratio of the cube After solving, we get a = 30 and b = 20
roots of the antecedent and the consequent. Sub- Required ratio = 30 : 20 = 3 : 2
triplicate ratio of x : y = x :
3 3
y . 2. The monthly incomes of Meera and Vinita are in
6. Inverse ratio is derived by interchanging the the ratio 4:5, their expenses are in the ratio 5 : 6.
If Meera saves `25 per month and Vinita saves
positions of the antecedent and the consequent. `50 per month, what are their respective
Inverse ratio of x : y = y : x. incomes?
7. Compound ratio refers to the ratio derived by Solution: Let Meera’ income = 4x; Vinita’s income
multiplying all the antecedents of the given ratios to = 5x
the product of all the consequents. For example, if
Therefore, Meera’s expenses = 4x 25
we are given three ratios: a : b, c : d and e : f; then
the compound ratio = (a × c × e): (b × d × f) And Vinita’s expenses = 5x 50
8. If the denominator of two ratios is same, then the The given ratio of their expenses is
ratio with larger numerator is greater than the ratio 4 x 25 5
with the smaller numerator. 5 x 50 6
9. If the antecedent and the consequent in a ratio are 6 (4x – 25) = 5(5x – 50)
both surd quantities (quantities represented as nth
24x x
root of any number), then the ratio cannot be
x = 100
represented in the form of integral numbers until
and unless both the antecedent and the consequent Thus, Meera’ income = `400; Vinita’s income
have the same surd values. = `500
3. Values of 1 rupee, 50 paise and 25 paise coins in
5
Example: If we have a ratio as , it cannot be a bag are in ratio of 13:11:7. The total number of
3 coins are 378. Find the number of 50 paise coins.
represented as the ratio of two integral numbers
Solution: Let the income of father `5x and that of
because it cannot be simplified by using any rule or
his son `3x.
property of ratio.
Therefore,
a c e g
10. If we are given that = = = = k, then Expenditure of father = 5x–1300
b d f h
a c e g Expenditure of son = 3x–900
k=
b d f h Thus,
a 5 c 10 5 x 1300 9
Illustration: Given that = = =
b 10 d 20 3 x 900 5
15 5(5x 1300) = 9(3x 900)
K= = = 0.5
30

________________________________________________________
492
Subhash Institute of Competitive Exams (SICE) Vijayawada
Vijaya Ph: 9030924949
________________________________________________________
Mathematics for Competitive Exams ( 25 )
Section III – Chapter 3: Ratio, Proportion and Partnership

x = 800 a c a b
9. Alternendo rule: If = , then =
b d c d
Therefore,
a n1 b n2
The income of father = 5×800 = `4,000 10. If we are given two proportions as and
b d1 c d2
The income of son = 3×800 = `2,400 ; then the ratio a : b : c is calculated as:
a : b : c = (n1 × n2) : (d1 × n2) : (d1 × d2)
PROPORTION
Some Important Examples:
A proportion simply implies that one ratio is equal to
another. The proportion is signified by double colons. For a 5 b 16
1. There are two ratios: = , and = . Find the
example, ratio 6 : 8 is the same as ratio 3 : 4. This can b 8 c 13
ratio a : b : c.
be written as 6 : 8 : : 3 : 4 or as:
Solution: a : b : c = (5 × 16) : (8 × 16) : (8 × 13)
3 6 = 10 : 16 : 13
=
4 8
2. If X : Y = 6 : 11 and Y : Z = 9 : 13, then find X : Y :
In general, if we are given a proportion a : b :: c : d, then Z.:
it can be stated that a is to b as c is to d. If we are given X 6 Y 9
a proportion, a : b :: c : d; then a, b, c and d are called as Solution: Given, and
Y 11 Z 13
first, second, third and fourth proportionals respectively.
LCM of 11 and 9 = 99
a and d are called extremes, whereas b and c are called
6 9 54 9 11 99
means. and
11 9 99 13 11 143
Important Characteristics and Formulae Hence, X : Y : Z = 54 : 99 : 143
Related to Proportions 3. If P : Q = 3:4, Q : R = 8 : 10 and R : S = 15 : 17,
a c then find P : Q : R : S.
1. Since = , the product of extremes is equal to the
b d
Solution: As per the given problem:
product of means, i.e. ad = bc
P : Q = 3 × 8 : 4 × 8 = 24 : 32
2. If a proportion is such that a : x : :x : b, then x is
called the mean proportional or the second Q : R = 8 × 4 : 10 × 4 = 32 : 40
proportional of a and b. P : Q : R = 24×3 : 32×3 : 40×3
3. If a proportion is such that a : b :: b : x, then x is = 72 : 96 : 120
called the third proportional of a and b.
R:S = 15×8 : 17×8 = 120 : 136
a c a+b c+d
4. Componendo rule: If = , then = Therefore,
b d b d
a c a– b c d
P : Q : R : S = 72 : 96 : 120 : 136 = 9 : 12 : 15 : 17
5. Dividendo rule: If = , then =
b d b d a b c a b c
4. If , then =?
a 7 8 9 b
6. Componendo and Dividendo (C and D) rule: If =
b a b c
Solution: Let x
c a b c d 7 8 9
, then =
d a b c d
a = 7x, b = 8x and c = 9x
a c a a c a c c
7. If = , then = = = a b c 7x 8x 9x
b d b b d b d d
b 8x
a c b d 24 x
8. Invertendo rule: If = , then = 3
b d a c 8x

________________________________________________________
493
Subhash Institute of Competitive Exams (SICE) Vijayawada
Vijaya Ph: 9030924949
( 26 )
________________________________________________________
Mathematics for Competitive Exams

PARTNERSHIP Some Important Examples:


1. Ramesh and Suresh started a business of tiles
Partnership is generally defined as a legal agreement
investing ` 30000 and ` 50000, respectively.
between two or more persons who agree to share profits Find the share of Ramesh if they had an annual
or losses incurred by a business entity. Each person in profit of ` 44000 after 1 year?
the partnership is called a partner. Working partners are Solution: Ratio of investment of Ramesh and
those who invest their money and also actively manage Suresh = 30000 : 50000 = 3: 5
the operations of the business entity. In contrast, 3
Ramesh’s share in profit = (44000 × )
sleeping partner refers to a kind of partner that invests 3 5
money but does not actively manage the company. 3
= ` (44000  )
If there are n partners who invest I1, I2, .... In amounts for 8
t1, t2, ...... tn time periods, then the PSR or LSR of n = ` 16500
partners is calculated as: 2. Reema started business of bread manufacturing
with ` 4500. After 4 months, Anshu joins Reema
Profit (or loss) of Partner 1: Profit (or loss) of Partner
as his business partner. After 1 year, the profits
2: ............... Profit (or loss) of Partner n are divided in the ratio of 5 : 6. How much did
Anshu contribute?
= I1 t1 : I2t 2 : …………..: Intn
Solution: Let Anshu invested ` x in the partnership
business.
Short-tricks Formulae
1. If n partners have invested amounts in ratio I1 : I2 : I3 According to the question,
: ……….. : In and the ratio of their profits is P1 : P2 : P3 Investment of Reema : Investment of Anshu =
: ……….. : Pn, then the ratio of the time periods for Reema’s share of profit : Anshu’s share of profit
which they have invested is given by:
(4500 × 12) : (x × 8) = 5 : 6
P1 P2 P
: : ……… : n
I1 I2 In
x
Illustration: Varun, Tarun and Aarti invested in a
x = ` 8100
project in ratio 6 : 7 : 8. If their profit sharing ratio is
3 : 4 : 7; then calculate the ratio of the time periods 3. Purab and Sonam invested capitals in the ratio 2 :
for which they made the investments. 7 and the timing of their investments is in the
ratio 1 : 3. In what ratio would their profits be
Solution: Here, I1 : I2 : I3 = 6 : 7 : 8 distributed?
The PSR = 3 : 4 : 7 Solution: Let the capitals of Purab and Sonam be
P P P 2x and 7x and their time of investments be y and
The ratio of the time periods = 1 : 2 : 3 3y, respectively.
I1 I2 I3
According to the question,
3 4 7 1 4 7
= : : = : : = 28 : 32 : 49
6 7 8 2 7 8 Ratio of investments = Ratio of profits
2. If there are n partners whose profit sharing ratio is Ratio of profits = (2x × y) : (7x × 3y) = 2 : 21
P1 : P2 : P3 : ……….. : Pn and they invest their money
4. Neha and Preeti are two partners in a business.
for time periods t1 : t2 : ….. : tn, then the ratio of their Neha invests `15000 and Preeti `25000. Neha is
investments is given by : the working partner and gets 20% of the profit for
her contribution in the management of the firm.
P1 P2 P
: : ……… : n Preeti is the sleeping partner. If the profit is Rs.
t1 t2 tn
475, find the share of Preeti.
Solution: Neha’s investment = `15,000; Preeti’s
investment = `25,000

________________________________________________________
494
Subhash Institute of Competitive Exams (SICE) Vijayawada
Vijaya Ph: 9030924949
________________________________________________________
Mathematics for Competitive Exams ( 27 )
Section III – Chapter 3: Ratio, Proportion and Partnership

Total profit = `475 Now, the remaining profit has to be divided


between Neha and Preeti. The ratio of their
Neha’s share for management = 475 × 20 investment is
100
15000 : 25000 = 3 : 5
= `95
5
Therefore, Preeti’s share = 380× = `237.5
8

________________________________________________________
495
Subhash Institute of Competitive Exams (SICE) Vijayawada
Vijaya Ph: 9030924949
( 28 )
________________________________________________________
Mathematics for Competitive Exams

PRACTICE QUESTIONS
Answer the following questions by selecting the most appropriate option.
ratio of the ages of Arati and Kiran was 2 : 3,
1. Akhil and Bharat entered into partnership with
respectively. Kiran's present age is 20 years. Find
capitals in the ratio of 5 : 4. After 4 months, Akhil
Rakshak's present age.
1 1 (1) 45 years (2) 50 years
withdrew of his capital and Bharat withdrew
5 4 (3) 35 years (4) 40 years
of his capital. The gain at the end of 9 months
was ` 1065. Find out Bharat’s share in this profit. 9. Vishal invested 3/11 th part of his monthly
(1) ` 320 (2) ` 465 income in stocks share and that of 1/4 th part in
(3) ` 275 (4) ` 185 mutual funds. He spent the remaining amount on
domestic needs and apparel in the ratio 5:3,
2. The ratio among investments of A, B and C in a respectively. The expenditure on domestic needs
project is 6 : 7 : 8. If the ratio among their profits was ` 2100. What is his annual income?
is 3 : 4 : 7, the time for which they made the (1) ` 84,480 (2) ` 85,480
investments is in the ratio of (3) ` 86,480 (4) ` 83,480
(1) 14 : 15 : 19
(2) 12 : 13 : 27 10. In a piggy bank, there are 1 rupee, 50 paise and
(3) 56 : 35 : 69 25 paise coins. The respective ratio among their
(4) 28 : 32 : 49 numbers is 10 : 8 : 5. In the bank, there is a total
sum of ` 976. How many 25 paise coins are there
3. Find out the minimum number of sweets a person in the piggy bank?
should have to distribute among five children in (1) 256 (2) 360
1 1 1 1 1 (3) 320 (4) 640
the ratio of .
2 5 8 10 4
11. The ratio between the three angles of a
(1) 40 (2) 43
quadrilateral is 3 : 5 : 9. The value of the fourth
(3) 47 (4) 51
angle of the quadrilateral is 71º. What is the
4. A sum of ` 529 is to be divided among P, Q and R difference between the largest and the smallest
in such a way that 4 times of P's share, 5 times of angles of the quadrilateral?
Q's share and 8 times of R's share are all equal. (1) 82º (2) 106º
Find out the share of P. (3) 102º (4) 92º
(1) ` 170 (2) ` 230
12. The ratio between the incomes of Ram and Shyam
(3) ` 195 (4) ` 290
are 3:2 and their expenditures are in the ratio 5:3.
5. The average age of a man and his son is 30 years. If each of them saves ` 2, 000, what would be
The ratio of their ages four years ago was 10:3 their incomes?
respectively. What is the difference between the (1) ` 6,000 and ` 4,000
present ages of the man and his son? (2) ` 5,000 and ` 10,000
(1) 28 years (2) 16 years (3) ` 9,000 and ` 8,000
(3) 26 years (4) 44 years (4) ` 12,000 and ` 8,000
6. The ratio between number of boys and girls in a 13. The ages of Sangeeta and Mohan are in the ratio
school is 5 : 3. If 30% of the boys and 40% of the of 7 : 2. Fifteen years hence, their ages would be
girls are scholarship holders, the percentage of in the ratio 2 : 1. What was Sangeeta’s age when
students who do not get scholarship is Mohan was born?
(1) 23.85% (2) 38.2% (1) 25 years (2) 30 years
(3) 66.25% (4) 57.82% (3) 35 years (4) 40 years
7. The area of a rectangular field is 3584 m2 and its 14. The original salaries of Manohar and Ramit were
length and breadth are in the ratio 7:2. What is in the ratio of 2 : 3. After their yearly appraisal,
the perimeter of the field? the salary of each increased by ` 4,000. The new
(1) 246 m (2) 292 m ratio becomes 40 : 57. What is Ramit's salary now
(3) 286 m (4) 288 m after the increment?
(1) `17,000 (2) ` 20,000
8. The present age of Rakshak is twice the present
(3) ` 25,000 (4) ` 38,000
age of Sonal. Five years hence, Sonal's age will be
twice the present age of Arati. Five years ago, the

________________________________________________________
496
Subhash Institute of Competitive Exams (SICE) Vijayawada
Vijaya Ph: 9030924949
________________________________________________________
Mathematics for Competitive Exams ( 29 )
Section III – Chapter 3: Ratio, Proportion and Partnership

15. By mistake instead of dividing 117 among P, Q 23. The annual income and expenditure of a man and
1 1 1 his wife are in the ratios of 5 : 3 and 3 : 1,
and R in the ratio , , , it was divided in the
2 3 4 respectively. If they decide to save equally and
ratio of 2 : 3 : 4. Who gains the most and by how find a balance of `4,000 at the end of the year,
much? their incomes were
(1) P, ` 28 (2) Q, ` 3 (1) ` 5,000, 3,000
(3) R, ` 20 (4) R, ` 25 (2) ` 6,000, 4,000
(3) ` 3,000, 2,000
16. A husband and a wife have monthly incomes in (4) ` 5,000, 6,000
the ratio 5:6 and monthly expenditure in the ratio
3:4. If they save `1,800 and ` 1,600, respectively, 2
24. The sum of 5% of x and 4% of y is of the sum of
find the monthly income of the wife 3
(1) ` 3,400 (2) ` 2,700 6% of x and 8% of y. The ratio of x to the y is
(3) ` 1,720 (4) `7,200 (1) 2 : 3 (2) 3 : 2
(3) 3 : 4 (4) 4 : 3
17. The weights of Mohan and Meera are in the ratio
of 7:8 and their total weight is 120 kg. After 25. Neha and Preeti are two partners in a business.
joining a gym, Mohan reduces by 6 kg and the Neha invests `15000 and Preeti ` 25000. Neha is
ratio between their weights becomes 5 : 6. So, the working partner and gets 20% of the profit for
Meera has reduced weight by her contribution in the management of the firm.
(1) 2 kg (2) 4 kg Preeti is the sleeping partner. If the profit is
(3) 3 kg (4) 5 kg ` 475, find the share of Preeti.
18. The incomes of A and B are in the ratio of 8 : 11 (1) ` 237.5 (2) ` 257.5
and their expenditure is in the ratio of 7 : 10. If (3) ` 247.5 (4) ` 238.5
each of them saves `500, what are their incomes 26. Four years ago, the ratio of the ages of Sonal and
and expenditures? Komal was 11:14. Four years hence, their ages
(1) ` 8,000, ` 7,500, ` 6,500, ` 5,000 will be in the ratio of 13:16. The present age of
(2) ` 5,000, ` 6,500, ` 3,500, ` 6,000 Sonal’s is
(3) ` 6,000, ` 4,500, ` 8,500, ` 7,000 (1) 48 years (2) 26 years
(4) ` 4,000, ` 5,500, ` 3,500, ` 5,000 (3) 44 years (4) 28 years
19. A and B invest in the ratio 3 : 5. After 6 months, C 27. Tarun, Raman and Yash entered into a
joins the business investing an amount equal to 3
B’s. At the end of the year, what will be the ratio partnership in a business. Tarun got of the
5
of the profits? profit. The remaining profit was distributed
(1) 3 : 5 : 2 (2) 3 : 5 : 5 equally between Raman and Yash. If Yash got
(3) 6 : 10 : 5 (4) 8 : 10 : 5 `400 less than Tarun, the total profit was
20. A particular amount of money is to be divided (1) ` 1600 (2) ` 1200
among Sita, Gita, Rita and Mita in the ratio of (3) ` 1000 (4) ` 800
5 : 2 : 4 : 3. If Rita gets `1000 more than Mita, 28. Meera, Smitha and Shiela invest `1,000, ` 4,000
what is Gita's share? and ` 5,000, respectively in a business. At the
(1) ` 500 (2) `1500 end of the year, the balance sheet shows a loss of
(3) ` 2000 (4) ` 2500 20% of the total initial investment. Find the share
21. Two numbers are respectively 120% and 150% of loss of Smitha.
more than a third number. The ratio of the two (1) ` 1,000 (2) ` 200
numbers is (3) ` 800 (4) ` 1,200
(1) 2:5 (2) 3:5 29. A, B and C together started a business. The ratio
(3) 22:25 (4) 6:7 of investment of A and B is 7:8 and that of B and
22. The present ages of A and B are in the ratio of C is 4:9. B gets a share `7104 in annual profit.
4 : 3. If 4 years ago, the ratio of their ages was What is C’s share in the profit?
2 : 1, the present age of A is (1) ` 16984
(1) 8 years (2) 10 years (2) ` 13984
(3) 12 years (4) 15 years (3) ` 15984
(4) ` 17984

________________________________________________________
497
Subhash Institute of Competitive Exams (SICE) Vijayawada
Vijaya Ph: 9030924949
________________________________________________________
Mathematics for Competitive Exams ( 30 )

30. Shrikanth and Vividh started a business investing 36. Values of 1 rupee, 50 paisa and 25 paisa coins in
amounts of `1,85,000 and `2,25,000, respectively. a bag are in the ratio of 13:11:7. The total
If Vividh’s share in the profit earned by them is numbers of coins are 378. Find the number of 50
` 9,000, what is the total profit earned by them paisa coins.
together? (1) 132 (2) 128
(1) ` 17,400 (2) ` 16,400 (3) 136 (4) 133
(3) ` 16,800 (4) ` 17,800
37. In a school, there were 1554 students and the
31. Present ages of father and the son are in the ratio ratio of the number of boys and girls was 4 : 3.
of 6:1 respectively. Four years hence, the ratio of After a few days, 30 girls joined the school but
their ages will become 4:1 respectively. What is few boys left; as a result, the ratio of the boys and
the son's present age? girls became 7 : 6. The number of boys who left
(1) 10 years (2) 6 years the school is
(3) 4 years (4) 8 years (1) 84 (2) 74
(3) 86 (4) 76
32. Four years ago the ratio between the ages of
Taani and Ananya was 4:5. The ratio between 38. 60 kg of an alloy A is mixed with 100 kg of alloy
their present ages is 14:17. What will be Ananya's B. If alloy A has lead and tin in the ratio 3 : 2 and
age 5 years hence? alloy B has tin and copper in the ratio 1 : 4, the
(1) 39 years (2) 33 years amount of tin in the new alloy is
(3) 38 years (4) 28 years (1) 80 kg (2) 53 kg
(3) 24 kg (4) 44 kg
33. In a library, the ratio of the story books and the
other books is 7 : 2 and there are 1,512 story 39. A particular amount of money is to be distributed
books. Due to collection of some more story among fireman 1, fireman 2 and fireman 3 in the
books, the said ratio becomes 15 : 4. The number ratio of 2:7:9. The total share of fireman 1 and
of story books collected is fireman 2 is equal to the share of fireman 3. Find
(1) 100 (2) 97 the difference between the shares of fireman 1
(3) 108 (4) 205 and fireman 2.
(1) ` 5,000
34. 12 years ago, the ages of A and B were in the
(2) ` 7,500
ratio of 15:13, respectively. 19 years from now
(3) ` 9,000
the respective ratio of their ages will be 23:22.
(4) Information inadequate
What is the sum of their ages at present?
(1) 57 years (2) 54 years 40. Suppose you are given the responsibility of
(3) 52 years (4) 59 years calculating the daily wages of labour in a
construction company. You need to divide `700
35. Populations of two villages X and Y are in the
among the plumber, painter and electrician in a
ratio of 5:7 respectively. If the population of
manner that the ratio of the amounts the plumber
village Y increased by 25,000 and the population
and for the painter is 2:3 and that of the painter
of village X remains unchanged, the respective
and electrician is 4:5. Find the amounts received
ratio of their populations become 25:36. What is
by each.
the population of village X?
(1) `150, `250, `300 (2) `160, `240, `300
(1) 6,25,000 (2) 6,75,000
(3) `150, `250, `290 (4) `150, `240, `310
(3) 8,75,000 (4) 9,00,000

________________________________________________________
498
Subhash Institute of Competitive Exams (SICE) Vijayawada
Vijaya Ph: 9030924949
________________________________________________________
Mathematics for Competitive Exams ( 31 )
Section III – Chapter 3: Ratio, Proportion and Partnership

ANSWERS AND EXPLANATIONS


1. (2) Let Akhil’s and Bharat’s Therefore, let the present age of Therefore, the perimeter of the
monthly installments be 5x and the man is x years and his son’s rectangular field = 2 l + 2 b = (2
4x, respectively. age is (60 x) years. × 112 + 2 × 32) = 288 m
Ratio of investments of Akhil Four years ago, 8. (2) Let the present age of Arati
and Bharat = = x years
x 4 10
5x 5 4x 5 60 x 4 3 Then, the present age of Sonal
(5 x 9) : (4 x 9) = (2x years (as five years
5 5
3x 12 = 560 x later Sonal’s age will be twice
= (45x – 5x) : (36x – 5x) the age of Arati)
x = 44
= 40 : 31 Five years ago, the ratio of the
Therefore, the man’s age is 44
ages of Arati and Kiran is
Bharat’s share in profit years and the son’s age is
31 x 5 2
= = ` 465 = (60 years
71 15 3
= 16
2. (4) The ratio of investments of x = 15 years
A, B and C = 6 : 7 : 8 The difference between their
Therefore, Arati’s present age is
ages = 44 16 = 28 years
And the ratio of their profits 15 years.
=3:4:7 6. (3) Let there are 100 students
Now, the Sonal’s present age
in the school
The ratio of the time periods = (2 × 15 years = 25 years
3 4 7 1 4 7 Therefore, number of boys =
= : : = : : Therefore, Rakshak’s Age
6 7 8 2 7 8 500 and number of girls = 300
= (2 × 25) years = 50 years
= 28 : 32 : 49 Number of boys get scholars
9. (1) Let monthly income = ` x
3. (3) The sweets are distributed 30
= 500 = 150
among children in ratio of 100 Monthly income invested in
1 1 1 1 1 3
: : : : Number of girls get scholars stocks = x
2 5 8 10 4 11
40
= = 120
LCM of (2, 5, 8, 10 and 4) = 40 100 Monthly income invested in
1
Minimum number of sweets Number of students do not get mutual funds = x
4
required to distribute scholars = 800 – (150 + 120)
40 40 40 40 40 = 530 Amount spent on domestic
= needs: Amount spent on
2 5 8 10 4 Required percentage
= 20 + 8 + 5 + 4 + 10 = 47. apparel = 5:3
530
= = 66.25%
4. (2) 4P = 5Q = 8R 800 2100 5
=
3
4P 4P 7. (4) Area of rectangle = l × b
Q= and R = Amount spent on apparel
5 8
3584 = `1,260
b=
P + Q + R = 528 l
Now,
4P 4P l 7
P+ = 529 3 1
5 8 b 2 x= x + x + 3360
11 4
By putting the value of b, we get
P’s share = x = `7,040
92 l = 112
= ` 230 Total annual income = 7040 ×
b = 32 12 = `84,480
5. (1) The sum of the ages of man
and his son = 2 × 30 = 60 years

________________________________________________________
499
Subhash Institute of Competitive Exams (SICE) Vijayawada
Vijaya Ph: 9030924949
________________________________________________________
Mathematics for Competitive Exams ( 32 )

10. (3) Since, the ratio among the x = 2y 15. (4) The original ratios were
number of coins is 10:8:5.
Now, putting the value of x in 1 1 1
: :
Let equation 2 3 4
The number of coins of 1 rupee 2000 = 3x – 5y Therefore, P’s original share
= 10 ; the number of coins of 1 12
50 paise = 8x; the number of 2000 = (3 × 2y) – 5y = 117 = ` 54
2 13
coins of 25 paise = 5x y = 2000
Q’s original share =
8x x = 4000 1 12
Value of 50 paise coins =
2 3 13
(as there two 50 paise coins in The incomes of Ram and Shyam
1 rupee) are `12,000 and `8,000, = ` 36
respectively.
5x S’s original share =
Value of 25 paise coins = 13. (1) Let Sangeeta’s present age 1 12
4 117
be 7x years and Mohan’s 4 13
(as there four 25 paise coins in
present age be 2x years.
1 rupee) = ` 27
Then, 15 years hence, their
Now, Now, by mistake, the amount
ages will be:
5x was divided in the ratio of 2 : 3 :
10x + 4x + = 976 7 x 15 2 4.
4
2 x 15 1
x = 64 Let P’s new share =2x; Q’s new
(7x +15) = 2(2x + 15) share = 3x; R’s new share = 4x
So, the number of 25 paise
coins = 5 × 64 = 320 7x + 15 = 4x +30 2x + 3x + 4x = 117

11. (3) Let the first angle = 3 º; the x=5 x =13


second angle = 5 º; third angle Therefore, Sangeeta’s present P’s new share = 2 × 13 = ` 26
=9 º age = 7 × 5 = 35 years and
Q’s new share = 3×13 = ` 39
Mohan’s present age = 2 × 5 =
The total sum of all the angles
10 years R’s new share = 4×13 = ` 52
of a quadrilateral is 360º.
Sangeeta was 25 years old So, it is clearly visible that R
Now,
when Mohan was born because gains the most and by ` 25.
3x 5x + 9x + 71 = 360 the difference between their
16. (4) Let the monthly income of
ages is 25 years.
x = 17 husband ` 5x and that of his
14. (4) Let Manohar’s salary = 2x; wife ` 6x.
Therefore,
Ramit’s salary = 3x
Expenditure of husband
First angle = 3 × 17 = 51
If their salary increases by = 5x – 1800
Second angle = 5 × 17 = 85 2 x 4000 40
4,000, then Expenditure of wife = 3x –1600
Third angle = 9 × 17 = 153 3 x 4000 57
5 x 1800 3
Fourth angle = 71 57 (2x + 4000) = 40 (3x + Thus, = 4(5x +
6 x 1600 4
4000)
The difference between the 1800) = 3(6x 1600)
largest and smallest angles = 114x + 228000 = 120x +
160000 = 20x 7200 = 18x 4800 = x
153 51 = 1200
= 102º 6x = 68000
Therefore, the income of wife =
12. (4) Let their incomes be 3x and 3x = 34000 6 × 1200 = `7,200
2x and their expenditures be 5y
and 3y, respectively. Therefore, Ramit’s present 17. (2) Let Mohan’s initial weight
salary = `(34,000 + 4,000) = 7x kg
2000 = (3x – 5y) = (2x – 3y)
= ` 38,000 Meera’s initial weight = 8x kg
= 3x – 2x = 5y 3y

________________________________________________________
500
Subhash Institute of Competitive Exams (SICE) Vijayawada
Vijaya Ph: 9030924949
________________________________________________________
Mathematics for Competitive Exams ( 33 )
Section III – Chapter 3: Ratio, Proportion and Partnership

Therefore, 7x + 8x=120 x = 1000 5x 4y 2 6x 8 y


x=8 Therefore, Gita’s share = 2 × 100 100 3 100 100
1000 = ` 2,000
Mohan’s weight = 7 × 8 = 56 kg 15x + 12y = 12x + 16y
21. (3) Let the third number be x.
Meera’s weight = 8 × 8 = 64 kg 3x = 4y
First number = x + 120% of x
Let Meera reduces by kg. = 220 % of x x 4
Thus, y 3
Second number = x + 150% of x
56 6 5 = 250% of x Therefore, x : y = 4:3
64 y 6
Ratio of first two numbers 25. (1) Neha’s investment =
6(50) = 5(64 y) 220% of x 22 `15,000; Preeti’s investment
= = 22 : 25 = ` 25,000
300 = 320 5y 250% of x 25
22. (1) Let A’s present age be 4x Total profit = ` 475
y=4
years and B’s present age be 3x Neha’s share for management
Thus, Meera has reduced by years 20
4 kg. = 475 ×
Then, 4 years ago, their ages 100
18. (4) Let their incomes be 8x and will be: = ` 95
11x and their expenditures be Now, the remaining profit has to
7y and 10y, respectively. 4x 4 2
3x 4 1
be divided between Neha and
500 = (8x y) = (11x y) Preeti. The ratio of their
6x 8 = 4x 4 investment is
= 8x–11x = –10y+7y
2x = 4 15000 : 25000= 3 : 5
x=y
x=2 Therefore, Preeti’s share =
Now, putting the value of x in 5
equation Therefore, A’s present age = 380× = ` 237.5
8
2×4 = 8 years
500 = 8x y
26. (1) Let the ages of Sonal and
23. (1) Let their incomes be 5x and
500 = (8×y) – 7y Komal four years ago be 11x
3x and their expenditures be 3y
years and 14x years,
y= 500 and y, respectively.
respectively.
x = 500 2000 = (5x – 3y) = (3x y) =
According to the given problem,
5x – 3x = 3y – y
The incomes of Ram and Shyam 11 8 13
are ` 4,000 and ` 5,500, x=y
14 8 16
respectively. Their expenditures Now, putting the value of x in
are ` 3,500 and ` 5,000, equation 16(11x + 8) = 13(14x + 8)
respectively.
2000 = 5x y 176x + 128 = 182x + 104
19. (3) Let A’s investment = ` 3x
2000 = (5×y) – 3y x=4
B’s investment = ` 5x
y = 1000 Therefore, Sonal’s present age
= C’s investment
= (11 × 4) + 4 = 48 years
Ratio of equivalent investment x = 1000
27. (3) Let the total profit =` x
of A, B and C = (3x × 12):(5x ×
The incomes of man and his
12) : (5x × 6) = 36x : 60x : 30x 3x
wife are ` 5,000 and ` 3,000, Tarun’s share in profit =
= 6 : 10 : 5 5
respectively.
20. (3) Let Sita’s share = 5x; Gita’s x
24. (4) As per the given problem, Raman’s share in profit =
share = 2x; Rita’s share = 4x;
5
Mita’s share = 3x 2
5 4
3 x
If Rita gets `1000 more than Yash’s share in profit =
6 8 5
Mita, then 4x – 3x = 1000

________________________________________________________
501
Subhash Institute of Competitive Exams (SICE) Vijayawada
Vijaya Ph: 9030924949
________________________________________________________
Mathematics for Competitive Exams ( 34 )

As per the given problem, (6x + 4) = 4(x + 4) 35. (1) Let the population of village
X = 5 and village Y = 7
3x x 6x + 4 = 4x + 16
– = 400
5 5 If the population of village Y
6x – 4x = 16 increased by 25,000 and the
x = 1,000 population of village X remains
x=6
28. (3) Sum of the investments unchanged, then
Therefore, son’s age = 6 years
= 1000 + 4000 + 5000 = 5x 25
`10,000 32. (1) Let Tanni’s present age be 7 x 25000 36
14 x years and Ananya’s
20 x = 1,25,000
Total loss = 10000 present age be 17x years
100
= ` 2,000 Then, 4 years ago, their ages Therefore, the population of
were village X = 5 × 1,25,000 =
Ratio of the investments of 6,25,000
Meera, Smitha and Shiela = 14 x 4 4
1000:4000:5000 = 1:4:5 17 x 4 5 36. (1) Since ratio of their value is
13 : 11 : 7.
4 5(14x 4(17x
Share of loss of Smitha = × Ratio of their numbers is 13 :
10
70x – 20 = 68x 11 × 2 : 7 × 4 = 13 : 22 : 28
2000 = ` 800 belong to the
category given at the second 70x – 68x = 20 Sum of the ratio = 13 + 22 + 28
place. = 63
x=2
29. (3) Let the total profit = P Therefore, Number of 50-paise
Therefore, Ananya’s present age
B’s share in annual profit 22
= 17 × 2 = 34 years coins = × 378 = 132
= 7,014 63
Ananya’s age after 5 years
4 3
P = 7104 = 34 + 5 = 39 years 37. (4) Number of girls = × 1554
13 7
33. (3) Number of other books in = 666
P = ` 23, 088
2 Number of boys = 1554 – 666
Therefore, C’s share in annual the library = 1512 = 432
7 = 888
9
profit = × 23088 Let additional story books be x Let the number of boys who left
13
= ` 15,984 the school be X
(1512 + x) : 432 = 15 : 4
30. (2) Shrikant’s investment: x = 108 888 X 7
Then = 5328
Vividh’s investment = 185000 : 34. (3) Let the ages of A and B 12 666 30 6
225000 years ago be 15x years and 13x 6X = 4872 X = 76
185 years, respectively. 38. (4) Amount of tin in alloy A =
=
225 Then, 2
60 = 24 kg
Let total profit = ` P 5
15 x 12 19 23
Vividh’s profit = ` 9,000 13 x 12 19 22 Amount of tin in alloy B =
1
225 100 = 20 kg
P = 9,000 22(15x + 31) = 23(13x + 31) 5
410
330x + 682 = 299x + 713 Amount of tin in new alloy = 24
P = `16400 + 20 = 44 kg
x=1
31. (2) Let father’s age be 6 x years 39. (4) Let the amount to be
and son’s age be years. Now, the present age of A = 15 distributed = `
+ 12 = 27 years; the present
Then, 4 years hence, their ages Sum of ratio = 2:7:9 = 18
age of B = 13 + 12 = 25 years
will be
The sum of the ages of A and B 2 x
6x 4 4 Share of fireman 1 = x
= 27 + 25 = 52 years 18 9
x 4 1

________________________________________________________
502
Subhash Institute of Competitive Exams (SICE) Vijayawada
Vijaya Ph: 9030924949
________________________________________________________
Mathematics for Competitive Exams ( 35 )
Section III – Chapter 3: Ratio, Proportion and Partnership

7 40. (2) Total amount = `700 Amount received by painter


Share of fireman 2 = x
18 12
New ratio of the shares of three = × 700 = ` 240
of them = 8 : 12 : 15 35
9 x
Share of fireman 3 = x
Amount received by electrician
18 2 Sum of ratio = 35
x 7x x 15
Therefore, = × 700 = ` 300
9 18 2 35
Amount received by plumber
There is no conclusion. Thus,
8
the information provided here is = × 700 = `160
inadequate. 35

________________________________________________________
503
Subhash Institute of Competitive Exams (SICE) Vijayawada
Vijaya Ph: 9030924949
________________________________________________________
Mathematics for Competitive Exams ( 36 )

4 PERCENTAGE AND ITS APPLICATIONS

PERCENTAGE
The word “per cent” means “percentum”, which means “per hundred”. A percentage is a
fraction with denominator as hundred. It is denoted by the symbol ‘% ’.
To express % equivalent to fraction:
x
x%
100

45
For example, % equivalent of 45% will be
100

a
To express as a percentage,
b

a a
100
b b

35 35
For example, as a percentage will be equivalent to × 100 = 70%
50 50
For quick calculations under percentages, the fraction equivalents for various percentages
are given in the table below:
Fraction Percentage
1
100%
1
1
50%
2
1
33.33%
3
1
25%
4
1
20%
5
1
16.66%
6

________________________________________________________
Subhash Institute of Competitive Exams (SICE) Vijayawada
Vijaya Ph: 9030924949
________________________________________________________
Mathematics for Competitive Exams ( 37 )
Section III – Chapter 4: Percentage and its Applications

Fraction Percentage If the price of a commodity decreases by Q%, then


1 the increase in consumption so as not to decrease
14.28%
7 the expenditure is:
1 Q
12.5% [ × 100] %
8 100 Q

1
11.11% Results on Population
9
Let the current population of a town be P which
1
10% increases at the rate of R% per annum, then:
10
n
1 R
9.09% 1. Population after n years = P 1
11 100

1 P
8.33% 2. Population n years ago = n
12 R
1
100
Some Important Points to Remember
To calculate a% of b: Results on Depreciation
Let the present value (PV) of a machine be P and let
a ab b
( )×b= =( ) × a = b% of a it depreciate at the rate of R% per annum. Then:
100 100 100
To find what percentage of a is b: 1. Value of the machine after n years
n
b R
× 100 =P 1
a 100

To calculate the percentage change in value, 2. Value of the machine n years ago
change P
% change = × 100 = n
initial value R
1
100
a
To increase or decrease N by a% = N ( 1 ± )
100
Some Important Examples
If the value of a certain number N increases or
1. The difference between 78% of a number and 3/4
decreases by a%, then the percentage reduction or of the same number is 108. What is 41% of that
increment needed to bring it back to the original number?
100x Solution: Let the number be x, then
point is %.
100 x
3
8% of a number of a same number=108
If there is a successive percentage increase of a % 4
and b%, then the effective percentage increase is
78 3
given by, 108
100 4
ab 312 x 300 x
(a + b + )% 108
100 400
If the price of a commodity increases by P%, then the
108 400
reduction in consumption so as not to increase the 3600
12
expenditure is given by:
41% of a number
P
[ × 100] % 41
100 P = 3600 1476
100

________________________________________________________
505
Subhash Institute of Competitive Exams (SICE) Vijayawada
Vijaya Ph: 9030924949
________________________________________________________
Mathematics for Competitive Exams ( 38 )

2. Rohit spends 50% of his monthly salary on rent Using successive percentage formula,
and food. He spends 50% of the remaining in
child’s education and 50% of the remaining in 25b
25 + b + =5
other expenses. At the end of the month, he saved 100
` 3000. Find his monthly salary.
1.25b = –20 b = – 16
Solution: Let Rohit’s salary is x
Percentage decrease in consumption = 16%
So, expense in rent and food = x/2 remaining
amount = x/2 6. Ashwani has a certain amount of money with him.
He can buy either 60 apples or 40 mangoes. He
Expense in education is = (x/2)/2 = x/4 wants to spend only 70% of his money. So, he
buys 14 mangoes and some apples. Find the
Remaining amount = x/2 – x/4 = x/4 number of apples purchased by Ashwani.
Other expenses = (x/4)/2 = x/8 Solution: Amount required to buy either 60 apples
Remaining amount = x/4 – x/8 = x/8 or 40 mangoes = 100% of available amount

x/8 = 3000 Since 14 is 35% of 40

Therefore, x = 24,000 Amount spent on buying 14 mangoes = 35% of


available amount
3. The price of an article is increased by 50%, then
20% and later on by 10%. What is the overall Remaining amount = (70% 35%) of available
percentage increase in the price of an article? money = 35% of available amount
Solution: Overall percentage increase: Number of apples that can be purchased with 35%
of available amount = 35% of 60 = 21
100 50 100 20 100 10
= 1 100 7. Due to 40% increase in price, 2 kg less sugar is
100 100 100
available for ` 280. What should be the quantity
= (1.5 × 1.2 × 1.1 1) × 100 of sugar purchased so that there is no change in
expenditure?
= (1.98 – 1) × 100 = 0.98 × 100 = 98%
Solution: Let P be the initial price and Q be the
4. The length of a rectangle increases by 12% and initial quantity of sugar.
breadth decreases by 13%. Find the percentage
change in the area of a rectangle. New price after 40% hike in price = P + 40% of P
7
Solution: Using the successive percentage change = 1.4P = P
5
formula, we have
In order to have expenditure unchanged, the new
% change in area of rectangle
5
12 ( 13)
quantity has to be Q
= 12 13 % 7
100
Decrease in quantity
= (–1 –1.56)%
5 2
= –2.56% =Q Q= Q
7 7
The negative sign indicates the decrease in area. 2
Q = 2 kg Q = 7 kg
Area decreases by 2.56%. 7

5. The price of petrol increases by 25%, but the 5


New quantity = 7 = 5 kg
expenditure on petrol increases by 5%. What is 7
the percentage increase or decrease in
consumption?
Solution: For the same consumption, if price
increases by 25%, then the expenditure also
increases by 25%. Since expenditure increases by
5% only there is a decrease in consumption.

________________________________________________________
506
Subhash Institute of Competitive Exams (SICE) Vijayawada
Vijaya Ph: 9030924949
________________________________________________________
Mathematics for Competitive Exams ( 39 )
Section III – Chapter 4: Percentage and its Applications

PRACTICE QUESTIONS
Answer the following questions by selecting the most appropriate option.

1. The number of chicks in a farm is 8,000. It 9. Raj and Raju appeared for RRB NTPC examination
increases each year at the rate of 10% p.a. in Bihar. Out of the total number of applicants
Calculate the population after 2 years. applying for the exam, 6% got selected. Delhi also
(1) 10,750 (2) 11,560 had the same number of applicants but 7% got
(3) 9,560 (4) 9,680 selected, which is nearly 80 more than the
applicants selected in Bihar. Calculate the
2. In a certain examination, 77% candidates passed
number of applicants who appeared for the RRB
in English and 34% failed in Mathematics. If 13%
NTPC examination.
failed in both the subjects and 784 candidates
(1) 5,400 (2) 7,890
passed in both the subjects, the total numbers of
(3) 8,000 (4) 4,500
candidates were
(1) 1800 (2) 1200 10. Asha and Ritu participated in a college election
(3) 1400 (4) 1600 where the candidate getting 79% of the total
votes is the winner by 2,726 votes. Calculate the
3. In a society election, 10% votes were declared
total number of votes casted in the election.
invalid. A candidate named Radha achieved 80%
(1) 3,400 (2) 5,400
of the total effective votes. If the total number of
(3) 4,700 (4) 3,400
votes is 60,000, calculate the number of valid
votes polled in favour of the candidate. 11. Shalu observed that in an election between two
(1) 13,600 (2) 20,000 candidates out of the total votes that were cast,
(3) 35,640 (4) 43,200 36% were invalid. Her brother got 2,208 votes out
of the total valid votes. which was 69% of the
4. Anuradha went to a grocer and bought 600
total valid votes. Calculate the total number of
apples and 400 guavas. She found that 15% of
votes.
apples and 8% of guavas were rotten. Calculate
(1) 4,200 (2) 2,400
the percent of fruits that are in good condition.
(3) 5,000 (4) 6,500
(1) 71% (2) 45.9%
(3) 87.8% (4) 67.5% 12. After selling 70% oranges, a fruit seller had 360
oranges left. Originally, how many oranges did he
5. Arjun spent 30% of the amount he took for buying
have?
groceries and at end he was left with `2100.
(1) 1080 (2) 1400
Calculate the amount he took along with him.
(3) 1750 (4) 1200
(1) 2,500 (2) 3,000
(3) 4,500 (4) 3,560 13. The difference between 31% of a number and 13%
of the same number is 576. What is 17% of that
6. Sheetal and Meetali appeared for SSC
number?
examination. One of them secured 80 marks more
(1) 561 (2) 457
than the other and also her marks was 75% of the
(3) 544 (4) 574
total marks obtained by them. Calculate the
marks obtained by them: 14. Nandita appeared for the test of five subjects –
(1) 40,120 (2) 20, 100 Hindi, Science, Maths, English and Sanskrit. She
(3) 30,110 (4) 50,130 scored 80% marks in the test. The maximum
marks of each subject are 105. How many marks
7. Two students Ria and Yana were paid a total
did she score in Science if she scored 89 marks in
amount of ` 840 per week by their parents. If Ria
Hindi, 92 in Sanskrit, 98 in Maths and 81 in
was paid 140% of the sum paid to Yana, then
English?
calculate the amount paid to Yana per week.
(1) 60 (2) 75
(1) 450 (2) 550
(3) 65 (4) 70
(3) 675 (4) 350
15. The price of sugar increases by 20%. By how
8. The ratio of weights of Virat and Rohit is 20:19.
much percent should a person decrease his
By what percent is the weight of Rohit less than
consumption of sugar so that there is no change
Virat?
in expenditure?
(1) 20% (2) 5%
(1) 20% (2) 16.67%
(3) 2.5% (4) 10%
(3) 25% (4) 22.22%

________________________________________________________
507
Subhash Institute of Competitive Exams (SICE) Vijayawada
Vijaya Ph: 9030924949
________________________________________________________
Mathematics for Competitive Exams ( 40 )

16. 45% of a number is 60% of another number. What 27. A is 30% more than B and B is 60% less than C. By
is the ratio of two numbers? what percentage is A less than C?
(1) 3 : 2 (2) 3 : 1 (1) 35% (2) 50%
(3) 4 : 3 (4) 2 : 5 (3) 30% (4) 48%
17. 96% of a number is 19,584. What is 56% of that 28. A man had a certain amount with him. He spent
number? 20% of that to buy an article and 5% of the
(1) 11,016 (2) 11,222 remaining on transport. Then he gifted ` 120. If
(3) 11,628 (4) 11,424 he is left with ` 1,400, the amount he spent on
transport is
18. The sum of 25% of a number and 15% of the same
(1) ` 95 (2) ` 80
number is 144. What is 44% of that number?
(3) ` 76 (4) ` 61
(1) 180 (2) 174.2
(3) 162.5 (4) 158.4 29. The population of a town is 3,11,250. The ratio
between women and men is 43: 40. If there are
19. What is the value of 72% of two-fifth of 450?
24% literate among men and 8% literate among
(1) 648.4 (2) 129.6
women, the total number of literate persons in
(3) 324.2 (4) 162.6
the town is
20. Smitha invests ` 1,87,000 in the stock market, (1) 56,800 (2) 99,600
which is 25% of her annual income. What is (3) 41,800 (4) 48,900
Smitha’s approximate monthly income?
30. In an examination, 52% of the candidates failed in
(1) ` 63,343 (2) ` 61,206
English and 43% failed in Mathematics. If 17%
(3) ` 60,196 (4) ` 62,333
failed in both the subjects, then the percentage of
21. The salary of a man increases by 20% every year candidates, who passed in both the subjects was
in the month of January. His salary was ` 5,000 in (1) 25 (2) 22
the month of February in 2009. What will be his (3) 23 (4) 21
salary in the month of February in 2011?
31. In an election there were only two candidates.
(1) ` 7,200 (2) ` 200
One of the candidates secured 40% of votes and
(3) `7,800 (4) ` 6,800
is defeated by other candidate by 298 votes. The
22. Find out the greatest possible number for which total number of votes polled is
30% of that number is less than 100. (1) 1490 (2) 1500
(1) 331 (2) 335 (3) 745 (4) 1460
(3) 325 (4) 333
32. If each side of a cube is increased by 10%, the
23. The ratio of weights of Sachin and Shoaib is 20 : volume of the cube will increase by
23. By what percent is the weight of Shoaib more (1) 30% (2) 10%
than Sachin? (3) 33.1% (4) 25%
(1) 30% (2) 15%
33. The number that is to be added to 10% of 320 to
(3) 25% (4) 3%
have the sum as 30% of 230 is
24. 46% of a number is 115. What is 150% of that (1) 37 (2) 32
number? (3) 23 (4) 73
(1) 370 (2) 385
34. The strength of a school increases and decreases
(3) 365 (4) 375
in every alternate year by 10%. It started with an
25. If twenty-five percent of three-seventh of twenty- increase in 2000. Then the strength of the school
six percent of a number is 136.5, what is the in 2003 as compared to that in 2000 was
number? (1) increased by 8.9% (2) decreased by 8.9%
(1) 6,300 (2) 5,600 (3) increased by 9.8% (4) decreased by 9.8%
(3) 4,800 (4) 4,900
35. In a community group of a society, 60% of the
26. A fruit seller had some oranges. After selling 40% residents were Christian and 50% were Adivasi. If
oranges, he had 420 oranges left. Originally, how every resident is either Christian or Adivasi or
many oranges did he have? both, calculate the %age of residents belonging to
(1) 610 (2) 700 both communities.
(3) 750 (4) 745 (1) 67% (2) 10%
(3) 13% (4) 10%

________________________________________________________
508
Subhash Institute of Competitive Exams (SICE) Vijayawada
Vijaya Ph: 9030924949
________________________________________________________
Mathematics for Competitive Exams ( 41 )
Section III – Chapter 4: Percentage and its Applications

ANSWERS AND EXPLANATIONS


1. (4) Population of chicks after 2 Hence, total number of spoiled Total number of applicants
110 110 fruits = 90 + 32 = 122 appeared in each state = 80 ×
years = 8,000 × × 100 = 8000
100 100 Therefore, the number of fruits
= 9,680 that remained in a better 10. (3) Let the total number of votes
2. (3) Let the total number of condition = 1000 122 = 878 =X
candidates =100 Hence, the percent of fruits that Now, 79% of X 21% of X =
77% candidates passed in remained in a better condition = 2726
English and 23% failed in it. (878/1000) × 100% = 87.8%
58% of X = 2726
34% candidates failed in 5. (2) Let the amount taken by
Arjun be x. X = 4,700
Mathematics.
Amount left with him = x – (3x 11. (3) Let the total number of votes
13% failed in English and
/10) = 7x/10 casted be X
Mathematics both
Therefore, 7x/10 = ` 2100 = x Number of valid votes = 64% of
Out of 23% failed in English,
= ` 3,000 X = 0.69 × 0.64 × X = 2208
13% failed in Mathematics also
Arjun took ` 3000 with him. X = 5,000
23 13 =10% failed in
English alone 6. (1) Let their individual marks be 12. (4) Let the original number of
(X + 80) and X. oranges be x
Similarly, 34 13 = 21% failed
in Mathematics alone Oranges left = 30% of x = 360
75
Then, X + 80 = (X 80 X)
Total number of failures = 10 + 100 100
21 + 13 = 44% x= ×360 = 1200
4X + 320 = 6X + 240) 30
Number of candidate passed Original number of oranges =
X = 40
in both the subjects =100 44 1200
= 56% Hence their marks were 40 and
120, respectively. 13. (3) Let the number be x.
Total number of candidates =
100 7. (4) Consider the amount paid to 31% of x 13% of x = 576
784 1400
56 Yana = x. 18% of x = 576
3. (4) Out of the total votes Amount paid to Ria = 140% of x
576
collected the number of invalid = 1.4x 1% of x = = 32
18
votes = 10 % of 60,000 =
Now, x + 1.4x = 840 2.4x =
10/100 × 60,000 = 6,000 17% of x = 17 × 32 = 544
840 x = 350
Total number of valid votes = 14. (1) Maximum aggregate marks
8. (2) Let 20x and 19x be the
60,000 – 6,000 = 54,000 of 5 subjects = 105 × 5 = 525
weights of Virat and Rohit,
Therefore, the number of valid respectively. Aggregate marks scored by
votes polled in favor of Radha = Nandita in 5 subjects = 80% of
Difference between their
80 % of 54,000 = (80/100) × 525 = 420
weights = 20x – 19x = x
54,000 = 43,200
Marks scored by Nandita in
Percentage by which the weight
4. (3) Total number of fruits Science = 420 – (89 + 92 + 98
of Rohit is less than that of Virat
bought by Anuradha = 600 + + 81) = 420 360 = 60
400 = 1000 x
= 100 = 5% 1
20x 15. (2) If increase in price is of
Number of spoiled apples = x
15% of 600 = (15/100) × 9. (3) Let the number of applicants original price, then decrease in
600 = 90 in Bihar and Delhi each = X quantity purchased so that
Number of spoiled guavas= 8% 7% of X 6% of X = 80 expenditure remains unchanged
of 400 = (8/100) × 400 = 32

________________________________________________________
509
Subhash Institute of Competitive Exams (SICE) Vijayawada
Vijaya Ph: 9030924949
( 42 )
________________________________________________________
Mathematics for Competitive Exams

1 Salary in January 2011 Oranges left = 60% of x = 420


is equal to of original = 1.2 × 6000
x 1
100
quantity. = ` 7,200 x 400 700
60
Increase in price = 20% of Salary in February 2011
= ` 7,200 27. (4) Let C = 100
1
original price = of original
5 22. (4) In the given options, the B = 100 – 60% of 100
price greatest number is 335. = 100 – 60 = 40
In order to maintain the same 30% of 335 = 100.5 A = 40 + 30% of 40
expenditure, decrease in = 40 + 12 = 52
1 Since 100.5 > 100
quantity = of original quantity Percentage by which A is less
6 335 is not the required 100 52
number. than C = 100 = 48%
= 16.67% 100

16. (3) Let x and y be the first and The second greatest number in 28. (2) Let us assume that the
second numbers, respectively. the given options is 333. amount with the man is =x
45% of x = 60% of y 30% of 333 = 99.9 Money spent to buy an article
Since 99.9 < 100 = 20% of x
x 4
3x = 4y =
y 3 Required number = 333 The remaining amount is
80% of x
17. (4) Let the number be x. 23. (2) Let 20x and 23x be Sachin
and Shoaib’s weight, Money spent on transport
96% of x = 19,584 = 5% of 80% of x
respectively.
19584 Difference between their 80 5 4
56% of x = 56 = x x = 4% of x
96 respective weight = 23x – 20x 100 100 100
11,424 = 3x
Money left with the man
18. (4) Let the number be x. Then Percentage by which Shoaib’s = 76% of x
25% of x + 15% of x = 144 weight is more than Sachin’s
Money gifted = ` 120
3x
40% of x = 144 4% of x more than Sachin’s= 100
20x 76
= 14.4 Amount left x 120
= 15% 100
44% of x = 144 + 14.4 = 1400
24. (4) Let the number be X.
158.4 x = 1520 ×100/76 = ` 2000.
46% of X = 115
2 Money spent on transport
19. (2) 72% of of 450 115
5 X= 100 = 250 4 4 2000
46 x ` 80
72 2
100 100
= 450 = 129.6 150% of X = 1.5 × 250 = 375
100 5 29. (4) Total number of literate
25. (4) Let x be the required people is [24/100 × 40/83 ×
1
20. (4) 25% or of Smitha’s annual number. Then, 311250] + [8/100 × 43/83 ×
4 311250] = 36000 + 12900
income = `1,87,000 3
= 48900.
25% of of 26% of x = 136.5
7
Smitha’s annual income = 30. (2) Let the total be 100%
187000 × 4 = ` 7,48,000 25 3 26
of x 136.5 The percentage of students who
Smitha’s monthly income 100 7 100
failed only in English = 52% -
748000 136.5 100 100 7 17% =35%
= x=
12 25 3 26
= 62333.33 ` 62,333
The percentage of students who
= 4,900 failed only in Mathematics =
21. (1) Salary in January 2010 43% – 17% = 26%
= 1.2 × 5000 = ` 6,000 26. (2) Let the original number of
oranges be x.

________________________________________________________
510
Subhash Institute of Competitive Exams (SICE) Vijayawada
Vijaya Ph: 9030924949
________________________________________________________
Mathematics for Competitive Exams ( 43 )
Section III – Chapter 4: Percentage and its Applications

So, the percentage of people Thus, increased volume Strength of school after 3 years
who passed in both the subjects 11x
3 = x (1 + 10%)(1 – 10%)
= 100% - (35+26+17)% = 22%. = – x3 = 0.331x3 (1 + 10%)
10
31. (1) Let us assume that the total = x(1.1)(0.9)(1.1) = 1.089x
number of votes polled was ‘x’ 0.33x 3
Increased % = × 100 Increase = 1.089x – x = 0.089x
x3
Therefore, = 33.1% 0.089x
% increase = × 100%
60 x 40 x 33. (1) Let the required number be x
298
100 100 x. = 8.9%
20 x
298 Now, 10% of 320 + x = 30% of
100 35. (2) Let the total number of
x 298 5 1490 230 residents in society = x
32. (3) Let x be the side of the cube 10 30 Number of residents belonging
320 x 230
and volume be x3. 100 100 to both communities = (60% +
32 + x = 69 50%) of x 100% of x = 10% of
10x 11x
Increased side = x x
100 10 x = 69 – 32 = 37
Hence, the %age of residents
3
11x 34. (1) Number of years after 2000 belonging to both communities
New volume =
10 to 2003 = 3 = 10%
Let initially strength be x

________________________________________________________
511
Subhash Institute of Competitive Exams (SICE) Vijayawada
Vijaya Ph: 9030924949
________________________________________________________
Mathematics for Competitive Exams ( 44 )

5 PROFIT, LOSS AND DISCOUNT

PROFIT, LOSS AND DISCOUNT


When a person runs a business, he/she either faces loss or gets profit.

COST PRICE (C.P.)


Cost price is the price at which a person purchases a product.

Selling Price (S.P.)


Selling price is the price at which a person sells a product.

Marked Price (M.P.)


It is the price that is marked on an article or commodity. It is also known as list price or tag
price. If there is no discount on the marked price, then selling price is equal to marked price.

MARKUP
It is the amount by which C.P. is increased to reach at M.P.
Markup = M.P. C.P.

Discount
The reduction offered by a merchant on marked price is called discount.
Discount = M.P. S.P.

Profit
When a person sells a product at a higher rate than the cost price, the difference of both
amounts is called profit.
Profit = S.P. C.P.

________________________________________________________
Subhash Institute of Competitive Exams (SICE) Vijayawada
Vijaya Ph: 9030924949
________________________________________________________
Mathematics for Competitive Exams ( 45 )
Section III – Chapter 5: Profit, Loss and Discount

Loss If S.P. = (100 x)% of C.P., then there is a loss of x%


When a person sells a product at a lower rate than the and vice versa.
cost price, the difference of both amounts is called loss.
For example, if S.P. is equal to 92% of C.P., then
Loss = C.P. S.P. loss percentage (100 – 92)% = 8%
SMART TIP
If two articles are sold at same selling price and
While solving profit and loss on one article, there is a profit of x% and on
problems, S.P. and C.P. for the same other article, there is a loss of x%, then there is a
amount of quantities are considered. x2
loss of %
For example, if you consider C.P. of 1 100
kg of sugar, then consider S.P. of 1 kg If cost prices of two articles are equal and one is
of sugar to find the profit or loss. If sold at a% profit and the other is sold at b% loss,
C.P. is given for x kg of sugar and S.P. then there is
is given for y kg of sugar, then we can
a b
take LCM of x and y to get C.P. and Overall profit of % if a > b
2
S.P. for the same quantities.
b a
Overall loss of % if b > a
2
Important Formulae
Neither profit nor loss if a = b
Profit
Profit % = × 100 If gain of x% is required by selling milk at C.P.,
C.P.
then quantity of water mixed with milk = x% of
Loss
Loss % = × 100 milk
C.P.
Let us now consider a few examples based upon what
Markup we have learnt so far.
Markup % 100
C.P.
1. Niraj incurred a loss of 55% on selling an article
Discount for ` 9,549. What was the cost price of the
Discount % = 100 article?
M.P.
100 x Solution:
If profit percentage is x, then S.P. = C.P. ×
100 Loss percentage = 55%
100 x Therefore, S.P. = (100 – 55)% of C.P.
If loss percentage is x, then S.P. = C.P. ×
100 = 45% of C.P.
Note: Profit percentage and loss percentage are always
45% of C.P. = 9,549
calculated on C.P. unless stated otherwise.
9549
C.P. = 100
Points to Remember 45
Some points to remember while resolving profit, loss and = ` 21,220
discount related problems are listed as follows: 2. In a sale, perfumes are available at a discount of
If there is a profit of x%, then amount of profit = x% 25% on the marked price. If the cost of the
perfume in sale is ` 5,895, then what is its
of C.P. marked price?
If there is a loss of x%, then amount of loss = x% of Solution:
C.P.
Since discount is 25%, S.P. = 75% of M.P.
If S.P. = (100 + x)% of C.P., then there is a profit of
75% of M.P. = ` 5895
x% and vice versa.
5895
For example, if S.P. is equal to 112% of C.P., M.P. = 100 = ` 7,860
75
then profit percentage is 12%.

________________________________________________________
513
Subhash Institute of Competitive Exams (SICE) Vijayawada
Vijaya Ph: 9030924949
( 46 )
________________________________________________________
Mathematics for Competitive Exams

3. Dipankar sold an item for ` 6,150 and incurred a Profit = 12% of 3000 = ` 360
loss of 25%. At what price should he have sold the
item to gain a profit of 25%? S.P. = 3000 + 360 = ` 3,360

Solution: Required overall profit = 18% of 6000 = ` 1,080

Since the item is sold at a loss of 25%, Required profit from the remaining half of goods =
1080 – 360 = ` 720
S.P. = 75% of C.P.
Profit percentage for remaining goods
In order to gain a profit of 25%, S.P. must be equal
to 125% of C.P. 720
= 100 = 24%
3000
Now
6. A man sold his mobile phone at a loss of 12%. Had
75% of C.P. = ` 6,150 he sold it for ` 3,492 more, he would have made a
6150
profit of 24%. Find the cost price of mobile phone.
25% of C.P. = = ` 2050
3 Solution:
125% of C.P. = 2050 × 5 = ` 10,250 Original S.P. = 88% of C.P.
4. A dishonest shopkeeper claims to sell his product New S.P. = 124% of C.P.
at a loss of 4% but he gives only 840 g for 1 kg.
What is his profit or loss percentage? 124% of C.P. 88% of C.P. = ` 3,492

Solution: 36% of C.P. = ` 3,492

Let C.P. for 1 kg = ` 1,000 3492


C.P. = 100 = ` 9,700
36
S.P. of 1 kg = 1000 – 4% of 1000 = ` 960
7. A fruit seller buys 2 lemons at `1 and sells 5
Since shopkeeper is giving only 840 g for 1 kg, his lemons at ` 3. His gain percentage is
actual C.P. is not ` 1000
Solution:
Actual C.P. = C.P. of 840 g = ` 840
In profit and loss questions, we must consider the
Profit = 960 – 840 = ` 120 C.P. and S.P. of the given quantity.
120 Consider the number of lemons purchased and sold
Profit percentage = 100 = 14.28%
840 = L.C.M of 2 and 5 = 10
5. A merchant bought goods worth ` 6,000 and sold Now, C.P. of 2 lemons = ` 1
half of them at 12% profit. At what profit
C.P. of 10 lemons = ` 5
percentage should he sell the remaining to make
an overall profit of 18%? S.P. of 5 lemons = ` 3
Solution: S.P. of 10 lemons = ` 6
For the first half of goods Profit gained by selling 10 lemons = 6 – 5 = ` 1
Since the goods were bought at ` 6000, then half of 1
the goods will cost ` 3000. Profit percentage = 100 = 20%
5
C.P. = ` 3,000

________________________________________________________
514
Subhash Institute of Competitive Exams (SICE) Vijayawada
Vijaya Ph: 9030924949
________________________________________________________
Mathematics for Competitive Exams ( 47 )
Section III – Chapter 5: Profit, Loss and Discount

PRACTICE QUESTIONS
Answer the following questions by selecting the most appropriate option.

1. By selling an article at 80% of its marked price, a 10. If a trader sells his stock of oranges at ` 18,270,
trader makes a loss of 10%.What will be the profit then he gains 45%. What is the cost price of stock
percentage if he sells it at 95% of its marked of oranges?
price? (1) ` 12,600 (2) ` 13,000
(1) 5.9 (2) 12.5 (3) ` 12,650 (4) ` 13,650
(3) 6.9 (4) 5
11. The C.P. of 13 articles is equal to the S.P. of 10
2. A man purchased a car for ` 1,35,000 and spent articles. Find the gain or loss percentage.
` 25,000 on repairs. At what price was the car (1) Loss 30% (2) Gain 30%
sold if he suffered 10% loss on it? (3) Loss 33.33% (4) Gain 33.33%
(1) ` 1,44,000 (2) ` 1,21,500
12. Raj sold an item for ` 6,384 and incurred a loss of
(3) ` 1,50,000 (4) ` 1,76,000
30%. At what price should he have sold the item
3. The marked price of a table fan is `1100 which is to gain a profit of 30%?
10% above the cost price. It is sold at a discount (1) ` 14,656
of 5% on the market price. Find the profit percent. (2) ` 11,856
(1) 4.5% (2) 2.45% (3) ` 13,544
(3) 5.75% (4) 10% (4) Cannot be determined
4. A shopkeeper sells 10 notebooks for the same 13. By selling an umbrella for ` 30, a shopkeeper
money as he paid for 20. What is his gain%? gains 20%. During a clearance sale, the
(1) 25% (2) 40% shopkeeper allows a discount of 10% of the
(3) 45% (4) 100% marked price. His gain % age during the sale
season is
5. Pooja wants to sell a watch at a profit of 20%. She
(1) 8 (2) 9
bought it at 10% less and sold it at ` 30 less, but
(3) 7 (4) 7.5
still she gained 20%. The cost price of the watch
is 14. What is the maximum percentage discount
(1) ` 220 (2) ` 225 (approximately) that a merchant can offer on his
(3) ` 240 (4) ` 250 marked price so that he ends up selling at no
profit or loss, if he initially marked his goods up by
6. A shopkeeper sold his goods at half the list price
40%?
and, thus, lost 20%. If he had sold on the listed
(1) 33.5% (2) 28.5%
price, his gain percentage would have been
(3) 60% (4) No discount
(1) 20% (2) 35%
(3) 60% (4) 72% 1
15. A sells an article to B making a profit of of his
7. A profit of 12% is made when a mobile phone is 5
sold at ` P and there is 4% loss when the phone is outlay. B sells it to C, gaining 20%. If C sells it for
sold at ` Q. Then Q : P is 1
` 600 and incurs a loss of of his outlay, the
(1) 4 : 5 (2) 3 : 1 6
(3) 1 : 1 (4) 6 : 7 cost price of A is
(1) ` 720 (2) ` 800
8. A sells a cycle to B at a profit of 20% and B sells it (3) ` 600 (4) ` 500
to C at a loss of 25%. If C bought the cycle for ` P,
then its cost price for A was 16. The cost price of an article is 60% of its marked
9 10 price for sale. How much percent does the
(1) ` P (2) ` P tradesman gain after allowing a discount of 16%?
10 9
1 9 (1) 25% (2) 40%
(3) ` P (4) ` P (3) 30% (4) 50%
20 20
17. The cost price of a washing machine is 75% of its
9. A man buys a toy for ` 25 and sells it for `30. His
selling price. Find the profit percentage.
gain percent is
(1) 30% (2) 33.33%
(1) 5% (2) 2.5%
(3) 25% (4) 66.66%
(3) 20% (4) 10%

________________________________________________________
515
Subhash Institute of Competitive Exams (SICE) Vijayawada
Vijaya Ph: 9030924949
________________________________________________________
Mathematics for Competitive Exams ( 48 )

18. Anita purchased a bicycle at a cost of ` 3,200. 27. By selling an article for ` 102, there is a loss of
She sold it at a loss of ` 240. At what price did 15%. When the article is sold for ` 134.40, the net
she sell the bicycle? result in the transaction is
(1) ` 2,860 (2) ` 2,690 (1) 12% gain (2) 12% loss
(3) ` 3,440 (4) ` 2,960 (3) 10% loss (4) 15% gain
19. The owner of a furniture shop charges his 28. If the cost price of 16 bags is equal to the selling
customer 15% more than the cost price. If the price of 20 of them, what will be the loss in this
customer paid ` 9,039 for a sofa set, then what is transaction?
its cost price? (1) 20% (2) 25%
(1) ` 7,680 (2) ` 7,860 (3) 5% (4) 10%
(3) ` 7,950 (4) ` 7,600
29. Rita will lose 16% by selling a mobile phone for
20. Vandana sells an article for ` 3,240 and earns a ` 1008. What percent shall she gain or lose by
profit of 20%. What is the cost price of the selling it for ` 1500?
article? (1) 25% (2) 20%
(1) ` 2,800 (2) ` 2,820 (3) 5% (4) 10%
(3) ` 2,750 (4) ` 2,700
30. Rahul sold a calculator at a gain of 8%. Had he
21. A fruit seller had some oranges. After selling 40% sold it for ` 90 more, he would have gained 12%.
oranges, he had 420 oranges left. Originally, how Find the cost price of the calculator.
many oranges did he have? (1) ` 1025 (2) ` 2250
(1) 610 (2) 700 (3) ` 1500 (4) ` 1845
(3) 750 (4) 745
31. A shopkeeper sells an article at a gain of 4%. Had
22. Find the single discount equivalent to a series he sold it at a loss of 10%, its selling price would
discount of 20%, 15% and 10%. have been ` 126 less. What is the cost price of
(1) 52.6% (2) 45% the article?
(3) 42.2% (4) 38.8% (1) ` 845 (2) ` 925
(3) ` 900 (4) ` 845
23. If the cost price of 15 tables is equal to the
selling price of 20 tables, then the loss 32. The marked price of an item is ` 6,000. If discount
percentage is increases from 10% to 30%, then profit decreases
(1) 25% (2) 16.66% from 35% to 5%. Find the cost price of the item.
(3) 22.22% (4) 33.33% (1) ` 4,000 (2) ` 4,500
(3 ) ` 3,600 (4) ` 4,26
24. An item costing ` 200 is being sold at 10% loss. If
the price is further reduced by 5%, the selling 33. A mobile phone retailer sells a mobile for ` 912.
price will be The sale is made at a profit of 20%. If the mobile
(1) ` 170 (2) ` 171 is sold at ` 855, find the retailer’s actual profit or
(3) ` 175 (4) ` 179 loss.
(1) 10% (2) 13%
25. A shopkeeper buys 144 items at 90 paise each.
(3) 12.5% (4) 14%
On the way, 20 items are broken. He sells the
remainder at ` 1.20 each. His gain per cent 34. A manufacturer sells an article to a wholesale
correct to one place of decimal is dealer at a profit of 10%. The wholesale dealer
(1) 13.8% (2) 14.6% sells it to a shopkeeper at 20% profit. The
(3) 14.8% (4) 15.8% shopkeeper sells it to a customer for ` 56,100 at
a loss of 15%. Then the cost price of the article to
26. There is a profit of 20% on the cost price of an
the manufacture is
article. The % of profit, when calculated on selling
(1) ` 50,000 (2) ` 55,000
price, is
(3) ` 25,000 (4) ` 10,000
2
(1) 16 % 35. A dealer sold two types of goods for `10,000
3
(2) 20% each. On one of them, he lost 20% and the other
1 he gained 20%. His gain or loss percent in the
(3) 33 % entire transaction was
3
(1) 4% gain (2) 4% loss
(4) None of these
(3) 2% loss (4) 20% gain

________________________________________________________
516
Subhash Institute of Competitive Exams (SICE) Vijayawada
Vijaya Ph: 9030924949
________________________________________________________
Mathematics for Competitive Exams ( 49 )
Section III – Chapter 5: Profit, Loss and Discount

36. The profit made by selling 30 metres of cloth is 38. The cost prices of two books are same. One book
equal to the cost price of 7.5 metres of cloth. Find is sold at a profit of 45% and the other is sold at a
the profit percentage made by selling 8 metres of loss of 17%. Find the overall profit percentage.
cloth. (1) 14% (2) 20%
(1) 22.5% (2) 22.22% (3) 17.72% (4) 12.67%
(3) 25% (4) 33.33%
39. A merchant makes a profit of 20% after giving a
37. The marked price of a tape recorder is `12,600. A discount of 50% to his customer. What will be his
festival discount of 5% is allowed on it. A second profit percentage, if the given discount is 40%?
discount of 2% is given on the cash payment. The (1) 36% (2) 25%
cash payment, in rupees, for buying the tape (3) 37.5% (4) 44%
recorder is
40. A person obtained the selling price of 4 articles by
(1) 11,370.60 (2) 11,073.60
selling 20 articles. What is his profit percentage?
(3) 11,703.60 (4) 11,730.60
(1) 20% (2) 16.66%
(3) 22.22% (4) 25%

________________________________________________________
517
Subhash Institute of Competitive Exams (SICE) Vijayawada
Vijaya Ph: 9030924949
( 50 )
________________________________________________________
Mathematics for Competitive Exams

ANSWERS AND EXPLANATIONS


1. (3) As per the question SP = 5. (4) Let the cost price of watch = 8. (2) Let the cost price of cycle for
80% of MP = 90% of CP (10% `x A=`
Loss)
Therefore, its selling price = Cost price of cycle for B = 120%
Therefore, MP = 1.125 CP 120 x 6x
` of ` x = `
SP = 95% of MP = 95% of 100 5
1.125CP = 1.06875 CP Now after applying the Selling price of cycle for B =
Profit = SP - CP = 1.06875 CP - conditions given in the question, 6x 9x
75% of ` =` =`P
CP =0.06875 CP or 6.875% or 9x 5 10
6.9% approximately. Cost price = `
10 10
=` P
2. (1) Total C.P. of a car = 9
6x
1,35,000 + 25,000 = ` Selling price = ` 30
5 9. (3) Cost price of toy = ` 25
`1,60,000
6x 9x 120 Selling price of toy = ` 30
According to the question, S.P.
5 10 100
100 loss% 30 25
= C.P. × Profit % = 100
100 x = ` 250 25
= 20%
90 6. (3) Let the list price of the goods
= 1,60,000 ` 1,44,000
100 be ` x 10. (1) Since there is a gain of 45%,
S.P. = 145% of C.P.
3. (1) M.P. of a table fan = ` 1100 Let the cost price of goods
= ` 100 145% of C.P. = ` 18,270
Let the cost price of fan be ` x
x 18270
x + 10% of x = 1100 Selling price = ` C.P. = 100
2 145
1100 100
x=` x = ` 12,600
110 100
Loss % = 2
11. (2) S.P. of 10 articles = C.P. of
` C.P. = x = ` 1000 100
13 articles
= 20%
S.P. = M.P. – Discount
13
x = 160 S.P. of 1 article = × C.P. of
S.P. = 1100 – 5% of 1100 = ` 10
1100 – 55 = ` 1045 Therefore, on selling the 1 article
product on list price, the
Profit = ` 1045 – 1000 = ` 45 S.P. = 1.30 C.P. = 130% of
shopkeeper will get a profit of
C.P.
45
Profit% = 100 % = 60%
1000 100 Gain percentage = 30%
= 4.5 % 12. (2) The article is sold at 30%
7. (4) Let the cost price of phone =
4. (4) Given, S.P. of 10 notebooks ` 100 loss. It means that it is sold at
= C.P. of 20 notebooks 70% of C.P.
If the phone is sold at 12%
Let C.P. of 1 notebook = ` 1 profit, then selling price = P = 70% of C.P. = ` 6,384
C.P. of 10 notebooks = ` 10 112 6384
100 = ` 112 C.P. = 100
100 70
S.P. of 10 notebooks = ` 20
If the phone is sold at 4% loss, = ` 9,120
Profit = S.P. – C.P. then the selling price = Q =
= ` 20 – `10 = ` 10 96 In order to gain a profit of 30%,
100 = ` 96 the article will be sold at 130%
100
10 of C.P.
Profit% = 100 % = 100%
Required ratio = 96 : 112
10 Therefore, the required S.P.
=6:7
= 130% of 9120 = ` 11,856

________________________________________________________
518
Subhash Institute of Competitive Exams (SICE) Vijayawada
Vijaya Ph: 9030924949
________________________________________________________
Mathematics for Competitive Exams ( 51 )
Section III – Chapter 5: Profit, Loss and Discount

13. (1) If the shopkeeper sells it for 19. (2) S.P. = 115% of C.P. As, 20 items are broken
` 30 and his profit is 20%, the
= ` 9,039 So, cost of 124 items = 12960
cost will be 30/1.20 = ` 25.
paise
If the shopkeeper allows 10% 9039
C.P. = 100 = ` 7,860 Selling price = 124 × 120
discount, the selling price 115
= 14880 paise
becomes 30 × 0.9 = ` 27. 20. (4) Profit = 20%
So he gains ` 2 on ` 25. 14880 12960
Therefore, S.P. = 120% of C.P. Profit %=
Therefore, the profit percentage 12960
will be (2/25 ×100) % = 8% 120% of C.P. = ` 3,240 1920
100 14.8%
12960
14. (2) Discount percent = 3240
C.P. = 100 26. (1) Let the cost price of article
120
MP SP be ` 100.
100
MP = ` 2,700
140 100 So, selling price = ` 120
100 21. (2) Let the original number of
140
oranges be x. So, percentage of profit on
40 selling price
100 28.5%
140 Oranges left = 60% of x = 420 20 2
= 100 16 %
15. (4) Let us assume that A’s 100 120 3
x 400 700
outlay was ` X. Therefore, A sells 60 27. (1) Let the cost price be x.
to B for 1.2x and B sells to C at
1.44 x. Now, for C to incur a loss 22. (4) Let the marked price be So,
of 1/6 of its outlay, C must sell ` 100
15x
at 1.44x ×(5/6) = 1.20x. x– = 102
20% 15% 10%
100
Now, it is given that 1.20x 100 80 68 61.2
85x = 10200
= 600.
Total discount = 100 – 61.2
x = 600/1.2 = 500. x = 120
= ` 38.8
16. (2) Let M.P. of an article be New selling price = ` 134.40
Discount percentage = 38.8%
` 100 So,
23. (1) 20 × S.P. of 1 table = 15 ×
C.P. of an article = 60% of 100 C.P. of 1 table 14.40
= ` 60 Profit% = × 100 = 12%
120
15
S.P. of an article = 84% of 100 S.P. of 1 table = × C.P.
20 28. (1) C.P. of 16 bags = S.P. of 20
= ` 84 bags
of 1 table
Profit = 84 – 60 = ` 24 Let C.P. of 1 bag be ` 100
S.P. = 0.75 C.P. = 75% of
Gain percentage = C.P. C.P. of 16 bags = ` 1600
24
100 40% Loss percentage = 25% C.P. of 20 bags = ` 2000
60
24. (2) S.P. of 20 bags = ` 1600
75 3
17. (2) C.P. = of S.P. = of Selling price of the item at 10%
100 4 Loss = C.P. – S.P.
S.P. loss = ` 180
= ` 2000-1600
5
4 So, 5% of 180 = × 180 = ` 400
S.P. = of C.P. = 133.33% 100
3
of C.P. =9 Loss
Loss% = 100
Therefore, Selling price C.P
Profit percentage = 33.33%
= 180 – 9 = ` 171
400
18. (4) In case of loss, S.P. 100 %
25. (3) 2000
= C.P. – loss
Total cost price = 144 × 90 20%
S.P. = 3200 – 240 = ` 2,960
= 12960 paise

________________________________________________________
519
Subhash Institute of Competitive Exams (SICE) Vijayawada
Vijaya Ph: 9030924949
________________________________________________________
Mathematics for Competitive Exams ( 52 )

100 2 2 loss % gain%


29. (1) C.P. = ` S.P C.P. = of M.P. = × 6000
100 L% 3 3 100
= ` 4,000 20 20
4% loss
100 100
` 1008 33. (3) C.P. of mobile phone
100 16
100 36. (3) Let C.P. of 1 metre of cloth
100 =` 912 =`1
` 1008 120
84
` 1200 C.P. of 30 metres of cloth
= ` 760
= ` 30
Profit = S.P. – C.P. New S.P. = ` 855
= ` 1500 – 1200 Profit = ` 855 – 760 Profit = C.P. of 7.5 metres of
cloth = ` 7.5
= ` 300 = ` 95
7.5
Profit Profit percentage = 100
95 30
P% 100 Profit% = 100 %
C.P 760 = 25%
300
100 % = 12.5% 37. (4) Selling price after discount
1200
of 5% = 12600 – 5% of 12600
25% 34. (1)
= ` 11,970
30. (2) Let the cost price of Manufa Wholesale Shopkeeper Customer
cturer dealer Selling price after a further
calculator be` x
X x+(10/100)x 11 x 11 x 56,100 at discount of 2% = 11970 – 2%
According to the question, = 11x/10 10 10 a loss of of 11970
20 33 x 15%.
108 x 112 x 100 25 = ` 11,730.60
90
100 100 38. (1) Let the C.P. of each book
112 x 108 x According to the given question:
90 = ` 100
100 100 33x
56,100 Total C.P. of two books = ` 200
112x 108x 25
90 33x 15 Total S.P. of two books = 145%
100
4x 25 100 of 100 + 83% of 100 = 145 +
90 33x 14,02,500
100 83
90 100 25 = ` 228
x 99x
4 Profit = 228 – 200 = ` 28
x ` 2250 500
16,500x 28
31. (3) Let C.P. of an article be ` x 70,12,50,000 Profit percentage = 100
200
According to the question, 2,475x
= 14%
104 x 90 x 16,500x 2,475x
126 Alternate method: If cost prices
100 100 70,12,50,000
of two articles are equal and
104 x 90 x 14,025x 70,12,50,000
126 one is sold at a% profit and the
100 x ` 50,000 other is sold at b% loss, then
126 100
x there is an overall profit of
14 Cost price of an article paid by
a b
`900 the manufacturer is equal to % if a > b
2
` 50,000.
32. (1) If discount is 10%, then
35. (2) Loss percentage = 20% Profit percentage
S.P. = 90% of M.P.
Gain percentage = 20% 45 17
If profit is 35%, then S.P. = = % = 14%
Loss percentage in the entire 2
135% of C.P.
transaction
90% of M.P. = 135% of C.P.

________________________________________________________
520
Subhash Institute of Competitive Exams (SICE) Vijayawada
Vijaya Ph: 9030924949
________________________________________________________
Mathematics for Competitive Exams ( 53 )
Section III – Chapter 5: Profit, Loss and Discount

39. (4) If discount is 50%, then S.P. If discount is 40%, then S.P. Profit = S.P. of 4 articles = ` 4
= 50% of M.P. = 60% of M.P.
C.P. of 20 articles = 20 – 4
and S.P. = 120% of C.P. S.P. = 60% of 240% of C.P.
= ` 16
= 144% of C.P.
50% of M.P. = 120% of C.P.
4
Profit percentage = 44% Profit percentage = 100
M.P. = 2.4 C.P. = 240% of 16
C.P. 40. (4) Let S.P. of 1 article = ` 1
= 25%
S.P. of 20 articles = ` 20

________________________________________________________
521
Subhash Institute of Competitive Exams (SICE) Vijayawada
Vijaya Ph: 9030924949
________________________________________________________
Mathematics for Competitive Exams ( 53 )

6 SIMPLE AND COMPOUND INTEREST

Interest (I): Interest is the money paid by the borrower to the lender for the use of money lent.
Principal: The money borrowed or lent out for a certain period of time (T) is called the
principal (P) or the Sum.
Rate of Interest (R): It is the rate at which principal is lent or borrowed. It may be calculated
annually, semi-annually, or quarterly. If the rate of interest is R% per annum, it means that R
is the interest on the principal for 1 year.
Time (T): Time period up to which the money is lent or borrowed.
Simple interest (SI): Simple interest is when you only pay interest on the amount you borrow.
PRT
SI = , where P = Principal, R = Rate and T = time.
100
For example, Ram invests 50,000 at a rate of 5%p.a. for a period of 10 years. What will be
the simple interest received after 10 years?
Here, P = 50,000, R = 5%, T = 10 years
PRT 50000 5 10
SI = = = 25,000
100 100
Amount (A): It is the sum of principal (P) and interest (SI) received after given period of time
(T) at given rate of interest (R)
A P SI

For example, Continuing with the above example, the amount will be,
A = P + SI = 50,000 + 25,000 = 75,000
Note: When the time is given in months or days, it must be converted to years.
If a sum of money becomes n times itself in T years at simple interest, then the rate of
interest is calculated by,
100 n 1
Rate= %
T
If a sum of money becomes n times in T years at SI then time required for P to become m
times of itself in x years is given by,
m–1
Required time= T years
n 1

________________________________________________________
Subhash Institute of Competitive Exams (SICE) Vijayawada
Vijaya Ph: 9030924949
________________________________________________________
Mathematics for Competitive Exams ( 55 )
Section III – Chapter 6: Simple and Compound Interest

Compound Interest (CI): It is the interest calculated on r1 r2 r3


A P 1 1 1
both the original principal and the previously 100 100 100
accumulated interest. This allows your investment to 2. When r is compounded half yearly, then we will take
grow even more than if you were paid only simple r
r , and T=2T
interest. So, Amount at the end of 1st year (or Period) will 2
become the principal for the 2nd year (or Period) and 3. When r compounded quarterly, then we will take
Amount at the end of 2nd year (or Period) becomes the r
r and T= 4T
Principal of 3rd year. 4

Amount = Principal + Interest 4. When time t is in fraction and r is compounded


annually then for the whole part of the time A is
where, A= Amount, P= Principal, r = Rate % compounded computed first and for the remaining fractional part
p.a. , T = no. of years and CI = compound interest. of the time SI is computed. After this the SI is added
to the A obtained.
The amount can be calculated using the formula,
Difference between Compound Interest and
t
r Simple Interest
A P 1
100
When T = 2, then
Compound Interest can be calculated using the formula, 2
r R SI
CI SI P Or CI SI
t 100 2 100
r
CI P 1 1
100 When T = 3
2
OR, Compound Interest = Amount - Principal R 300 R
CI SI P
100 100
For example, Geeta invested 20,000 at a rate of 5%p.a.
compounded annually for a period of 10 years. What will Some Important Examples:
be the amount and interest that she will receive after 2 1. A sum of money given at simple interest becomes
years? 3 times in 16 years. What is the rate of interest?

Here, P = 20000, R = 5%, T = 2 years Solution:

t
Let principal = P
r
A P 1 Amount after 16 years = 3P
100
S.I. = 3P – P = 2P
2
5
= 20000 × 1 = 22050 P × r × 16
100 2P =
100
Now, CI = A - P = 22,050 – 20,000 = 2050.
200
r= = 12.5%
16
2. Find the amount which Anuj will get on ` 8,192, If
he loaned it for 18 months at 12.5% per annum,
interest being compounded half yearly.
Solution:
2n
r2
A= P 1
Some important cases: 100
1. When Rates are different for different years, say r1%,
1
r2%, and r3% for 1st, 2nd and 3rd year respectively, There are 3 half years in 1 years.
2
then

________________________________________________________
523
Subhash Institute of Competitive Exams (SICE) Vijayawada
Vijaya Ph: 9030924949
________________________________________________________
Mathematics for Competitive Exams ( 56 )

12.5 7. The simple interest on a certain sum for 6 years is


Rate of interest = % = 6.25%
2 12
of the sum. The rate of interest is
3
25
6.25
A = 8192 1 = ` 9,826 Solution:
100

3. A motorcycle was bought for ` 1,25,000. Its value Let the sum = P
depreciated at the rate of 8% per annum. Find its
S.I. for 6 years = 12 P
value after two years. 25
Solution: 12
S.I. for 1 year = P
Value of motorcycle after 2 years 6 × 25
2
8 12
= 125000 1 = ×100% of P
100 6 × 25
= ` 1,05,800 = 8% of P
4. Population of a place increased to 97,020 in Rate = 8%
2015 at a rate of 5% per annum. What was its
population in 2013? 8. What will be the compound interest on 24,300 for
2 years if the rate of interest for the first year
Solution: being 9% and for the second year being 12%?
Let population in 2013 was P then Solution:
2

97020 = P 1
5 C.I. for the 1st year = 9% of 24300 = ` 2,187
100
C.I. for the 2nd year = 12% of 24300 + 12% of 2187
97020 = P 441 = 2916 + 262.44= ` 3,178.44
400
C.I. for 2 years = 2187 + 3178.44
400
P = 97020 × = 88,000 = ` 5,365.44
441
9. A Bank lent `5,12,000 to a company under
5. A sum of money is 5 times of itself in 15 years. In
compound interest and got `7,29,000 after three
how many years, it will be 125 times under
years. What is the rate of interest?
compound interest?
Solution:
Solution:
Here P = ` 5,12,000 and A = ` 7,29,000
Let the sum be ` x
Let r be the rate of interest per annum.
x becomes 5x in 15 years
Now
5x becomes 25x in next 15 years
3
r
25x becomes 125x in next 15 years 729000 = 512000 1
100
Thus, it will be 125 times in (15 + 15 + 15) = 45
3
years r 729
1 =
100 512
6. A loan was repaid in two annual instalments of `
1,331 each. If the rate of interest is 10% r 9
1 =
compounded annually, the sum borrowed was 100 8
Solution: r 1
=
Let the sum borrowed = P 100 8
1,331 + 1,331 × 1.1 = P × (1.1)2 100
r=
8
1331 2.1
P= = ` 2,310
1.12 = 12.5%

________________________________________________________
524
Subhash Institute of Competitive Exams (SICE) Vijayawada
Vijaya Ph: 9030924949
________________________________________________________
Mathematics for Competitive Exams ( 57 )
Section III – Chapter 6: Simple and Compound Interest

10. What annual payment will discharge a debt of 4


2
4
` 936.48 in 3 years at the rate of 4% per annum x 1 + x 1 x = 936.48
100 100
under compound interest?
Solution: 26 676
x 1 = 936.48
25 625
Here amount = ` 936.48
Let x be the annual installment then 625 93648
x = ` 300
1951 100

________________________________________________________
525
Subhash Institute of Competitive Exams (SICE) Vijayawada
Vijaya Ph: 9030924949
________________________________________________________
Mathematics for Competitive Exams ( 58 )

PRACTICE QUESTIONS
Answer the following questions by selecting the most appropriate option.

1. Find the difference between the compound 10. The rate of interest on a sum of money is 2% p.a.
interest and simple interest at 10% p.a. for 2 for the first 3 years, 5% p.a. for next 1 year and 6%
years on a principal of ` 1600? p.a. for last 2 years. If the simple interest on the
(1) ` 16 (2) ` 15 sum for a total period of 6 years. is `207 then the
(3) ` 20 (4) ` 18 sum is
(1) ` 500 (2) ` 600
2. The difference between compound interest and
(3) ` 700 (4) ` 900
simple interest on a certain sum of money for 2
years at 6% p.a. is ` 306. Find the sum. 11. Calculate the compound interest on ` 15000 in 3
(1) ` 60000 (2) ` 70000 years, when the rates of interest for successive
(3) ` 85000 (4) ` 320000 years are 5%, 10% and 15%.
(1) ` 2520 (2) ` 4923.75
3. What is the difference between the compound
(3) ` 4258.50 (4) ` 4653.80
interest and simple interest for the sum of
` 15,000 over a 3 year period if the compound 12. Rahul borrows ` 10,000 at 10% compound
interest is calculated at 10% p.a. and simple interest. If he repays ` 2450 at the end of first
interest at 8% p.a.? year, find the amount of loan which he still has to
(1) ` 5174 (2) ` 1365 pay?
(3) ` 1571 (4) ` 1671 (1) ` 2550 (2) ` 4923
(3) ` 8550 (4) ` 6650
4. In what time will ` 625 become ` 676 at 8% p.a.
interest compounded half-yearly? 13. If the simple interest on a certain sum of money is
(1) 2 years (2) 3 years four times the principal and the rate of interest
(3) 1 year (4) 6 years per annum equals the number of years, then the
rate of interest is
5. A sum of money becomes 5 times of itself in 16
(1) 14% (2) 18%
years. What is the rate of interest?
(3) 16% (4) 20%
(1) 25% (2) 18%
(3) 16% (4) 17% 14. If the simple interest on a certain sum of money
1 2
6. The principal amount which yields a compound after 3 years is th of the principal, then the
interest or ` 208 in the second year at 4% is 3 9
(1) ` 13000 (2) ` 6500 rate of interest per annum is
(3) ` 5000 (4) ` 10000 (1) 4.29% (2) 8.40%
(3) 6.66% (4) 7.33%
7. An amount is invested in a bank. The compound
rate of interest, after first and third year is ` 1200 15. The total number of public toilets in a town is
and ` 1587 respectively. What is the rate of approximately 3600. If the government has
interest? decided to increase the number of toilets in the
(1) 12% (2) 15% area by 10% every year, find the number of toilets
(3) 10% (4) 3.9% after 2 years?
(1) 5000 (2) 4356
8. The difference between compound and simple (3) 4536 (4) 5682
rates of interest on ` 10000 for 3 year at 5% p.a.
is 16. The difference between the interests received
(1) ` 76.50 (2) ` 76 from two different associates on ` 600 for 3 years
(3) ` 76.25 (4) ` 76.75 is ` 27. What is the difference between the rates.
(1) 4.5% (2) 2.5%
9. `15,494 is divide between A and B so that A’s (3) 1.5% (4) 0.5%
share at the end of 9 years is equal to B’s share at
the end of 11 years, compound interest being 17. Shruti lent ` 300 to Anjali for 1 year and ` 400 to
20% per annum. Then A’s share is Neeta for 3 years and received ` 105 as interest
(1) ` 9144 from both. Find the rate of interest, S.I. being
(2) ` 9414 calculated?
(3) ` 8000 (1) 4% (2) 8%
(4) ` 9140 (3) 6% (4) 7%

________________________________________________________
526
Subhash Institute of Competitive Exams (SICE) Vijayawada
Vijaya Ph: 9030924949
________________________________________________________
Mathematics for Competitive Exams ( 59 )
Section III – Chapter 6: Simple and Compound Interest

18. The simple interest for 4 years at 4% per annum is 27. Bank A lent ` 784 to a company B at compound
equivalent to ` 464 on a principal (in `) of interest and got ` 961 after 2 years. What is the
(1) 4640 (2) 46400 rate percent charged if the interest is compounded
(3) 1500 (4) 2900 annually?
(1) 5.80% (2) 8.35%
19. An amount doubles itself in 16 years at simple
(3) 10.71% (4) 6.05%
rate of interest. At what time will it treble itself?
(1) 32 years (2) 45 years 28. After how much time will a sum of ` 450 become
(3) 10 years (4) 5 years ` 531 at the rate of 3.6% p.a.?
(1) 2 years (2) 4 years
20. The compound interest on ` 5,000 for 3 years of
(3) 5 years (4) 6 years
10% p.a. will amount to
(1) ` 1,654 (2) ` 1,655 29. What is the difference between the simple
(3) ` 1,600 (4) ` 1,565 interest on a principal of ` 1000 being calculated
at 10% p.a. for 2 years and at 5% p.a. for 3 years?
21. The sum of money which becomes ` 2420 at 10%
(1) 20 (2) 50
rate of compound interest after two years will be
(3) 80 (4) 70
(1) ` 1000 (2) ` 1500
(3) ` 2000 (4) ` 2500 30. If the compound interest on a certain sum for
1 year is ` 31, what could be the simple interest?
22. A man borrows ` 5,000 and pays back after 4
(1) Remains same (2) Increases by 2
years at 15% simple interest. The amount paid by
(3) Decrease by 1 (4) Increases by 1
the man is
(1) ` 9,000 (2) ` 8,600 31. A sum of money is invested at compound interest
(3) ` 7,800 (4) ` 8,000 payable annually. The amount of interest in two
successive years are ` 2520 and ` 3000. Find the
23. A sum of ` 6,000 gives a simple interest of `
rate of interest.
5,040 in 7 years. The rate of interest per annum is
(1) 5% (2) 19%
(1) 11.11% (2) 6.75%
(3) 20% (4) 19.04%
(3) 12% (4) 15%
32. The compound interest calculated yearly at 10%
24. What is the compound interest accrued on a sum
on a certain sum of money amounts to ` 500 in
of ` 1,800 at the rate of 4% per annum in 2 years?
the third year. Calculate the compound interest
(1) ` 146.88
for the 4th year at the same rate and on the same
(2) ` 1,946.88
sum.
(3) ` 156.84
(1) ` 500 (2) ` 600
(4) ` 1,846.84
(3) ` 550 (4) ` 640
25. A sum of money becomes 3 times of itself in
33. A certain sum of money at compound interest
5 years. In how many years, will it be 81 times
amounts to ` 196 in 1 year and to ` 256 in
under compound interest?
3 years. Find the sum.
(1) 20 years
(1) ` 171.50 (2) ` 200
(2) 15 years
(3) ` 300 (4) ` 452
(3) 25 years
(4) 12 years 34. At what rate will a sum of ` 1,000 amount to
` 1,102.50 in 2 years at compound interest?
26. Mr Sharma invested an amount of ` 25,000 in
(1) 6.5% (2) 5%
fixed deposit at compound interest at the rate of
(3) 5.5% (4) 6%
8% per annum for 2 years. What amount will Mr
Sharma get on maturity? 35. A certain sum will amount to `12,100 in 2 years
(1) ` 28,540 at 10% per annum of compound interest, interest
(2) ` 29,160 being compounded annually. The sum is
(3) ` 29,240 (1) ` 12000 (2) ` 6000
(4) ` 28,240 (3) ` 8000 (4) ` 10000

________________________________________________________
527
Subhash Institute of Competitive Exams (SICE) Vijayawada
Vijaya Ph: 9030924949
________________________________________________________
Mathematics for Competitive Exams ( 60 )

ANSWERS AND EXPLANATIONS


1. (1) Let the principal be ` x 2500 306 therefore, the interest amount
x
9 at 4% per annum for 1 year
P R T
S.I. = x ` 85000 would be ` 200. Therefore, the
100
principal for the 2nd years would
t
R be 5000 + 200 = 5200. Now,
S.I. = 1600 10 2 3. (2) C.I. = P 1 1
100 100 the interest of 5200 at 4% per
annum would be 5200 × 4/100
= ` 320 3 = ` 208. Therefore, Option (3) is
10
In case of C.I, = ` 15000 1 1 correct.
100

R
t 7. (2) Let the amount be ` P and
A P 1 = ` 4965 rate of interest be R % annually.
100
15000 8 3 According to the question,
2
S.I. =
10 100
A 1600 1 Amount after 1st year = ` 1200
100 = ` 3600
R
2 Required difference = C.I. – S.I. P 1 1200 ..(i)
A 1600 1.1 100
= ` (4965 – 3600)
A = 1936 Amount after 3rd year = 1587
= ` 1365
Therefore, the compound t R
3

interest = 1936 – 1600 R P 1 1587 ...(ii)


4. (3) A P 1 100
= ` 336 100
On dividing Eq. (ii) from Eq. (i),
Difference between the C.I. and 8
2 t
we get
S.I. = 336 – 320 = ` 16 676 625 1
2 100
2
2. (3) Let the sum be ` P 676 208
2t R 1587 529
1
625 200 100 1200 400
t
R 2 2t
C.I. = P 1 1 26 26 R 23
100 1
25 25 100 20
2 2t 2
6 R 3
=` P 1 1 t 1 yr
100 100 20
5. (1) Let principal = ` x
309P R = 15 %
=` Amount (A) = 5x
2500 8. (2) The amount of simple
P 2 6 S.I. = A – P interest = 10000 × 5/100 × 3
S.I. = = ` 1500
100 S.I. = 5x – x
The amount of compound
=`
12P = ` 4x
interest = [10000 × (1.05) 3] =
100
x 16 R (10000 × 1.1576) – 10000
According to question, 4x = = 1576
100
C.I. – S.I. = ` 306 4 x 100
Therefore, the required
Rate = % difference = `1576 – ` 1500
309P 12P x 16
– = 306 = ` 76
2500 100 = 25%
9. (1) Let us assume that the
309 x 300 x 6. (3) These types of problems share of A and B are P and Q
306
2500 should be solved using the respectively.
9x options. In option (3), the
306 Therefore, P × (1.2)9 = Q ×
2500 principal amount is ` 5000;
(1.2)11

________________________________________________________
528
Subhash Institute of Competitive Exams (SICE) Vijayawada
Vijaya Ph: 9030924949
________________________________________________________
Mathematics for Competitive Exams ( 61 )
Section III – Chapter 6: Simple and Compound Interest

1.2
11
110 17. (4) Interest earned when Shruti
P A = ` 10000×
9
= (1.2)2 = 1.44 100 lent ` 300 to Anjali for 1 year
Q 1.2
300 1 r
A = ` 11,000 S.I. =
Now, solving through the 100
options, in option (1), the share Since Rahul has paid ` 2450 at
of A is `9144; therefore, the the end of 1st year = ` 3r
share of B is `6350. Interest earned when Shruti lent
Amount of loan which he still
` 400 to Neeta for 3 years
9144 has to pay = ` (11000 – 2450)
Now, = 1.44. Therefore,
6350 = ` 8550 400 3 r
S.I. =
an option (1) is correct. 100
13. (4) Let the principal be ` x
10. (4) Let the principal be ` x = ` 12r
S.I. = ` 4x
According to question, 3r + 12r = 105
&R=T
x 2 3 x 5 1 15r = 105
100 100 x R R
4x r = 7%
x 6 2 100
207 400 R 2
100 18. (4) Let the principal = P
R 400
6x 5 x 12 x Then,
207 R 20%
100 100 100
P 4 4
23 x 14. (3) Let the principal be ` x 464
207 100
100
207 100 2x
x S.I. = ` & T =10/3 years P = ` 2900
23 9
x ` 900 19. (1) Let Principal = P
According to the question,
t
When amount treble itself,
R 2x x R 10 interest will be equal to 2P.
11. (2) C.I. = P 1 1
100 9 3 100
Interest = P in 16 year
2 x 3 100
1
5
1
10 R Interest = 2P in 32 years
100 100 9 10 x
15000 3
15 20 10
1 1 R % 20. (2) Amount = 5000 1
100 100
3
105 110 115 R 6.66% = ` 6655
15000 1
100 100 100 Compound Interest = Amount –
15. (2) Number of public toilets
after Principal
1328250
15000 1
1000000 10
2
= ` 6655 – ` 5000
2 years = 3600 1
100 = ` 1,655
1328250 1000000
15000
1000000 = 3600
11 11 21. (3) Let the required sum of
10 10 money = ` P
328250
` 15000 = 4356 2
1000000 10
P 1 = 2420
16. (3) According to the information 100
` 4923.75
given in the question,
t
P = 2000
R 600 r1 3 600 r2 3
12. (3) A P 1 27 5000 15 4
100 100 100 22. (4) S.I. =
18r1 18r2 27 100
1
10 = ` 3,000
A=10,000 1+ 27
100 r1 r2
18 Amount = 5000 + 3000
r1 r2 1.5% = ` 8,000

________________________________________________________
529
Subhash Institute of Competitive Exams (SICE) Vijayawada
Vijaya Ph: 9030924949
________________________________________________________
Mathematics for Competitive Exams ( 62 )

P rt 3 R 3100 1
23. (3) S.I. = =
100 28 100 100
3 100
R S.I. = ` 31
S.I. 100 28
r=
P t R 10.71% Simple interest remains same
5040 100 28. (3) Given P =` 450 & Amount 31. (4) Interest for 1st year = ` 2520
r= = 12% (A) = ` 531
6000 7 Difference in interest of 2 years
S.I. = A – P = ` (3000 – 2520)
24. (1) C.I. for 1st year = 4% of 1800
= ` 72 = ` 531 – 450 = ` 480
C.I. for 2nd
year = 4% of 1800 S.I. = ` 81 480 100
+ 4% of 72 = 72 + 2.88 = ` Rate% =
S.I. 100 2520 1
74.88 T=
R P = 19.04%
C.I. for 2 years = 72 + 74.88
= ` 146.88 81 100 32. (3) C.I. for 4th year = C.I. of 3rd
=
450 3.6 year + interest on it for 1 year
25. (1) Let the sum be ` x
T = 5 years = ` 500 + 10% of ` 500
x becomes 3x in 5 years
29. (2) S.I. at 10%p.a for 2 years = ` 500 + 50
3x becomes 9x in next 5
years P R T = ` 550
S.I. =
100
9x becomes 27x in next 33. (1) Amount in 1st year = ` 196
5 years 1000 10 2
S.I. = r
1

27x becomes 81x in next 100 P 1 ` 196 ……(1)


100
5 years = ` 200
Amount in 3rd year = ` 256
Thus, it will be 81 times in S.I. at 5% p.a. for 3 years
(5 + 5 + 5 + 5) = 20 years r
3

P R T P 1 ` 256 ……(2)
26. (2) C.I. for 1st year = 8% of S.I. = 100
100
25000 = `2,000
Dividing (2) by (1), we get
1000 5 3
C.I. for 2nd year = 8% of (25000 S.I. =
+ 2000) = ` 2,160 100 r
2
256
1
C.I. for 2 years = 2000 + 2160 = ` 150 100 196
2 2
= ` 4,160 Required difference r 16
1
Amount after 2 years = 25000 = ` 200 – ` 150 100 14
+ 4160 = ` 29,160 = ` 50
r 16
27. (3) Principal = ` 784 & Amount 1
R
t
100 14
= ` 961 30. (1) C.I. = P 1 1
r 16
100
t
1
R 100 14
A P 1
100 R
1
r 2
31 = ` P 1 1
2
100 100 14
R 200 100
961 784 1 r %
100 PR 14 7
31 = ` ( P P)
2 100
961 R 1
1 PR = ` 3100 -(1) r
784 100 Given, P 1 196
2 2 100
31 R P R T
1 S.I. = 100
1
28 100 100 P 1 196
31 R 7 100
1
28 100

________________________________________________________
530
Subhash Institute of Competitive Exams (SICE) Vijayawada
Vijaya Ph: 9030924949
________________________________________________________
Mathematics for Competitive Exams ( 63 )
Section III – Chapter 6: Simple and Compound Interest

8 11025 r 11
2
P 196 1 12100 =
7 10000 100 10
196 7 r 105 5 1210000
P ` 1 P= = ` 10000
8 100 100 100 121
P ` 171.5
r = 5%
34. (2) 35. (4)
2
r 2
1102.5 1000 1 10
100 12100 =
100

________________________________________________________
531
Subhash Institute of Competitive Exams (SICE) Vijayawada
Vijaya Ph: 9030924949
( 64 )
________________________________________________________
Mathematics for Competitive Exams

7 AVERAGE, MIXTURES AND ALLIGATION

AVERAGE
Average is used as a measure of central tendency of a set of numbers. It is also known as
mean or arithmetic mean. There are three types of averages – mean, median and mode. Out
of these, arithmetic mean is the most important type of mean from the perspective of
insurance examinations.

Mean
The basic formula for average is as follows:

Sum of quantities
Average =
Number of quantities

S
A=
N
This formula is applicable for finding the average of numbers that fall within a particular group
only. However, when we are given two or more sets (say k sets) and each having certain
quantity of numbers, then the average of these sets is called the weighted average of k sets.
The formula for weighted average is:
n1 A1 + n2 A2 + .......+ nk Ak
Weighted Average =
n1 + n2 + ....... + nk

Where, Ax =Average of set x


nk = Number of elements in set k

________________________________________________________
Subhash Institute of Competitive Exams (SICE) Vijayawada
Vijaya Ph: 9030924949
________________________________________________________
Mathematics for Competitive Exams ( 65 )
Section III – Chapter 7: Average, Mixtures and Alligation

The properties and important formulae related to n+1


averages are as follows: 2

1. If the average of x1, x2, .............. xn is X; then 5. The average of the square of first n natural numbers
is given by:
i. the average of x1 + a, x2 + a, .............. xn + a is X
+a (n+1) (2n+1)
6
ii. the average of x1 – a, x2 – a, .............. xn – a is X
6. The average of the cubes of first n natural numbers
–a
is given by:
iii. the average of ax1, ax2, .............. axn is aX;
n (n + 1)2
however, it is essential that a 0
4
iv. the average of x1/a, x2/a, .............. xn/a is X/a;
7. You are given numbers from 1 till n. Then, the
however, it is essential that a 0 average of all the odd numbers in this series is given
Illustration: Given that the average of set {1, 2, 3, 4} by:
is 2.5, find the average of {5, 10, 15, 20}. Last odd number of series 1
Solution: Here, we know that if the average of x1, x2, 2
.............. xn is X, then average of ax1, ax2, .............. Illustration: Calculate the average of the set: {1, 3,
axn will be aX. Therefore, average of {5, 10, 15, 20} 5, 7, 9, 11, 13, 15}.
will be 5 × 2.5 = 12.5.
15 1
2. If it is given that the average of n numbers (x 1, x2, Solution: The average of the given set 8
2
..........xn) is X and if one number belonging to the set
8. You are given numbers from 1 till n. Then, the
having the value O (old) is replaced by another value
average of all the even numbers in this series is
N (new) and the new average becomes Y, then the
given by:
value of N is calculated as:
Last even number of series + 2
N = O + n(Y – X)
2
Illustration: You are given a set {1, 2, 3, 4} and 3 is
Illustration: Calculate the average of the set: {2, 4,
replaced by 6. Calculate the new average.
6, 8, 10, 12, 14, 16, 18, 20}.
Solution: The average of set {1, 2, 3, 4} is 2.5. Let us
20 2
replace 3 with 6. Then, the new average of set {1, 2, Solution: The average of the given set 11
2
6, 4} is calculated as:
9. If there is a series that comprises the first n
6 = 3 + 4 (Y – 2.5)
consecutive even numbers, then average of this
0.75 = Y – 2.5 series is given by:
Y = 3.25 n+1
3. It is given that the average of n numbers (x1, x2, Illustration: Calculate the average of the set: {2, 4,
.............. xn) is X. If one number is removed and new 6, 8, 10, 12}.
average becomes Y, then the number removed is:
Solution: The average of the given series = 6 + 1
n(X – Y) + Y = 7.
Illustration: From a set of numbers {1, 2, 3, 4, 5, 6}, 10. If the average of n consecutive numbers is m, then
one number was removed at random. The mean of the difference between the smallest and largest
new set after removing the number was 3.4. Which numbers is 2(m – 1).
number was removed from the set?
Solution: The number removed = 6 (3.5 – 3.4) + 3.4 ALLIGATION
= 0.6 + 3.4 = 4
4. The average of first n natural numbers is given by: Alligation is a rule that helps us solve the problems
related to mixtures. Alligation rule helps in finding out the

________________________________________________________
533
Subhash Institute of Competitive Exams (SICE) Vijayawada
Vijaya Ph: 9030924949
________________________________________________________
Mathematics for Competitive Exams ( 66 )

ratio in which two items or ingredients (having certain Mean price = m


cost) must be mixed to obtain a final mixture having the
Quantity of cheaper ingredient = qc
ingredients in a known ratio.
Quantity of dearer ingredient = qd
The price per unit of the final mixture is called mean
price or weighted price. Then,

Alligation problems involve application of the weighted qc y – m


=
average concept of average. You must be already aware qd m – x

that if we have 1, 2, 3, ……….., k sets, each containing Please note that this formula has been derived from the
n1, n2, n3, …….., nk elements; and the average of set k is alligation formula only.
represented as Ak, then the weighted average of all the
sets together is calculated as: Illustration: A fruit seller bought two types of oranges.
The cost of two types is ` 25/kg and ` 35/kg,
n1 A1 + n2 A2 + ....... + nk Ak respectively. The fruit seller purchased 10 kg of the first
Aw =
n1 + n2 + ....... + nk
type and 15 kg of the second type of oranges. What is
Illustration: Two types of oil having prices ` 40 per litre the average price of oranges if he mixed both types of
and ` 45 per litre are mixed together to produce a oranges?
mixture having price ` 43 per litre. If 7 litres of first type Solution:
of oil and x litres of second type of oil are mixed, then
find the value of x. =

Solution:
10 m – 250 = 525 – 15 m
n1 A1 n2 A2
Aw = 25 m = 775
n1 n2
m = ` 31/kg
43 =
7+x Graphical Solution for Alligation
43(7 + x) = 280 + 45x (Cross-Method)
We can also solve alligation problems by using graphical
x = 10.5 litre approach instead of doing it with the help of equations.
We represent the two-ingredient mixture problems as
Alligation Rule
follows:
Generally, many questions mention that two ingredients
with different prices (say x for cheaper ingredient and y
for dearer ingredient) are mixed together to form a
mixture. In such cases, the following formula is
considered:

rice
=
dient
Illustration: A fruit seller bought two types of oranges.
If The cost of two types is ` 25/kg and ` 35/kg
Cost of cheaper ingredient = x respectively. The fruit seller purchased 10 kg of the first
type and 15 kg of the second type of oranges. What is
Cost of dearer ingredient = y

________________________________________________________
534
Subhash Institute of Competitive Exams (SICE) Vijayawada
Vijaya Ph: 9030924949
________________________________________________________
Mathematics for Competitive Exams ( 67 )
Section III – Chapter 7: Average, Mixtures and Alligation

the average price of oranges if he mixed both types of name this process as ‘replace’. The ratio of the liquid
oranges? Calculate this by graphical method. A left in the container after ‘n’ replace operations to
the original quantity of liquid A at the beginning is
Solution:
calculated as:
Original quantity of liquid A = a litres

n
a–b
=
a

Liquid A left in container after ‘n’ replace operations


n
a–b
= a
a
= w

w Illustration: A container has 100 litres of honey. Out


of this container, 5 litres of honey is replaced with 5
2Aw – 50 = 105 – 3 Aw
litres of the honey substitute. This replacement
5 Aw = 155 process is done 3 times after which x litre of pure
honey remains. What is the value of x?
Aw = ` 31/kg
Solution: Here, a = 100 l; b = 5 l; and n = 3
Amount of honey left in the container after 3
MIXING WITH REPLACEMENT a–b
n

replace operations = a
a
Let us study the two important cases of mixture with
replacement. 100 5
3

= 100
100
A container is full of ‘a’ litres of a liquid (say liquid A).
Out of this, ‘b’ litres of liquid A are taken out and = 85.7375 l
replaced with another liquid (say liquid B). The
process of taking out ‘b’ litres and replacing with
same quantity of water is repeated ‘n’ times. Let us

________________________________________________________
535
Subhash Institute of Competitive Exams (SICE) Vijayawada
Vijaya Ph: 9030924949
( 68 )
________________________________________________________
Mathematics for Competitive Exams

PRACTICE QUESTIONS
Answer the following questions by selecting the most appropriate option.

1. A container contains a mixture of two pulses A 8. The average monthly income of 8 workers is
and B in the ratio of 8:3, when 8 kg of mixture are ` 3645. A worker whose income was ` 3000 left
drawn off, and the container is filled with pulse B, the job. Find out the average income of the
the ratio of pulses A and B becomes 18:7. How remaining 7 workers in the factory?
much of pulse A was initially in the container? (1) ` 3000 (2) ` 3458
(1) 12 kg (2) 14 kg (3) ` 3121.50 (4) ` 3737.14
(3) 18 kg (4) 16 kg
9. Average rainfall on Monday, Tuesday, Wednesday
2. P and Q are two alloys of bronze and platinum and Thursday is 420.5 cm and average on
prepared by mixing metals in the ratio of 5:6 and Tuesday, Wednesday, Thursday and Friday is
9:13, respectively. If equal quantities of alloys P 440.5 cm. If the ratio between rainfall for Monday
and Q are melted to form a third alloy R, find the and Friday is 20:21, find the rainfall in cm on
ratio of bronze and platinum in R. Monday and Friday.
(1) 7:9 (2) 11:16 (1) 1700, 1740 (2) 1682, 1762
(3) 13:36 (4) 19:25 (3) 1800, 1890 (4) 1600, 1680
3. Three bottles have same volume. The ratios of tea 10. The average of 15 numbers is 9.6. If the average
and milk in 1st, 2nd and 3rd bottles are 4:1, 7:3 of first 7 numbers is 4.5 and that of the last 7
and 13:2, respectively. If all the three bottles are numbers is 6.5, the 8th number is
mixed, find the ratio of tea and milk in the (1) 25 (2) 67
mixture. (3) 45 (4) 58
(1) 67:17 (2) 63:13
11. The average salary of 10 teachers in a school is
(3) 71:19 (4) 59:11
` 1500 per month. If the principal’s salary is
4. One alloy contains steel and aluminum in the added, the average becomes ` 2000 per month.
ratio of 2:1 and the other contains in the ratio of The principal’s salary is
5:9, respectively. What weight of the two must be (1) ` 6000 (2) ` 8000
melted together, so as to make a 130 lb mass (3) ` 7000 (4) ` 5000
with 60% steel?
12. The average of the test scores of a class of ‘m’
(1) 115 lb (2) 127 lb
students is 70 and that of ‘n’ students is 91.
(3) 198 lb (4) 102 lb
When the scores of both the classes are
5. The average weight of first 11 persons among 12 combined, the average is 80. What is n/m ?
persons is 95 kg. The weight of the 12th person is (1) 10/13 (2) 10/11
33 kg more than the average weight of all the 12 (3) 11/10 (4) 13/10
persons. The weight of the 12th person is
13. The average amount of 15 items was found to be
(1) 128 kg (2) 97.45 kg
` 80. On rechecking, it was found that the cost of
(3) 128.75 kg (4) 131 kg
2 items was wrongly taken as ` 30 and ` 15
6. A shopkeeper mixes two kinds of rice, one costing instead of ` 40 and ` 20, respectively. Find the
` 30.50 per kg and the other ` 20.25 per kg, such correct average amount.
that the price of the mixture is ` 25.75 per kg. (1) 61 (2) 71
What is the ratio of the first kind of rice to that of (3) 91 (4) 81
the second?
14. The average of first thirty natural numbers is
(1) 19:22 (2) 2:11
(1) 13.5 (2) 14.5
(3) 31:89 (4) 43:87
(3) 15.5 (4) 16.5
7. The ratio of quantities of sulphuric acid and water
15. The average of 6, 12, 18, … 120 is
in a 700 ml mixture is 2:5. How much water is
(1) 61
replaced with sulphuric acid to make the ratio
(2) 62
5:5?
(3) 63
(1) 230 ml (2) 185 ml
(4) 64
(3) 135 ml (4) 150 ml

________________________________________________________
536
Subhash Institute of Competitive Exams (SICE) Vijayawada
Vijaya Ph: 9030924949
________________________________________________________
Mathematics for Competitive Exams ( 69 )
Section III – Chapter 7: Average, Mixtures and Alligation

16. The average salary per head of all workers of an 25. The average of 50 numbers is 23. If each number
institution is ` 60. The average salary per head of is increased by 1, then the average of these 50
12 officers is` 400. The average salary per head numbers will be
of the rest is ` 56. Then the total number of (1) 21 (2) 22
workers in the institution is (3) 23 (4) 24
(1) 1062 (2) 1060
26. The average salary of 10 employees in an office is
(3) 1030 (4) 1032
` 10,000. If the salary of the CEO is added to the
17. The present ages of A and B are in the ratio of sum of the salaries of 10 employees, new average
4:3. If 4 years ago, the ratio of their ages was 2:1, salary would be ` 15000. What is the salary of the
the present age of A is CEO?
(1) 8 years (2) 10 years (1) ` 45000 (2) ` 50000
(3) 12 years (4) 15 years (3) ` 55000 (4) ` 65000
18. Present ages of father and the son are in the ratio 27. The average age of a husband and wife was 23
of 6:1 respectively. Four years hence, the ratio of years when they got married 5 years ago. At
their ages will become 4:1 respectively. What is present, the average age of the husband, wife and
the son's present age? son is 20 years. Find out the present age of the
(1) 10 years (2) 6 years son.
(3) 4 years (4) 8 years (1) 4 years (2) 2 years
(3) 6 years (4) 3 years
19. Four years ago the ratio between the ages of
Taani and Ananya was 4:5. The ratio between 28. If the average of 8 numbers is x, the average of
their present ages is 14:17. What will be Ananya's first four is y and the average of last four is z, then
age 5 years hence? which of the following relations is true?
(1) 39 years (2) 33 years (1) z = y + x (2) x = y + z
(3) 38 years (4) 28 years (3) 2x = y + z (4) x = 2y + z
20. The average age of a man and his son is 30 years. Directions (Q. 29–30): Consider the following table.
The ratio of their ages four years ago was 10:3
respectively. What is the difference between the Courses Institutes
present ages of the man and his son? A B C D E F G H
(1) 28 years (2) 16 years
(3) 26 years (4) 44 years P 520 410 550 450 570 210 750 450
21. The ages of Sangeeta and Mohan are in the ratio Q 410 540 610 580 380 550 570 310
of 7:2. Fifteen years hence, their ages would be in
R 430 210 590 530 730 510 530 480
the ratio 2:1. What was Sangeeta’s age when
Mohan was born? S 350 280 570 320 410 480 610 460
(1) 25 years (2) 30 years
T 370 480 380 250 180 370 590 660
(3) 35 years (4) 40 years
22. Among three numbers, the third number is half 29. What is the average number of students studying
the first number and also 1/3 of the second in institute D?
number. If the average of the three numbers is (1) 446 (2) 426
54, the largest number is (3) 540 (4) 454
(1) 54 (2) 27 30. What will be the average of the following set
(3) 26 (4) 81 of scores? (Round off to the nearest integer.)
23. The average of 2 numbers is 32. If 1 is added to 46, 54, 62, 68, 56, 29, 58
the larger number, the ratio between the numbers (1) 45 (2) 59
becomes 3:2. The larger number is (3) 62 (4) 53
(1) 25 (2) 26
(3) 38 (4) 42 31. A jar contains 20 litres of juice. If 2 litres of juice
is replaced by water thrice from the jar, identify
24. Jolly is 10 years older to his brother Monty. Find the quantity of juice left in the jar.
out the present age of Jolly, if 12 years ago, the (1) 14.42 (2) 16.28
age of Jolly was twice the age of Monty. (3) 18.26 (4) 14.58
(1) 32 years (2) 22 years
(3) 34 years (4) 36 years

________________________________________________________
537
Subhash Institute of Competitive Exams (SICE) Vijayawada
Vijaya Ph: 9030924949
________________________________________________________
Mathematics for Competitive Exams ( 70 )

32. 2 bottles of juice contain 20% and 40% of water, 36. Alloy 1 contains zinc, copper and tin in the ratio
respectively. Calculate the amount of juice to be 1:2:3. Alloy 2 contains copper, tin and lead in
added from each of the containers to get 14 litres ratio 4:5:3. Equal quantities of alloys 1 and 2 are
of juice containing water and juice in the ratio mixed to form alloy 3. What is the percentage of
2:5. lead in alloy 3?
(1) 7 litres, 7 (1) 9% (2) 11.5%
(2) 11 litres, 3 (3) 12% (4) 12.5%
(3) 10 litres, 4
37. A shopkeeper mixes two varieties of sugar
(4) 8 litres, 6
costing `28/kg and ` 38/kg. In what ratio should
33. Two vessels labeled as P and Q contains a he mix the two different varieties so that he can
mixture of ink and water in the ratio 4:1 and 6:5, sell it at` 36/kg including a profit of 20%?
respectively. Calculate the ratio in which these (1) 1:4 (2) 4:1
mixtures be mixed to obtain a new mixture in (3) 3:2 (4) 2:3
vessel R containing ink and water in the ratio 7:4.
38. 60 kg of an alloy A is mixed with 100 kg of alloy
(1) 3:4 (2) 4:3
B. If alloy A has lead and tin in the ratio 3 : 2 and
(3) 9:7 (4) 5:9
alloy B has tin and copper in the ratio 1 : 4, the
34. Some ink from a bottle containing 50% of blue ink amount of tin in the new alloy is
is replaced with a different bottle containing 20% (1) 80 kg (2) 53 kg
red ink and now the percentage of ink was found (3) 24 kg (4) 44 kg
to be 30%. What part of the blue ink was
39. A vessel contains 60 litres of milk. Six litres of
replaced?
milk is taken out and six litres of water is added
4 2
(1) (2) to the vessel. Again, six litres of milk from the
5 3 vessel is withdrawn and six litres of water is
3 3 added to the vessel. The ratio of milk and water in
(3) (4)
4 5 the resulting mixture in the vessel is
(1) 81:19 (2) 71:29
35. The average weight of 15 oarsmen in a boat is
(3) 61:39 (4) 61:29
increased by 1.6 kg when one of the crew, who
weighs 42 kg is replaced by a new man. Find out 40. The proportion of acid and water in three samples
the weight of the new man (in kg). is 3 : 4, 4 : 1 and 5 : 2, respectively. A mixture
(1) 43 containing equal quantities of all three samples is
(2) 65 made. The ratio between acid and water in the
(3) 67 mixture is
(4) 66 (1) 68:37 (2) 52:41
(3) 44:39 (4) 28:17

________________________________________________________
538
Subhash Institute of Competitive Exams (SICE) Vijayawada
Vijaya Ph: 9030924949
________________________________________________________
Mathematics for Competitive Exams ( 71 )
Section III – Chapter 7: Average, Mixtures and Alligation

ANSWERS AND EXPLANATIONS


1. (4) Let the container contain 8x Ratio of mixture in the 1st bottle By the rule of alligation,
and 3x kgs of mixture of pulses = 4:1
Ratio of the two in the mixture =
A and B, respectively.
4x 2 3 3 5 1 17
Tea in the 1st bottle = : : = 14:51
Quantity of mixture drawn off = 5 3 5 5 14 15 70
8 kg litres
The weight of the two that must
Quantity of pulse A drawn off in Milk in the 1st bottle = be melted together to make
8 x 130 lb
mixture = 8 x 8 litres
11 5 14
x mass = i.e. 28 lb and
= Ratio of mixture in the 2nd pot 65
11
= 7:3
51
Quantity of pulse B drawn off in = 102 lb
Tea in the 2nd bottle = 65
3 33 x 24
mixture = x = 7x
11 11 litres 5. (4) Let the weight of 12th
10 person = x kg
As the new ratio of the mixture
is 8:5 Milk in the 2nd bottle = Therefore, total weight of 12
3x persons = 11 × 95 + x = 1045
88 x 64 18 litres
10 +x
33 x 24 7
Ratio of mixture in the 3rd Average weight of 12 persons =
x=2 bottle 1045 x
Therefore, initial quantity of = 13:2 12
pulse A = 8 × 2 = 16 kg Tea in the 3rd bottle = According to the question, x =
2. (4) Let x be the quantity of each 13 x 1045 x
litres 33 +
melted alloy P and Q. 15 12

5x Milk in the 3rd bottle = 12x – x = 396 + 1045


Quantity of bronze in P =
11 2x
litres 11x = 1441
15
6x x = 131 kg
Quantity of platinum in P = Total Tea in the complete
11
4x 7 x 13 x 71 x 6. (1) By the rule of alligation,
9x mixture =
Quantity of bronze in Q = 5 10 15 30
22 Required ratio = (30.50 –
litres 25.75):(25.75 – 20.25) =
13 x Total milk in the complete 4.75:5.50 = 19:22
Quantity of platinum in Q =
22 x 3x 2x 19 x
mixture = 7. (4)
Total quantity of bronze in alloy 5 10 15 30
litres Quantity of water in the mixture
5x 9x 19 x
R= 5
11 22 22 Required ratio = 71:19 = = 500 ml
7
Total quantity of platinum in 2
4. (4) Steel in the Ist alloy = Quantity of sulphuric acid in the
6x 13 x 25 x 3
alloy R = 2
11 22 22 mixture = = 200 ml
5 7
19 x 25 x
Steel in the IInd alloy =
Required ratio = : 14 Let x ml of water is replaced
22 22 with sulphuric acid in the
60
= 19:25 Steel in the mixture = mixture
100
3. (3) As the volume of the three 3
bottles is same so let they =
5
contain x litres of mixtures each.

________________________________________________________
539
Subhash Institute of Competitive Exams (SICE) Vijayawada
Vijaya Ph: 9030924949
________________________________________________________
Mathematics for Competitive Exams ( 72 )

New quantity of water in the 11. (3) Sum of salary of 10 teachers 16. (4) Let the total number of
mixture = (500 – x) ml = ` 1500 × 10 workers be x.
And new quantity of sulphuric = ` 15000 Total salary =60x ………….. (1)
acid in the mixture = (200 + x)
Let principal’s salary be ` x. Salary of 12 officers = 12 × 400
ml
= ` 4800
Average when principal’s salary
According to the question,
is added = ` 2000 Salary of the rest = (x – 12) ×
(500 – x):(200 + x) = 1:1 56
15000 x
2000
x = 150 ml 11 Hence, totals salary of the
workers = (x – 12) × 56 + 4800
8. (4) Total income of 8 workers = 15000 + x = 22,000 …….. (2)
` 3645 × 8 = ` 29160
x = (22000 – 15000) Equating 1 and 2, we get:
Total income of 7 workers =
` (29160 – 3000) x = 7000 60x = 4800 + 56x – 672
= ` 26160 12. (2) (70m +91n)/ (m+n) = 80 4x = 4128
Average income of the 70m + 91n = 80m +80n X= 1032.
26160 10m = 11n 17. (1) Let A’s present age be 4x
remaining 7 workers = `
7 years and B’s present age be 3x
n/m = 10/11
= ` 3737.14 years
13. (4) Incorrect total price of 16
9. (4) Total rainfall on Monday, Then, 4 years ago, their ages
items = ` 80 × 15
Tuesday, Wednesday and will be:
Thursday is 420.5 × 4 = 1682 = ` 1200
4x 4 2
cm………..(1) Correct total price of 16 items = 3x 4 1
Total rainfall on Tuesday, ` [1200 – (30 + 15) + (40 +
20)] 6x 8 = 4x 4
Wednesday, Thursday, and
Friday is 440.5 × 4 = 1762 cm = ` 1200 – 45 + 60 2x = 4
Subtracting (1) from (2), we get = ` 1215 x=2
Rainfall on Monday- Rainfall on
Friday = 80 cm Correct average amount = Therefore, A’s present age =
1215 2×4 = 8 years
Now, that the ratio of rainfall on `
15 18. (2) Let father’s age be 6 x years
Monday and on Friday is 20:21,
the amount of rainfall on = ` 81 and son’s age be years.
Monday and Friday 1600 cm Then, 4 years hence, their ages
14. (3) Sum of first 30 natural
(20 ×80) and 1680 cm (21×80) will be
n(n 1)
respectively. numbers
2 6x 4 4
10. (2) Sum of first seven numbers x 4 1
= 7 × 4.5 = 31.5 30(30 1)
= 465
2 (6x + 4) = 4(x + 4)
Sum of last seven numbers
= 7 × 6.5 = 45.5 Therefore, average = 465/30 6x + 4 = 4x + 16
= 15.5
Average of 15 numbers = 9.6 6x – 4x = 16
15. (3) Sum of 6, 12, 18, ….. , 120
31.5 eighth no. 45.5 = 6 (1+2+3+…..+20) x=6
9.6
15
20(20 1) Therefore, son’s age = 6 years
Eighth number = (15 × 9.6 – Average 6
2 19. (1) Let Tanni’s present age be
31.5 – 45.5) 14 x years and Ananya’s
6 20(20 1)
= 144 – 77 = 63 present age be 17x years
20 2
= 67

________________________________________________________
540
Subhash Institute of Competitive Exams (SICE) Vijayawada
Vijaya Ph: 9030924949
________________________________________________________
Mathematics for Competitive Exams ( 73 )
Section III – Chapter 7: Average, Mixtures and Alligation

Then, 4 years ago, their ages Mohan’s present age = 2 × 5 = Therefore, the present age of
were 10 years Jolly = 22 + 10 = 32 years
14 x 4 4 Sangeeta was 25 years old 25. (4) Average of 50 numbers = 23
17 x 4 5 when Mohan was born because
Therefore, sum of 50 numbers
the difference between their
5(14x 4(17x = 23 × 50 =1150
ages is 25 years.
70x – 20 = 68x If each number is increased by
22. (4) Let the three numbers be x,
1, then the new sum of 50
y and z.
70x – 68x = 20 numbers = 1150 + 50 = 1200
According to the question,
x=2 Therefore, new average
x y 1200
Therefore, Ananya’s present age z= &z= = = 24
2 3 50
= 17 × 2 = 34 years
Ananya’s age after 5 years x = 2z & y = 3z 26. (4) Sum of salaries of 10
employees = 10 × 10000
= 34 + 5 = 39 years Average of 3 numbers = 54
= ` 100000
20. (1) The sum of the ages of man 2z 3 z z
54 Sum of salaries of 10
and his son = 2 × 30 = 60 years 3
employees + Sum of salary of
Therefore, let the present age of z = 27 CEO =15000 × 11
the man is x years and his son’s
x = 2 × 27 = 54 = ` 165000
age is (60 x) years.
& y = 3z = 3 × 27 = 81 Therefore, the salary of CEO =
Four years ago,
` 165000 – ` 100000
Largest number is y = 81
x 4 10
= ` 65000
60 x 4 3 23. (3) Let the larger number be x
and the smaller number be y. 27. (1) Sum of present ages of
3x 12 = 560 x husband and wife = 23 × 2 +
Average of 2 numbers = 32
10 = 56
x = 44
x+y
= 32 Sum of present ages of
Therefore, the man’s age is 44 2 husband, wife and son = 20 × 3
years and the son’s age is
x + y = 64 …(1) = 60
= (60 years
If 1 is added to the larger Age of son = 60 – 56 = 4 years
= 16 number, the ratio becomes 3:2. 28. (3) Assume that 8 numbers are
The difference between their x 1 3 = 1, 2, 3, …., 8
ages = 44 16 = 28 years y 2 (1 2 3 8)
21. (1) Let Sangeeta’s present age Average =
2(x + 1) = 3y …(2) 8
be 7x years and Mohan’s = 4.5
present age be 2x years. By solving (1) and (2), we get
10
Then, 15 years hence, their x = 38 & y = 26 Average of 1, 2, 3 and 4 =
4
ages will be: = 2.5
Larger number is 38.
7 x 15 2 26
24. (1) Let the age of Monty be x Average of 5, 6, 7 and 8 =
2 x 15 1 years. 4
= 6.5
(7x +15) = 2(2x + 15) and Jolly be (x + 10) years.
It is observed that 2(Average of
7x + 15 = 4x +30 Therefore, (x + 10) – 12 1, 2, … 8) = Average of 1, 2, 3, 4
= 2(x – 12) + Average of 5, 6, 7, 8
x=5
Therefore, Sangeeta’s present x – 2 = 2x – 24 Therefore, by observation,
age = 7 × 5 = 35 years and x = 22 2x = y + z

________________________________________________________
541
Subhash Institute of Competitive Exams (SICE) Vijayawada
Vijaya Ph: 9030924949
________________________________________________________
Mathematics for Competitive Exams ( 74 )

29. (2) Average = Quantity of water in 5 litre 36. (4) Quantity of lead in alloy 1
450 580 530 320 250 1 =0
mixture from vessel P = litre
5 5 Quantity of lead in alloy 2 =
2130 Let the total quantity of mixture 3 1
426
5 in vessel Q = 1 litres 12 4
30. (4) Average = Quantity of ink in 1 litre mixture Since alloys 1 and 2 are mixed
46 54 62 68 56 29 58 6 in equal quantities, the quantity
from vessel Q = litres of lead in alloy 3 will be 50% of
7 11
53.28 that contained in alloy 2 =
Quantity of water in 1 litre 1 1 1 1
31. (4) Quantity of juice in the jar 5 Value of means
mixture from vessel Q = 2 4 8 8
= 20 litres 11 that alloy 3 contains 12.5% of
litres lead.
Therefore, the quantity of juice
2 3 Let x litres of mixture from 37. (2) Given that the selling price
left in the jar = 20 (1 )
20 vessel P and y litres of mixture of the sugar mixture = ` 36/kg
= 14.58 litres from vessel Q are added to including 20% profit
make a new mixture in vessel R
32. (4) Let x litres of juice be mixed Let the cost price of sugar
from the Ist bottle and (14 – x) According to the question, mixture = ` x/kg
litres of juice be mixed from the
4x 6y According to the question,
second bottle. 7
5 11
Quantity of juice from first bottle x 5y 4 x + 20% of x = 36
= 80% of x = 0.8x 5 11 x = ` 30/kg
Quantity of water from the first 4(44x + 30y) = 7(11x + 25y) Now, we can calculate the ratio
bottle = 20% of x = 0.2x of mixing two varieties of sugar
x:y=5:9
Quantity of juice from the by alligation formula as:
34. (2) Percentage of blue ink in the
second bottle = 60% of (14 – x) n1 (38 – 30) 8
1st bottle = 50% Percentage of 4 :1
= (8.4 – 0.6x) n2 (30 – 28) 2
ink in 2nd bottle = 20% After
Quantity of water from the replacement, the percentage of
38. (4) Amount of tin in alloy A =
second bottle = 40% of (14 – x) blue ink in the mixture = 30%
2
= (5.6 – 0.4x) 60 = 24 kg
5
According to the question,
Amount of tin in alloy B =
Total quantity of water in 1
100 = 20 kg
mixture: Total quantity of juice 5
in the mixture = 2:5
Amount of tin in new alloy
(0.2x + 5.6 – 0.4x): (0.8x + = 24 + 20 = 44 kg
8.4 – 0.6x) = 2:5
39. (1)
5.6 0.2 x 2 Ratio of the 1st
and the 2nd
8.4 0.2 x 5 quantities = 10 : 20 = 1 : 2
x=8 2 2
i.e. part of the blue ink a–b
n

33. (4) Let the total quantity of 1 2 3 =


is being replaced. a
mixture in vessel P = 1 litre
Quantity of ink in 1 litre mixture 35. (4) Weight of new oarsman =
4 (42 + 15 × 1.6) kg. 60
from vessel P = litres
5 = (42 + 24) kg. = 66 kg. 60 6
2
486
81 : 19
60 114

________________________________________________________
542
Subhash Institute of Competitive Exams (SICE) Vijayawada
Vijaya Ph: 9030924949
________________________________________________________
Mathematics for Competitive Exams ( 75 )
Section III – Chapter 7: Average, Mixtures and Alligation

40. (1) Let quantity of each sample Total quantity of water in three 3 4 5 15 28 25
be x units. x +
4 1 2 7 5 7 35
samples = x
Total quantity of acid in three 7 5 7 4 1 2 20 7 10
x
7 5 7 35
3 4 5 Ratio of acid and water in the
samples = + x 68
7 5 7 new mixture = = = 68 : 37
37

________________________________________________________
543
Subhash Institute of Competitive Exams (SICE) Vijayawada
Vijaya Ph: 9030924949
________________________________________________________
Mathematics for Competitive Exams ( 76 )

8 TIME AND WORK

TIME AND WORK


Time and work problems are important because there is a certain relationship between the
number of persons doing the work, number of days or time taken by them to complete the
work and the amount of work that is done.

Work done = Number of persons Number of days (or amount of time)


W=M D ……………(Eq. 1)
Here, the unit of work is man-days or man-hours.
The problems of time and work can primarily be divided into two types. The first type includes
problems where individuals work with different efficiencies either alone or in combination to
complete a task. The second type includes problems where group efficiencies are involved.
Please remember that, if we have a person ‘A’ who completes a certain work in ‘a’ days, then
A’s one-day work is equal to 1/a.
Assume that two persons can complete the given work in ‘a’ and ‘b’ days, respectively. Then,
their one-day work together is equal to

1 1 a b ……………(Eq. 2)
a b ab

Short Trick
The number of days or time required to complete the work by A and B both is equal to the
reciprocal of Equation 2 as:
ab
Eq 3
a+b SMART TIP

Once you calculate the combined rate of two or more things, the total
time required to finish the task will simply be the reciprocal of this rate.
For example, if the combined rate of A and B is 2/9, the time they will take
to finish the work is 9/2 = 4.5 days.

________________________________________________________
Subhash Institute of Competitive Exams (SICE) Vijayawada
Vijaya Ph: 9030924949
________________________________________________________
Mathematics for Competitive Exams ( 77 )
Section III – Chapter 8: Time and Work

Please note that the concept of work can be applied to third of the time taken by B to do the same amount of
people doing some work, pipes filling tanks or any such work.
similar situation.
Illustration 2: Assume that 10 men can do a particular
work in 5 days and 15 women can do the same work in
Negative Work
Assume that there are two people A and B (carpenters) 10 days. Calculate the number of days 5 men and 5
who have been assigned to build a table. Here, work women will take to do that work.
refers to the building of a table. Also assume that there Solution:
is another person C who is assigned the task of
Amount of work = 10 × 5 = 50 man-days
damaging the table. Now, work done by C can be termed
as negative work, i.e. C is effectively lowering the amount Alternatively, amount of work = 15 × 10 = 150 woman-
of work done by A and B. days

Work Equivalence It means that the same amount of work can take 50
Please note that the work done by people/pipes can be man- days or 150 woman-days.
stated mathematically as: 50 man-days = 150 woman-days
Work done = Rate of work × Time ..................…(Eq. 4) which means that 1 man-day = 3 woman-days
Consider the following formula also: Or,
M1D1T1 = M2D2T2 (provided work remains the 1 man = 3 women
same)…(Eq. 5)
Now, we can calculate the number of days taken by 5
In the above formula, men and 5 women to complete the work as follows:
M = Number of persons 5 men + 5 women = 15 women + 5 women = 20 women
D = Number of days For 20 women to do 150 woman-day work, we require
T = Number of hours 15
= 7.5 days
2
Equation 5 is called the ‘work equivalence formula’.
The use of this formula can be explained by the following Problems Involving Individual Efficiencies
illustration: In such questions, the rates at which some individuals
complete a work alone are given and you are required to
Illustration 1: Assume that 10 persons can complete a calculate the rate at which they can complete the work
given work in 10 days working 6 hours per day. How together (or, vice versa). The basic formula for solving
many persons can complete the same work in 8 days such type of problems is:
working 3 hours per day?
1 1 1
Solution: ........................ (Eq. 6)
a b c
M1D1T1 = M2D2T2 where a and b are the times taken by two individuals to
10 × 10 × 6 = M2 × 8 × 3 complete a job while working alone and c is the number
of hours taken by them to complete the job together.
M2 = 25
ab
Refer to Equation 3 and note that here, c = …(Eq. 7)
a b
CONCEPT OF EFFICIENCY
Very often, you will get to read and solve time–work Problems Involving Group Efficiencies
problems where it is usually mentioned that the Till now, we have looked at problem types where
efficiency of A is twice the efficiency of B or that A is two individuals were working. We will now look at problems
times more efficient than B. In the first case, it means where people with the same efficiencies are working in
that A takes one half of the time taken by B to do the groups. Let’s try to understand this by an example.
same amount of work. In the second case, it means that Illustration: If 15 men work for 6 hours in a day to
A’s efficiency is thrice that of B and so A will take one- complete a work in 40 days, in how many days can 20

________________________________________________________
545
Subhash Institute of Competitive Exams (SICE) Vijayawada
Vijaya Ph: 9030924949
( 78 )
________________________________________________________
Mathematics for Competitive Exams

men complete the same work if they work 8 hours per Rules that give shortcut formulae for questions related to
day? pipes and cistern:
Solution: The easiest way to solve such type of problems I. Working alone, if two pipes can fill or empty the
is to equate the total work to be done. cisterns in x hours and y hours respectively, and
then working together they will fill or empty the
Therefore, the total work is 15 × 6 × 40
xy
It is the same as 20 × 8 × D entire cistern in hours .
x y
where D is the number of days. Example: Pipes P and Q can fill a cistern in 30 and
15 × 6 × 40 = 20 × 8 × D 45 minutes respectively. How long will it take to fill
the cistern when both are opened together?
Or, D = 45/2 = 22.5 days
Solution: Here, x = 30 minutes and y = 45 minutes.
SMART TIP
When both are opened together, time they will take
If A can do a work in X days, B can do
to fill the cistern will be,
it in Y days and C can do it in Z days,
then A, B and C can together 30 45
18 minutes
complete the work in 30 45
XYZ
= days II. Working alone, if three pipes can fill or empty the
XY XZ YZ
cisterns in x hours, y hours and z hours respectively,
If A and B can together complete a
and then working together they will fill or empty the
work in X days and A alone can
complete the work in Y days, then B xyz
entire cistern in hours .
alone can complete that work in xy yz zx
= XY/(Y – X) days Example: Three pipes A, B and C when opened alone
can empty a filled tank in 2, 3 and 5 hours
Pipe and Cistern respectively. When all the pipes are opened
Similar to time and work, problems related with pipe and together, then how long will they take to empty the
cisterns are solved. Pipes are either used to empty the tank?
cistern or to fill the cistern. Pipe filling the cistern is said Solution: Here, x = 2 hours, y = 3 hours and z = 5
to be doing positive work and those emptying the cistern hours. When opened together, they will empty the
is called to be doing a negative work. tank in,
There are two types of pipe: 2 3 5 30 9
1 hours
Inlet Pipe: Those which are used to fill the cistern or 2 3 3 5 5 2 21 21
which do positive work. III. Working alone, if one pipe can fill a cistern in x hours
Outlet Pipe: Those which are used to empty the and other can empty the cistern in y hours
cistern or which do negative work. respectively, and then working together they will fill
xy
Basic Concepts or empty the entire cistern in hours .
x y
If inlet pipe fills the cistern in x hours and outlet pipe
empties in y hours, then: Example: A pipe can fill a cistern in 4 hours. Due to
a small hole at the bottom of the cistern, it takes 5
Part of cistern filled in one hour when only inlet pipe hours to fill. How long will it take the hole to empty
1
is opened is . the entire cistern?
x
Solution: Let the time taken by the hole to empty the
Part of cistern emptied in one hour when only outlet
cistern is x hours. Now,
1
pipe is opened is . 1 1 1
y
4 x 5
When both inlet and outlet pipes are opened, part of
1 1 5x – 20 = 4x
cistern filled or emptied is ±
x y x = 20 hours

________________________________________________________
546
Subhash Institute of Competitive Exams (SICE) Vijayawada
Vijaya Ph: 9030924949
________________________________________________________
Mathematics for Competitive Exams ( 79 )
Section III – Chapter 8: Time and Work

IV. If two pipes can fill a cistern in x hours and y hours How long will it take to fill the tank all three of them
respectively and third can empty in z hours, then are opened together?
working together they can fill the cistern in
Solution: Here, x = y = z = 5 hours. Time taken by
xyz them together to fill the tank will be,
hours .
yz xz xy
5 5 5 125
5 hours
Example: Pipe A and B are used to fill a tank and 5 5 5 5 5 5 25
pipe C is used to empty the tank. All three can do
their respective jobs in 5 hours when opened alone.

________________________________________________________
547
Subhash Institute of Competitive Exams (SICE) Vijayawada
Vijaya Ph: 9030924949
________________________________________________________
Mathematics for Competitive Exams ( 80 )

PRACTICE QUESTIONS
Answer the following questions by selecting the most appropriate option.
1. A company employed 200 workers to complete a 7. Two pipes can fill a tank in 8 and 10 min,
1 respectively. If both are operated together, how
certain work in 150 days. If only th of the work
4 much time is needed to fill the tank?
has been done in 50 days, then in order to 40 40
(1) min (2) min
complete the whole work in time, the number of 3 9
additional workers to be employed was 9 3
(1) 300 (2) 200 (3) min (4) min
40 40
(3) 100 (4) 600
8. A and B can do a piece of work in 30 and 36 days,
1 respectively. They began the work together but A
2. x does th of a job in 6 days. y completes the
4 left after some days and B finished the remaining
rest of the job in 12 days. Then x and y could work in 25 days. After how many days did A
complete the job together in leave?
3 1 (1) 10 days (2) 6 days
(1) 9 days (2) 7 days
5 3 (3) 5 days (4) 11 days
1
(3) 9 days (4) 8 days 9. P and Q together can do a job in 6 days. Q and R
8 can finish the same job in 60/7 days. P started
3. Raj can do a work in 10 days and Mohit can do the work and worked for 3 days. Q and R
the same work in 20 days. How many days will continued for 6 days. Then the difference of days
they take to complete the work if they work in which R and P can complete the job is
together? (1) 12 (2) 10
20 (3) 8 (4) 15
(1) 15 days (2) days
3 10. If 90 men can do a certain job in 16 days, working
3 12 hours/day, then the part of that work which
(3) 6 days (4) days
20 can be completed by 70 men in 24 days, working
8 hours/day is
4. Nikhil, Ashish and Raghav can complete a work in
7 2
4, 8 and 12 days, respectively. Together they can (1) (2)
complete the work in 9 3
1 5
24 (3) (4)
(1) 10 days (2) days 3 8
3
21 24 11. John travelled 240 km at a certain speed and
(3) days (4) days
8 11 then another 192 km at 8 km/h faster. If the
entire trip of 10 hr, find his initial speed.
5. Ram and Shyam together can do a work in 10 (1) 45 km/h (2) 60 km/h
days. Ram alone can do the work in 25 days. How (3) 40 km/h (4) 75 km/h
many days will Shyam alone take to do the work?
25 12. Pipe A can fill an empty tank in 6 hours and pipe
(1) 15 (2) B in 8 hours. If both the pipes are opened and if
2
3 50 after 2 hours, pipe A is closed, then how much
(3) (4) time will B take to fill the remaining tank?
25 3
1 1
6. A and B together can do a work in 5 days. A alone (1) 2 hours (2) 7 hours
3 2
can do half of the work in 4 days. How many days 1 2
will B take to complete the work? (3) 3 hours (4) 2 hours
3 5
40
(1) 13. 75 men working 8 hours a days can do a work in
3
(2) 10 27 days. How many days are required by 45 men
(3) 9 working 12 hours a day to complete the work?
3 (1) 24 (2) 30
(4) (3) 36 (4) 32
40

________________________________________________________
548
Subhash Institute of Competitive Exams (SICE) Vijayawada
Vijaya Ph: 9030924949
________________________________________________________
Mathematics for Competitive Exams ( 81 )
Section III – Chapter 8: Time and Work

14. A alone can complete a task in 44 days while A than working together. The time required to finish
and B together can finish the task in 11 days. In the work together is
how many days can B alone finish the same task (1) 4 hours (2) 6 hours
working alone? (3) 5 hours (4) 8 hours
34 44
(1) (2) 21. A man is twice as fast as a woman and a woman
3 5 is twice as fast as a boy in completing a piece of
20 44
(3) (4) work. If all of them, a man, a woman and a boy,
11 3 can finish the work in 7 days, then in how many
15. Vijay is thrice as good a worker as Ajay. If these days will a boy do it alone?
two can finish the work in 13 days by working (1) 49 (2) 7
together, how many days are required by Ajay to (3) 6 (4) 42
finish the same work working alone? 22. A, B and C can do a job in 6 days, 12 days and 15
(1) 52 (2) 39 1
(3) 26 (4) 21 days respectively. After of the work are
8
16. Pipe P1 and P2 can fill the same tank in 84 completed, C leaves the job. Rest of the work is
minutes and 56 minutes, respectively. Another done by A and B together. Time taken to finish the
pipe P3 can empty the same tank in 70 minutes. work is
How much time (in minutes) is required to fill the 5 1
(1) 5 days (2) 5 days
tank if all three pipes are opened together? 6 4
13 6 1 3
(1) 32 (2) 58 (3) 3 days (4) 3 days
18 23 2 4
18 8
(3) 60 (4) 64 23. 15 men take 20 days to complete a job working 8
23 13 hours a day. The number of hours a day should 20
17. Sarita, Lalita and Punita and can do a work in 7 men take to complete the job in 12 days will be
days, 14 days and 21 days, respectively. How (1) 5 hours (2) 10 hours
much time is required to complete the work if all (3) 15 hours (4) 18 hours
three of them work together? 24. Having the same capacity, 9 taps fill up a water
36 52 tank in 20 minutes. How many taps of the same
(1) days (2) days
7 15 capacity are required to fill up the same water
22 42 tank in 15 minutes?
(3) days (4) days
5 11 (1) 10 (2) 12
(3) 15 (4) 18
18. A water tank takes 45 minutes to be filled by a
tap but due to leakage, it takes another 15 25. Raj and Ram working together complete a piece
minutes. In how many hours leakage can empty of work in 10 days. Raj alone can do it in 12 days.
the full water tank? Ram alone will complete the work in
(1) 3 hours (2) 2.4 hours (1) 20 days (2) 40 days
(3) 1.8 hours (4) 3.6 hours (3) 50 days (4) 60 days
19. A tank has two pipes. The first pipe can fill it in 4 26. A can work thrice as fast as B. Together, they can
hours and the second one can empty it in 16 finish a work in 6 days. How many days will B
hours. If two pipes are opened together at a time, alone take to finish the work?
then the tank will be filled in (1) 20 (2) 15
(1) 10 hours (3) 10 (4) 24
1
(2) 5 hours 27. Efficiencies of doing a work of Hari and Rahul are
3
in the ratio 4: 1. If they can complete the work
1
(3) 5 hours together in 10 days, how many days will Rahul
2 take alone to complete the work?
(4) 6 hours (1) 50 (2) 40
20. Two workers A and B are engaged to do a piece of (3) 30 (4) 10
work. A working alone would take 8 hours more to 28. Sheela and Jyoti can complete a work in 4 and 8
complete the work than when working together. If days, respectively. They work together and earn `
1 1200 for the work. What is the share of Sheela?
B worked alone, he would take 4 hours more
2 (1) ` 800 (2) ` 1200
(3) ` 1000 (4) ` 600

________________________________________________________
549
Subhash Institute of Competitive Exams (SICE) Vijayawada
Vijaya Ph: 9030924949
________________________________________________________
Mathematics for Competitive Exams ( 82 )

29. 10 women can do a work in 10 days. 5 men can 35. 4 men can complete a piece of work in 2 days. 4
do the same work in 12 days. Find the ratio of women can complete the same piece of work in 4
efficiency of men and women in completing the days, whereas 5 children can complete the same
work. piece of work in 4 days. If 2 men, 4 women and 10
(1) 1 : 2 (2) 2 : 3 children work together, in how many days can the
(3) 3 : 4 (4) 5 : 3 work be completed?
(1) 1 day (2) 3 days
30. X and Y together can paint a wall in 4 days. X
(3) 2 days (4) 4 days
alone can paint the wall in 6 days. After working
together for 2 days, X left the work. How many 36. 8 boys and 6 girls can complete a work in 6 days.
days more will Y need to finish the work? 10 boys and 12 girls can complete the same work
(1) 6 (2) 5 in 4 days. How long will 18 boys and 9 girls take
(3) 4 (4) 3 to finish the same work?
(1) 5 days (2) 3 days
31. Taps P and Q can fill a tank in 5 min and 10 min,
(3) 7 days (4) 4 days
respectively. After operating both taps together
for 2 min, tap P is closed. How much more time is 37. A, B and C are employed to do a piece of work for
needed to fill the tank using Q only? 19
` 575. A and C are supposed to finish of the
(1) 7 min (2) 8 min 23
(3) 6 min (4) 5 min work together. Amount that shall be paid to B is
32. A pipe can fill a tank in 32 hr. After 8 hr, 7 more (1) ` 210 (2) ` 100
similar pipes are also used. What is the additional (3) ` 200 (4) ` 475
time required to fill the tank? 38. Sanjay completes 40% of a work in 10 days.
(1) 3 hr (2) 2 hr Sunny is 25 % more efficient than Sanjay and
(3) 5 hr (4) 4 hr Dharam is 100 % more efficient than Sunny. In
33. If 20 workers take 15 days to build a building, how many days, work will be finished if they do
how many days will 25 workers take to build the the work together?
84 80
same building? (1) (2)
(1) 15 days 19 19
(2) 10 days 90 100
(3) (4)
(3) 18 days 19 19
(4) 12 days 39. 40 men and 28 women can complete a work in 5
34. A tank can be filled by a single tap in 4 hours and days. 4 men and 4 women can complete the same
by another tap in 2 hours. How long will it take to work in 45 days. How many days are required by
fill the tank if both the taps are opened together? 16 men and 4 women to complete the work?
3 (1) 15 (2) 18
(1) hours (3) 21 (4) 16
4
3 40. Two persons took the contract for a job for `
(2) hours
2 7600. They complete the job in 20 and 30 days,
11 respectively. If the work is completed with the
(3) hours
3 help of third person in 9 days, find the share of
4 the third person.
(4) hours
3 (1) ` 2100 (2) ` 1700
(3) ` 1600 (4) ` 1900

________________________________________________________
550
Subhash Institute of Competitive Exams (SICE) Vijayawada
Vijaya Ph: 9030924949
________________________________________________________
Mathematics for Competitive Exams ( 83 )
Section III – Chapter 8: Time and Work

ANSWERS AND EXPLANATIONS


1. (3) In 50 days, 200 workers 1 1 6. (1) A will need 8 days to
Together, they can do of
1 1 20 complete the work
complete th work.
4 3
the work = of the work 1
20 So, in 1 day, A can do of the
3 8
Remaining work = 1
4 Together, they will take work
3
Remaining days = 150 – 50 = 20 1
In 1 day, A and B can do of
100 20 5
days = days the work
Let x be the extra number of 30
workers required 4. (4) In 1 day, 1
in 1 day, B alone can do –
5
200 50 (200 x) 100 1
Nikhil can do of the work 1 3
1 3 4 = of the work
8 40
4 4
1
600 = 400 + 2 Ashish can do of the work B alone will need
40
days to
8
3
x = 100 1 complete the work.
Raghav can do of the work
1 12 1
2. (1) x does job in 6 days. 7. (2) In 1 min, pipe 1 and fill of
4 1 1 8
Together, they can do+ +
Work done by x in 1 day 4 8 1
the tank and pipe 1 can fill
1 24 10
1 of the work = of the
= 12 11 of the tank
24
work
3 1
Remaining work = 1 So, together they can fill +
Together, they will take
 8
4 11
1 9
3 24 of the tank = of the
y does work in 12 days. 10 40
4 24 tank
days = days
11
work done by y in 1 day 40
1 5. (4) In one day, Time required = minutes
= 9
16 1
Ram + Shyam can do of the 8. (3) Work done by B in last 25
work done by x and y 10
25
together in 1 day = work days =
36
1 1 5 1
Ram alone can do of the Work done by A and B together
16 24 48 25
work 25 11
Required time taken =1 =
36 36
1
3 Shyam alone can do –
= 9 days 10 Work done by A and B together
5
1 3 in
of the work = of the
3. (2) In 1 day, 25 50 1 1 11
1 day = =
work 30 36 180
1
Raj can do of the work 1
10 Shyam alone will take Number of days taken by A and
3
1 50 11
Mohit can do of the work B together to finish th of
20 36
50
days to finish the work = work
3
days

________________________________________________________
551
Subhash Institute of Competitive Exams (SICE) Vijayawada
Vijaya Ph: 9030924949
________________________________________________________
Mathematics for Competitive Exams ( 84 )

11 180 240(x + 8) + 192x = 10(x² + 17. (4) Let the total work = LCM
= =5 (7, 14, 21) = 42 units
36 11 8x)
Number of days for A = 5 10x² – 352x –1920 = 0 One day work of all three person
42 42 42
9. (2) Let P, Q, R do the work alone x = 40, –24/5 together = + + = 11
7 14 21
in p, q, r days, respectively.
By ignoring the negative value, units
1 1 1 the required speed of John = 40 Time required to finish the work
=
p q 6 km/h
42
by working together = days
1 1 7 12. (3) Portion of tank filled in 2 11
= 2 2 7
q r 60 hours = = 18. (1) Let time taken by leakage to
6 8 12 empty the tank = X minutes
3
Work done by P in 3 days = Remaining portion of tank
p 1 1 1
7 5 –
Work done by Q and R together =1– = 45 X 45 15
12 12
7 7 1 1 1 1
in 6 days = ×6= Time taken by B to fill the = – =
60 10 X 45 60 180
5
remaining portion of tank =
3 7 3 12 X = 180
+ =1
p 10 p 1 5 1
÷ = × 8 = 3 hours Time taken by leakage to
8 12 3
3 empty the tank = 180 minutes
= p = 10 13. (2) Let 45 men working 12
10 = 3 hours
hours a day can finish the work
1 1 1 1 in D days. 19. (2) Change in water level
= = through first pipe in an hour
q 6 10 15
Then, 75 × 27 × 8 = 45 × 12 × 1
1 7 1 1 D =
= = r = 20 4
r 60 15 20
D = 30 Change in water level through
Required difference = 20 – 10 14. (4) Let B alone can finish the 1
second pipe in an hour =
= 10 days task in b days. 16
10. (1) Let the amount of work done Overall change in water level in
1 1 1
by 90 men be W1 and by 70 Then, = 1 1 3
44 b 11 an hour =
men be W2. 4 16 16
1 3 44
90 16 12 70 24 8 = b= Total time required to fill the
Then, = b 44 3
W1 W2 16 1
tank = hours = 5 hours
15. (1) Let Ajay takes X days to 3 3
Part of work completed by 70 finish the work working alone.
men in 24 days, working 8 20. (2) Let A and B together
W 3 1 1 complete the work in ‘N’ hours.
hours = 2 Then, =
W1 X X 13 Number of hours taken by A
4 1 alone to complete the work = N
70 24 8 7 X = 52
= = X 13 +8
90 16 12 9
16. (4) Part of tank filled in 1 Number of hours taken by B
11. (3) Let the initial speed of John alone to complete the work = N
be x km/h 1 1 1 13
minute = = + 4.5
84 56 70 840
According to the question, 1 1 1
Time required to fill the tank Therefore,
240 192 N N 8 N 4.5
840 8
x x = = 64 minutes
13 13 1 N 4.5 N 8
N N2 12.5N 36

________________________________________________________
552
Subhash Institute of Competitive Exams (SICE) Vijayawada
Vijaya Ph: 9030924949
________________________________________________________
Mathematics for Competitive Exams ( 85 )
Section III – Chapter 8: Time and Work

N2 = 36 24. (2) 28. (1) Efficiency of Sheela = 1/4

N = 6 hours Since 9 taps take 20 minutes, Efficiency of Jyoti = 1/8

21. (1) so, 1 tap will take 20 × 9 = 180 Ratio of efficiency of Sheela and
minutes to fill up the tank. Jyoti = 2 : 1
Let the time taken by boy alone
be x. Let x taps will fill up three tanks Let their share of pay be 2x and
in 15 minutes. x, respectively.
1 2 4 1
x x x 7 180 2x + x = 1200
Thus, = 15
4 2 1 1 x
3x = 1200
x 7 x = 12 taps
x = 400
x = 49 days 25. (4)
Sheela’s share = ` 800
22. (3) Assume that Ram alone can do
the work in x days. 29. (4) 1 woman can complete the
1 1 1 1
work in 10 × 10 = 100 days
6 12 15 x According to question,
10 5 4 1 In 1 day, 1 woman can
60 x 1 1 1 complete 1/100 of the work
19 1 12 x 10
x 12 1 1 man can complete the work in
60 x
12x 10
5 × 12 = 60 days
19
So, work done in 1 day = In 1 day, 1 man can complete
60 10x + 120 = 12x
1/60 of the work
60 2x = 120
= Complete work is done in Ratio of efficiency of man to
19
x = 60 days woman is (1/60) : (1/100) =
days
100 : 60 = 5 : 3
26. (4) Let A can complete the work
1
work is done in in x days 30. (1) In 1 day,
8
60 1 15 So, B can complete the work in Y alone can do 1/4 – 1/6 =
days
19 8 38 3x days 1/12 of the work in one day

Now, let work done by A and B In one day, they can together Remaining work = 1/2 (they
together is in y days complete 1/x + 1/3x of the work 2 days together)
work = 4/3x of the work
1 1 7 Time required by Y to finish
y 4/3x = 1/6 the remaining work =
6 12 8
(1/2)/(1/12) = 6 days
x=8
7 12 7 1
y 3 31. (3) In 1 min,
8 3 2 2 Time taken by B to finish the
work by working alone = 3 × 8 = Tap P can fill 1/5 of the tank
Therefore, time taken to finish
24 days Tap Q can fill 1/10 of the tank
1
the work is 3 days.
2 27. (1) Let the efficiencies of Hari Tap P + Q can fill 1/5 + 1/10 =
and Rahul be 4x and x 3/10 of the tank
23. (2)
Number of man hours = 15 × Together they can do 4x + x = In 2 min, Tap P and Q together
20 × 8 men hours 5x of the work in a day can fill 6/10 = 2/5 of the tank
So, So, 5x = 1/10 Remaining part after 2 min = 1
x = 1/50 – 2/5 = 3/5 of the tank
Number of hours 20 men take
15 20 8 Rahul alone will take Required time by Tap Q =
in 12 days = = 10
20 12 1/(1/50) days = 50 days to (3/5)/(1/10) = 6 min
hours complete the work.

________________________________________________________
553
Subhash Institute of Competitive Exams (SICE) Vijayawada
Vijaya Ph: 9030924949
________________________________________________________
Mathematics for Competitive Exams ( 86 )

32. (1) The pipe can fill 1/32 of the 1 One day work of all three
1-day work of 10 children =
tank in 1 hr 2 1 1 1
persons = + +
After 8 hr, remaining part = 3/4 25 20 10
1-day work of 2 men, 4 women
1 1 1
19
Remaining time = and 10 children = + + =
4 4 2 100
(3/4)/(8×1/32) = 3 hr
=1 100
33. (4) Number of days required by Required time = days
Days required to finish the 19
1 worker to build the building =
15 × 20 work = 1 39. (1) Let work done by man and
36. (2) Let B and G be the efficiency woman in one day be m and w,
Therefore, the number of days
of boy and girl, respectively. respectively.
required by 25 workers to build
Now, (8B + 6G) × 6 = 4(10B + Now
the same building =
25 12G) (40 m + 28 w)5 = (4 m + 4 w)45
= 12 days
B = 1.5G 40 m + 28 w = 36 m + 36 w
34. (4) Time taken by the first tap to
fill the tank = 4 hours Let number of days required by 4m=8w
18 boys and 9 girls be D.
Time taken by the second tap to Let D days are required by 16
fill the tank = 2 hours (8B + 6G)× 6 = (18B + 9G) × D men and 4 women to complete
(8 × 1.5G + 6G)× 6 = (18 × the work.
Tank filled by the first tap in 1
1 1.5G + 9G) × D (4 m + 4 w)45 = (16 m + 4 w)D
hour =
4 D = 3 days (8 w + 4 w)45
Tank filled by the second tap in 37. (2) = (32 w + 4 w)D
1
1 hour = 19 4 D = 15 days
2 Work done by B = 1
23 23 40. (4) Let the total work = LCM
Thus, the tank filled by both the
Amount paid to B = (20, 30) = 60 units
1 1 3
taps in 1 hour = 4
4 2 4 575 ` 100 Work done by first and second
23 person together in one day =
Time taken by both the taps
38. (4) Time taken by Sanjay to 60 60
4 = 5 units
to fill the tank = hours 10 20 30
3 complete the work = = 25
0.4 Work done by first and second
1
35. (1) 1-day work of 4 men = One day work of Sanjay persons together in 9 days = 9
2
× 5 = 45 units
1
1 =
1-day work of 2 men = 25 Work done by third person = 60
4
– 45 = 15 units
Now, One day work of Sunny =
1 Share of third person =
1-day work of 4 women = 1.25 1
4 =
25 20 15
7600 = ` 1900
1 60
1-day work of 5 children = One day work of Dharam = 2 ×
4
1 1
=
20 10

________________________________________________________
554
Subhash Institute of Competitive Exams (SICE) Vijayawada
Vijaya Ph: 9030924949
( 87 )
________________________________________________________
Mathematics for Competitive Exams

9 TIME, SPEED AND DISTANCE

TIME, SPEED AND DISTANCE


The basic concept of speed, time and distance is the relation between these three variables.
The speed of a body is the distance covered by the body per unit time, that is,
Distance
Speed
Time
d
s or
t
s×t=d
SMART TIP

You can remember the basic formula for speed, distance and time as the
acronym STD, where S × T = D

Please note that Speed = Distance/Time is the most important formula as all other formulae
that we will present in the upcoming text have been directly or indirectly derived from this
formula only.

Questions related to time, speed and distance include various categories such as straight
line, relative motion, circular motion, trains, boats, clocks, races, etc.
The following are the definitions and units of these three variables:
Speed: It refers to the rate at which a particular distance is covered by an object in motion.
The unit of speed may be m/s, km/h, m/min, km/min or km/day.
Distance: It refers to the extent of space between two points. The unit of distance may be
metres (m), kilometres (km), centimetres (cm), inches (in), etc.
Time: It refers to an interval separating two events. The unit of time may be seconds (sec or
s), minutes (min), hours (h), days, etc.

________________________________________________________
Subhash Institute of Competitive Exams (SICE) Vijayawada
Vijaya Ph: 9030924949
( 88 )
________________________________________________________
Mathematics for Competitive Exams

Concept of Motion It means that time remaining constant, if we have


It is said that an object attains motion or movement two vehicles moving two distances d1 and d2 at two
when it changes its position with respect to any external different speeds s1 and s2 respectively, then
stationary point. Speed, distance and time are the three
s1 d1
variables that represent the mathematical model of
s2 d2
motion as:
Let’s understand this by an example.
s×t=d
SMART TIP
Illustration: There are two vehicles: a car and a
sports bike. The speed of the car is 80 km/h and
Remember to check whether the units
that of is 100 km/h. Both the car and the sports bike
given to you are consistent—either
move for 3 hours and then halt. Find the ratio of
km/h (kilometres and hours) or m/s
distances covered by both.
(metres and seconds). Do not mix up
the two groups. Solution: Since the time taken by both the vehicles
In any problem, one has to keep a is the same,
track of the units of speed, distance sc dc
and time. The widely used units of
sb db
speed include km/h and m/s. If
required, one can also 80 dc
convert from one unit into another. 100 db
You can convert km/h into m/s by
multiplying km/h with 5/18. Similarly, dc 4
you can convert m/s into km/h by db 5
multiplying m/s with 18/5.
3. s 1/t, speed is inversely proportional to time.
It means that distance remaining constant, if we
Please note the following relations:
have two vehicles moving at two different speeds s 1
1. t d, time is directly proportional to distance. and s2 and taking times t1 and t2 respectively, then
It means that speed remaining constant, if we have s1 t2
two vehicles moving two distances d1 and d2 for two s2 t1
different time durations t1 and t2 respectively, then
Let’s understand this by an example.
t1 d1
Illustration: There are two vehicles: a car and a
t2 d2
sports bike. The speed of the car is 100 km/h and
Let’s understand this by an example. that of the sports bike is 120 km/h. Both car and
Illustration: There are two vehicles: a car and a sports bike travel 50 km and then halt. Find the ratio
sports bike. The speed of both is 80 km/h. The car of the time taken by the car and the sports bike.
moves for 5 hours, while the sports bike moves for Solution: Since the distance travelled by both the
only 3 hours. Find the ratio of distance covered by vehicles is the same,
the sports bike and the car.
sc tb
Solution: Since the speed of both the vehicles is the sb tc
same,
100 tb
t1 d1
120 tc
t2 d2
db 3 tb 5
dc 5 tc 6

2. s d, speed is directly proportional to distance. tc 6


tb 5

________________________________________________________
556
Subhash Institute of Competitive Exams (SICE) Vijayawada
Vijaya Ph: 9030924949
________________________________________________________
Mathematics for Competitive Exams ( 89 )
Section III – Chapter 9: Time, Speed and Distance

SMART TIP In case a body travels half of the given distance at


speed S1 and the other half distance at speed S2,
Please note that in the problems
2S1 S2
given above, the ratio between the then the average speed of the body is,
S1 S2
speed of the car and the bike has
been asked. You should very carefully In case of train-related problems, the time taken by
check the question in the exam and a train to pass a stationary object is given by
see whether the required (asked) ratio Length of train
is A to B or B to A because, more Time =
Speed of train
often than not, the options include a
wrong answer that is the reciprocal of In case of boat-related problems, if the speed of a
the correct answer. boat in still water is Sb km/h and the speed of the
river is Sr km/h, then
Important Concepts and Formulae:
Speed of boat downstream (Sd) = Sb + Sr km/h
The fundamental concepts of time, speed and
distance are as follows: Speed of boat upstream (Su) = Sb – Sr km/h

Average speed: If the speed of a boat upstream is Su km/h and


Total distance travelled downstream is Sd km/h, then speed of boat in still
Average speed =
Total time taken Su Sd
water = km/h
Relative speed: It is defined as the speed of a 2
moving body with respect to another body (which Sd – Su
Speed of stream = km/h
might be in motion or not). The two possible cases of 2
relative motion are: SMART TIP
a. Same direction: If two bodies are moving in the In case a body travels one-third of a
same direction, the relative speed is the distance at speed S1, the next one-
difference between their speeds and is always third at speed S2 and the last one-
third of the distance at speed S3, then
expressed as a positive value. Therefore, if two
the average speed of the body is
bodies are moving at speeds S1 and S2, then calculated as:
Relative speed = S1 – S2 (if S1 > S2) or 2S1S2S3/ (S1 S2 + S2 S3 + S1 S3)
Relative speed = S2 – S1 (if S2 > S1) You may refer to this formula.
However, questions based on it are
Or, we can simply write it as: not usually asked in the exam.
Relative speed = |S1 – S2 |
The use of modulus sign indicates that the
relative speed will always be positive,
APPLICATIONS OF TIME, SPEED AND
irrespective of the values of S1 and S2.
DISTANCE
There are four specific cases of time, speed and distance
b. Opposite direction: If two bodies are moving in
that usually come in the exam. These include:
the opposite direction the relative speed is the
sum of their speeds. Therefore, if two bodies are Trains
moving in the opposite direction at speeds S1 Please note that, in the case of train problems, the
and S2, then distance to be covered when crossing an object is
equal to
Relative speed = (S1 + S2)
Distance to be covered = Length of train + Length of
In case a body travels half of the given time at speed object
S1 and the other half time at speed S2, then the
average speed of the body is, Remember that, in case the object under consideration
is a pole or a person or a point, we can consider them to
S1 + S2
SS be point objects with zero length. It means that we will
2

________________________________________________________
557
Subhash Institute of Competitive Exams (SICE) Vijayawada
Vijaya Ph: 9030924949
________________________________________________________
Mathematics for Competitive Exams ( 90 )

not consider the lengths of these objects. However, if the L


object under consideration is a platform (non-point Time taken to meet for the first time =
V1 V2
object), then its length will be added to the formula of
Time taken to meet for the first time at the starting point
the distance to be covered.
L L
Now, assume that we have the following information: = LCM of ,
V1 V2
ST = Speed of train In case these two persons are running in opposite
directions then
So = Speed of object
L
LT = Length of train Time taken to meet for the first time =
V1 V2
Lo = Length of object Time taken to meet for the first time at the starting point
t = Time taken L L
= LCM of ,
V1 V2
Then, the following cases will hold true:
If the train crosses a stationary point object, Boats and Streams
We have already introduced the formulae related to
LT
ST boats and streams in the above section.
t
Illustration: Assume that the speed of a boat upstream
If the train crosses a stationary object of length Lo, is 6 km/h and downstream is 12 km/h. Calculate the
LT LO speed of boat in still water.
ST
t Su Sd
Solution: Speed of boat in still water = km/h
If the train crosses a moving point object having speed 2
So going in the opposite direction, 12 6
Sb = 9 km/h
2
LT SMART TIP
t
ST SO
There may be questions that
If the train crosses a moving object of length L o going in introduce the concept of two or more
the opposite direction, moving objects around a circular
track either in the same direction or
LT Lo in the opposite direction. As, in the
t=
ST So case of linear relative motion, here
also, the relative speed of two objects
If the train crosses a moving object of length Lo going in
moving in the same circular direction
the same direction, is given by
LT Lo S1 – S2. Similarly, the relative speed of
t= two objects moving in the opposite
S T – So
circular direction is given by S1 + S2.
Illustration: A train of length 300 metres crosses a
person (stationary) standing on a platform in 10
seconds. What is the speed of the train? Clocks
A clock is usually represented by a circular dial that is
Solution: Here, we are given t = 10 seconds divided into 12 equal parts called hour spaces. Further,
LT = 300 m each hour space is divided into 5 equal parts, each
representing one minute space. Therefore, we can say
18 that the total circumference of the clock is divided into
ST = 300/10 = 30 m/s or 30 108 km/h
5 60 (12 5) minute spaces. The clock has three hands,
namely the hour hand, the minute hand and the seconds
Circular Motion hand. Of the three, the hour hand is the smallest. The
If two persons, P1 and P2, are running in a circular track hour and minute hands meet each other once in every
of length L in same direction at a speed of V1 and V2
single hour, i.e. 24 times in a day.
respectively, and V1 > V2 then

________________________________________________________
558
Subhash Institute of Competitive Exams (SICE) Vijayawada
Vijaya Ph: 9030924949
________________________________________________________
Mathematics for Competitive Exams ( 91 )
Section III – Chapter 9: Time, Speed and Distance

We can consider that the minute hand travels at a speed Also remember the following results:
of 60 km/h and the hour hand travels at 5 km/h.
Number of times the hour and minute hands are in a
Alternatively, we can also consider that the relative
straight line (0° or 180°) in a day (24 hours) = 44
speed of minute hand with respect to the hour hand is times
55 km/h.
Number of times the hour and minute hands are at
the right angle to each other (0° or 180°) in a day
(24 hours) = 44 times
Both the hands of a clock meet after every 65 5/11
minutes. Therefore, if the hour and minute hands
meet each other in less than 65 5/11 minutes, the
clock is fast, and if the hour and minute hands meet
each other in more than 65 5/11 minutes, the clock
We can summarise some important results as follows: is slow.
Illustration: Akash went out of the home to take his
Angle traced by the minute hand in 1 hour = 360°.
tuition class between 8 and 9. Since there was no tuition
Angle traced by the minute hand in 60 minutes = due to some emergency, he returned home before 9,
360° and he observed that the positions of the hour hand and
Angle traced by the minute hand in 1 minute = 6° the minute hand had interchanged. Calculate the time
360 for which Akash was out.
Angle traced by the hour hand in 1 hour =
12 Solution: Note that since the minute and hour hands
= 30° had exchanged their positions, they covered 360°
30 collectively. Assuming that Akash was out for t minutes,
Angle traced by the hour hand in 1 minute =
60 0.5° t + 6° t = 360° (because the hour hand moves
1 0.5° in one minute, while the minute hand moves 6° in
=
2 SMART TIP one minute)

While the minute hand covers 6° in 360


t 55.38
one minute, the hour hand covers 6.5
only 1/2°.

________________________________________________________
559
Subhash Institute of Competitive Exams (SICE) Vijayawada
Vijaya Ph: 9030924949
( 92 )
________________________________________________________
Mathematics for Competitive Exams

PRACTICE QUESTIONS
Answer the following questions by selecting the most appropriate option.
1. P and Q are 27 km away. Two trains with speed of 9. The taxi charges in a city comprise fixed charges
24 km/hr and 18 km/hr respectively start and additional charges/km. The fixed charge is
simultaneously from P and Q and travel in the for a distance of upto 5 km and additional
same direction. They meet at a point R beyond Q. charge/km thereafter. This charge for a distance
Distance QR is of 10 km is ` 350 and for 25 km is ` 800. The
(1) 48 km (2) 36 km charge for a distance of 30 km is
(3) 126 km (4) 81 km (1) ` 900 (2) ` 950
2. A man can row at 10 kmph in still water. If it (3) ` 800 (4) ` 750
takes a total of 5 hours for him to go to a place 10. A train goes from Ballygunge to Sealdah at an
24 km away and return, then the speed of the average speed of 20 km/hour and comes back at
water current is an average speed of 30 km/hour. The average
1 speed of the train for the whole journey is
(1) kmph (2) 1 kmph
(1) 27 km/hr (2) 26 km/hr
(3) 2 kmph (4) 3 kmph (3) 25 km/hr (4) 24 km/hr
3. A man started 20 minutes late and travelling at a 11. If a boy walks from his house to school at the rate
of 4 km per hour, he reaches the school 10
speed of times of his usual speed reaches his minutes earlier than the scheduled time.
2
office in time. The time taken by the man to reach However, if he walks at the rate of 3 km per hour,
his office at his usual speed is he reaches 10 minutes late. Find the distance of
(1) 1 hour (2) 30 minutes his school from his house
(3) 40 minutes (4) 1 hr 20 minutes (1) 5 km (2) 4 km
(3) 6 km (4) 4.5 km
4. A bus left from the depot 1 hr late than the usual
time. If it covers the delay for a distance of 120 12. John travelled 240 km at a certain speed and
km by moving at 20 kmph extra speed, find the then another 192 km at 8 km/h faster. If the
usual speed of the bus. entire trip is of 10 hr, find his initial speed.
(1) 30 km/h (2) 8 km/h (1) 45 km/h (2) 60 km/h
(3) 16 km/h (4) 40 km/h (3) 40 km/h (4) 75 km/h

5. A certain distance is covered in certain time. If 13. Speed of a car is 50% more than the speed of a
the same distance is covered in thrice the time, truck. If the truck travels a certain distance in ‘T’
what is the ratio of the usual speed to new speed? hours then the time taken by car to travel half the
(1) 1 : 3 (2) 3 : 1 distance travelled by truck is
(3) 2 : 3 (4) 1 : 4 T T
(1) (2)
2 3
6. Due to delayed start, a fighter jet had to reach its
T T
destination in half the usual time. What should (3) (4)
the ratio of the usual speed to required speed? 5 4
(1) 1 : 2 (2) 3 : 1 14. Ratio among the speeds of P, Q and R in covering
(3) 2 : 3 (4) 2 : 1 a certain distance is 6:7:4. What is the ratio of
7. The ratio of speeds of two cars is 2 : 3. What will time taken by P, Q and R?
be the ratio of the time required to travel the (1) 7:4:6 (2) 12:14:21
same distance? (3) 2:4:7 (4) 14:12:21
(1) 5 : 3 (2) 2 : 3 15. Monika covers a certain distance at a speed of 48
(3) 3 : 4 (4) 3 : 2 km/h and then she travels back at a speed of 32
8. Rahul travelled 200 km in 3 hr and another 500 km/h to go to her initial position. What is the
km in 4 hr. Find his average speed of the entire average speed of Monika for the entire journey?
journey. (1) 36 km/h
(1) 150 kmph (2) 100 kmph (2) 40 km/h
(3) 200 kmph (4) 125 kmph (3) 45 km/h
(4) None of these

________________________________________________________
560
Subhash Institute of Competitive Exams (SICE) Vijayawada
Vijaya Ph: 9030924949
________________________________________________________
Mathematics for Competitive Exams ( 93 )
Section III – Chapter 9: Time, Speed and Distance

16. Two persons A and B are 200 metres apart. They 24. If a distance of 50 m is covered in 1 min, of 90 m
start moving towards each other at speeds of 20 in 2 min and of 130 m in 3 min, find the distance
m/s and 15 m/s respectively. What is the covered in 15 min.
distance between A and B if they stop 2 seconds (1) 610 m (2) 750 m
before they can cross each other? (3) 1000 m (4) 650 m
(1) 50 m (2) 60 m
25. Three men step off together from the same spot.
(3) 70 m (4) 80 m
Their steps measure 63 cm, 70 cm and 77 cm,
17. Two trains are running at 40 km/hr and 20 respectively. The minimum distance each should
km/hr, respectively in the same direction. The cover so that all can cover the distance in
fast train completely passes a man sitting in the complete steps is
slow train in 5 seconds. The length of the fast (1) 9630 cm (2) 9360 cm
train is (3) 6930 cm (4) 6950 cm
2
(1) 23 m (2) 27 m 26. A train moves for 400 km. The train could have
9 saved 2 hours if it had moved at an extra 10
7 kmph. Find the original speed of the train.
(3) 27 m (4) 23 m
9 (1) 15 kmph (2) 20 kmph
18. If a clock strikes appropriate number of times at (3) 25 kmph (4) 40 kmph
each hour, how many times will it strike in a day? 27. A boat takes 8 hr more when sailing upstream
(1) 300 (2) 156 than when downstream. The speed of the boat in
(3) 68 (4) 78 still water is 6 km/h and the distance travelled is
19. A train of length 300 m is moving at a speed of 45 36 km. Find the total time of journey if the boat
km/h. It crosses a platform of length l in 30 goes downstream and comes back.
seconds. The value of l (in metres) is (1) 15 hours
(1) 50 m (2) 75 m (2) 16 hours
(3) 100 m (4) 125 m (3) 20 hours
(4) 25 hours
20. A bus and a car are both travelling in the same
direction. At time t hours, the bus is ahead of the 28. Rohit sails 72 km downstream in 3 hr and 60 km
car by 30 km. The speed of bus is 60 km/h. At upstream in 6 hr. Find the speed of the current
time t+2 hours, it is observed that the car is and the still water speed of the boat.
ahead of the bus by 100 km. What is the speed of (1) 7 km/h and 17 km/h
car? (2) 8 km/h and 15 km/h
(1) 100 km/h (2) 115 km/h (3) 10 km/h and 20 km/h
(3) 120 km/h (4) 125 km/h (4) 9 km/h and 16 km/h

21. A boat sails at a speed of 20 m/s in still water. 29. Neha rides her powerboat 360 km upstream and
When sailing upstream, the boat takes twice the then comes back. She took 5 hr extra in the
time taken during sailing downstream. What is upstream journey compared to the return journey.
the speed of the current? If the speed of current is 6 kmph, calculate the
(1) 3.34 m/s (2) 2.25 m/s speed of boat in still water.
(3) 6.67 m/s (4) 4.56 m/s (1) 15 kmph (2) 30 kmph
(3) 25 kmph (4) 35 kmph
22. A and B start from the same point but in opposite
direction along a circular path of radius 14 m with 30. A boat with the speed of 8 km/h in still water
speeds 11 m/s and 22 m/s, respectively. After covers a distance of 60 km downstream in 6 hr.
what time, will they meet again at the starting Calculate the time required for the return journey.
point? (1) 10 hr (2) 12 hr
(1) 6 seconds (2) 8 seconds (3) 15 hr (4) 8 hr
(3) 12 seconds (4) 20 seconds 31. A 280 m long train crosses a platform thrice its
23. Two trains of equal length are running on parallel length in 6 min 40 s. What is the speed of the
lines in the same direction at the rate of 46 train?
km/hr and 36 km/hr respectively. The faster train (1) 3.2 m/s
passes the slower train in 36 seconds. The length (2) 1.4 m/s
of each train is (3) 2.8 m/s
(1) 50 m (2) 72 m (4) Cannot be determined
(3) 80 m (4) 82 m

________________________________________________________
561
Subhash Institute of Competitive Exams (SICE) Vijayawada
Vijaya Ph: 9030924949
________________________________________________________
Mathematics for Competitive Exams ( 94 )

32. A car covers four successive 7-km distances at 36. A man starts from a place P and reaches the
speeds of 10 km/h, 20 km/h, 30 km/h, and place Q in 7 hours. He travels 1/4th of the
60 km/h, respectively. Its average speed over this distance at 10 km/h and the remaining distance
distance is at 12 km/h. The distance, in kilometres, between
(1) 60 km/h (2) 20 km/h P and Q is
(3) 40 km/h (4) 30 km/h (1) 72 (2) 90
(3) 70 (4) 80
33. Two places P and Q are 162 km apart. A train
leaves P for Q and simultaneously another train 37. Raj goes to his relatives’ place at a distance of
leaves Q for P. They meet at the end of 6 hours. If 200 km with his bike in 8 hr. He took 10 hr while
the former train travels 8 km/h faster than the coming back. What is the ratio of the average
other, then the speed of train from Q is speed of Raj during the two journeys?
5 1 (1) 3 : 2 (2) 2 : 3
(1) 12 km / h (2) 8 km / h
6 2 (3) 3 : 4 (4) 5 : 4
5 1 38. A train travels a distance of 200 km at a constant
(3) 10 km / h (4) 9 km / h
6 2 speed. If the speed is increased by 5 km/h, the
34. A boat moves downstream at the rate of 1 km in travel time is reduced by 2 hr. What is the original
1 speed of the train?
7 minutes and upstream at the rate of 5 km per (1) 15 km/h (2) 10 km/h
2
hour. What is the speed (in km/hour) of the boat (3) 20 km/h (4) 25 km/h
in still water? 39. The distance between Pune and Mumbai is 180
1 km. Bus A starts from Mumbai towards Pune at
(1) 6 (2) 8
2 40 kmph and Bus B starts from Pune towards
1 Mumbai at 50 kmph. They will meet after:
(3) 4 (4) 3 (1) 1 hr (2) 5 hr
2
(3) 3 hr (4) 2 hr
35. The diameter of each wheel of a car is 70 cm. If
each wheel rotates 400 times per minute, then 40. A train passes a man standing at a 120 m long
platform in 5s. The train takes 10s to completely
22
the speed of the car (in km/h) is Take pass the platform. Find the length and speed of
7
the train.
(1) 52.8 (2) 5.28 (1) 140 m, 21 m/s (2) 150 m, 25 m/s
(3) 528 (4) 0.528 (3) 110 m, 20 m/s (4) 120 m, 24 m/s

________________________________________________________
562
Subhash Institute of Competitive Exams (SICE) Vijayawada
Vijaya Ph: 9030924949
________________________________________________________
Mathematics for Competitive Exams ( 95 )
Section III – Chapter 9: Time, Speed and Distance

ANSWERS AND EXPLANATIONS


1. (4) Let us assume that the 120 Accordingly, x + 10y = 350
Usual time = hr
trains meet after t hours x equation 1
Relative speed of the trains = 120 x + 25y = 800 equation 2
New time = hr
(24 -18) = 6 km/h
Subtracting equation 1 from
120 120
Distance = 27 =1 equation 2, we get: 15y = 450
27 9 1 1 1 or y = 30
t= h
6 2 x x 20 120 Putting the value of y in
QR distance travelled by train x2 + 20x – 2400 = 0 equation 1, we get:
which is travelling at a speed of x + 300 = 350
18 km/h = 18t = 18 × 9/2 = 81 x = 40 or –60
km or x= 50.
Disregarding the negative value,
2. (3) Let the speed of the water the usual speed of the bus is 40 Therefore, the total charge for a
current be y km/hr. km/h distance of 30 km will be 50 +
(30 × 30) = ` 950.
Upstream speed = (10–y) km/hr 5. (1) Since the time is tripled, the
speed must be one-third. 10. (4) The average speed of the
Downstream speed = (10+y) train for the whole journey =
km/hr The required ratio = 1:3
2xy
24 24 6. (4) Since the time is halved, the x y
Total time = speed must be doubled.
10 y 10 y
2 20 30
The required ratio of speed = =
480 20 30
5 2:1
100 y 2 = 24 km/h
7. (4) Let the speeds of two cars
480 = 500 – 5y2 be 2x km/h and 3x km/h and 11. (2) Let the distance, covered by
the distance be y km him be x km
5y2 = 20
y
Difference of time = 10 min +
y = 2 km /hr Time required = hr and 10 min = 20 min
2x
3. (1) Suppose the usual speed of y 20
hr = h
a man is v and time is t h. 3x 60
By the formula, v1t1 = v2t2 The required ratio of time = According to the question,
3 20 y y
vt = t : =3:2 x x 20
2v 60 2x 3x
3 4 60

3 1
8. (2) Total distance covered by
x 20
t= t Rahul = 200 km + 500 km =
2 3 12 60
700 km
3 1 x=4
t–t= Total time by Rahul = 3 hr + 4
2 2
hr = 7 hr 12. (3) Let the initial speed of John
t=1 be x km/h
700
Average speed = kmph
Hence, the time taken by the 7 According to the question,
man to reach his office at his = 100 kmph
240 192
usual speed is 1 hour.
9. (2) Let us assume that the fixed x x
4. (4) Let the usual speed of the charge for the first 5 km is x and
240(x + 8) + 192x = 10(x² +
bus be x km/h thereafter, y is charged from
8x)
every km.

________________________________________________________
563
Subhash Institute of Competitive Exams (SICE) Vijayawada
Vijaya Ph: 9030924949
________________________________________________________
Mathematics for Competitive Exams ( 96 )

10x² – 352x –1920 = 0 250


Speed of car =
Speed time 2
x = 40, –24/5
= 125 km/h
50
By ignoring the negative value, = ×5 m = 250/9 m
9 21. (3) Let the speed of the current
the required speed of John = 40
7 is x m/s and the distance be y
km/h = 27 m
9 m
3
13. (2) Speed of car is of the 18. (2) Suppose it is a 12-hour According to the question,
2
speed of truck therefore time clock, it will strike 1 time at 1, 2
times at 2, 3 times at 3, and so y 2y
taken by car to cover the same
on up to 12 times at 12. x x
2
distance will be of the time 20 + x = 40 – 2x
3 This makes 1 + 2 + 3 + 4 + 5 +
taken by the truck. 12 = 78 times in every 12
20
hours. x= = 6.67
Time taken by car to cover half 3
1 2 T Since, a clock takes 2 rounds in
the distance = T = a day, so it will strike 78+78 = 22. (2) Let A and B meet after x
2 3 3
156 times a day. seconds
hours
19. (2) LTrain = 300m, LPlatform = l, They will meet again at the
14. (4) For constant distance, speed
STrain = 45 km/h, t = 30 s (given) starting point when both have
is inversely proportional to time.
covered complete rotation in
Ratio of time taken = (300 l)
30 seconds same time.
1 1 1 5
: : = 14:12:21 45
6 7 4 18 A completes one rotation in
14
15. (4) Let the distance covered 12.5 30 = 300 + l every 8 seconds 2
11
from one side = d l = 75 m
Total distance = 2d B completes one rotation in
20. (4) Initial positions of car and
d d 14
Total time taken = bus at time t: every 4 seconds 2
48 32 22
5d They will meet after 8 sec
=
96 since in 8 sec, A will complete
2d one rotation and B will complete
Average speed = 5d two rotations.
Positions of car and bus at time
96 t+2 hours: 23. (1)Difference between speed of
= 38.4 km/h two trains = 46 – 36 = 10 km/h
16. (3) The distance between A and 36
B when they cross each other Time taken = 36 sec
60 60
will be 0. 1
Hour = Hour
Therefore, they will be Let’s combine the pictorial 100
separated by a distance that representations of the above
Sum of length of two trains = 10
they will cover together in 2 two cases:
1 1
seconds. × km km
100 10
Distance covered in 2 seconds
Length of each train
= 20 × 2 + 15 × 2 = 70 m
1 1000
17. (3) Relative speed of the trains = km 50 m
20 20
It is clear from the combined
5
= (40 20) km/h = 20 × representation that the car 24. (4)
18
travelled a distance of (30 +
m/sec First 50 m in 1 minute and
120 +100) = 250 km in two
50 subsequent 40 m in 1 minute
= m/sec hours
9 each.

________________________________________________________
564
Subhash Institute of Competitive Exams (SICE) Vijayawada
Vijaya Ph: 9030924949
________________________________________________________
Mathematics for Competitive Exams ( 97 )
Section III – Chapter 9: Time, Speed and Distance

So, x = 3 or –12 Speed of current = (10 – 8)


km/h = 2 km/h
For 15 minutes Disregarding the negative value,
the required speed of the So, upstream speed = (8 – 2)
50 + 15(40) = 50 + 600
current = 3 km/h km/h = 6 km/h
= 650 m
Total time of the journey = Time required while returning
25. (3)
36 36 60
To find the minimum distance hr = 16 hr = hr = 10 hr
3 9 6
here, we need to find the lowest 72
common multiple (LCM) of 63, 28. (1) Speed of downstream = 31. (3) We use the following formula
3
70 and 77. for non-point stationary objects:
km/h = 24 km/h
So, LCM of 63, 70 and 77 = 60 LT Lo
Speed of upstream = km/h ST =
6930 6 t
= 10 km/h
26. (4) Let the original speed of the (280 (3% 280)) m
train be x kmph Let the speed of current be x ST = s
400
km/h and the speed of boat in [6 min 40 s = 400 s]
400 still water is y km/h
Original time = hr
x ST = 2.8 m/s
According to the question,
New speed of train = (x + 10) 32. (2) Total distance = 7 × 4 = 28
kmph and new time required = x + y = 24 km
400 y – x = 10
hr 7 7 7 7
x Total time = + + +
y = 17 km/h and x = 7 km/h 10 20 30 60
According to the question,
29. (2) Let the time for downstream = 1.4 h
400 400 journey be x hr and the speed of
x x boat in still water be y kmph 28
Average speed = = 20
1.4
x2 + 10x – 2000 = 0 For upstream journey, time
km/h
x = 40 and –50 taken = (x + 5) hr and speed =
360 33. (4) Let the speed of train from Q
Disregarding the negative value, kmph to P = X km/h
x
required original speed of train
= 40 kmph For downstream journey, time Speed of train from P to Q = X
360
+ 8 km/h
27. (2) Let the speed of current be x taken = x hr and speed =
x According to the question,
km/h
kmph
Upstream speed = (6 – x) 162
According to the question, =6
km/h X X 8
360 360
and downstream speed = (6 + x) =2×6 12X + 48 = 162
km/h 1
x2 + 5x – 150 = 0 X= 9
36 2
Time for upstream sail = x = 10 (taking positive root)
x Speed of train from Q
hr and for downstream sail 360 1
36 So, downstream speed = = 9 km/h
= hr 10 2
kmph = 36 kmph
34. (1) Downstream speed of boat
According to the question, So, Speed of boat in still water = 60
= × 1 = 8 km/h
36 36 (36 – 6) kmph = 30 kmph 7.5
=8 60
30. (1) Downstream speed = Upstream speed of boat = 5
6
x2 + 9x – 36 = 0 km/h
km/h = 10 km/h

________________________________________________________
565
Subhash Institute of Competitive Exams (SICE) Vijayawada
Vijaya Ph: 9030924949
( 98 )
________________________________________________________
Mathematics for Competitive Exams

Speed of boat in still water Average speed of Raj during According to the question,
8 5 1 200
= = 6 km/h return journey = km/h = 40x + 50x = 180
2 2 10
20 km/h   90x = 180
35. (1) Distance covered by wheel in
22 The required ratio of his x=2
one rotation = 2 × × 35
7 speeds = 25 : 20 = 5 : 4
The buses will meet after 2 hr.
= 220 cm 38. (3) Let the original speed be x
40. (4) Let the length of the train be
km/h
Distance covered by car in one x meters and the speed be y
minute = 400 × 220 = 88,000 200 m/s
Original time = hr
cm = 0.88 km x
x
=5
Distance covered by car in one 200 y
New time = hr
hour = 60 × 0.88 = 52.8 km x
x = 5y
Speed of car = 52.8 km/h According to the question,
Again length of train and
36. (4) Let the total distance = 4X 200 200 platform together = (x + 120) m
=2
km x x
= 10
X 3X 1 1 1 y
Then, =7
10 12 x x
x + 120 = 10y
6X 15 X
=7 x2 + 5x – 500 = 0
60 60 5y + 120 = 10y
x = 20 or –25
X = 20 120
Disregarding the negative value, y= = 24
5
Total distance = 20 × 4 = 80 the original speed = 20 km/h
km Length of the train = 5 × 24 m
39. (4) Let the buses meet after x hr = 120 m and speed = 24 m/s
37. (4) Average speed of Raj during
Distance travelled by bus A in
200
first journey = km/h = 25 x hours = 40x km and by bus B
8
= 50x km
km/h

________________________________________________________
566
Subhash Institute of Competitive Exams (SICE) Vijayawada
Vijaya Ph: 9030924949
________________________________________________________
Mathematics for Competitive Exams ( 99 )

10 ALGEBRA

BASIC ALGEBRA
Algebra is just a higher version of Arithmetic. In algebra, we have an equation with an equality
sign, which means that both sides are equal. We need to perform some arithmetical
operations to obtain the value for the variable or the solution of the equation. Let us take a
simple example:
The expression, 8 + x = 3 × 6, means that there is a variable x which needs to be replaced by
some number which when added to 8 will give the same result as the product of 3 and 6.
x=3×6–8
= 18 – 8 = 10
Note: In an equation, values on the Right Hand Side (RHS) and Left Hand Side (LHS) are
equal for very few values of the variable. Those values are known as the solution of the
equation. In the above example, 10 is the solution of the equation.

Application
For finding the exact value of some unknown terms with the help of some known information.

Some Important Examples:


1. Two numbers are such that their sum is 85 and one of the numbers is 5 more than the
other number. What are the numbers?
Solution:
Let us assume that one of the numbers is x. Other number will be x + 5. Now,
x + x + 5 = 85
2x = 80
x = 40
So, the two numbers are 40 and 45.
2. Ages of Ram and his father are in the ratio of 3 : 5. If the sum of their age three years
ago was 74, then what is the current age of Ram’s father?
Solution:
Let the age of Ram and his father be 3x and 5x respectively. Now,

________________________________________________________
Subhash Institute of Competitive Exams (SICE) Vijayawada
Vijaya Ph: 9030924949
________________________________________________________
Mathematics for Competitive Exams ( 100 )

(3x – 3) + (5x – 3) = 74 2 xy y y 1
3x + 5x = 74 + 6 = 80 x2 y 2 y 2
y2
8x = 80 a b c
8. If = 3, then what will be the
X = 10 1 a 1 b 1 c
2 2 2
So, the current age of Ram’s father is 5 × 10 value of ?
1 a 1 b 1 c
= 50 years
Solution:
3. The length of a rectangle is twice its width. If the
area of the rectangle is 450 m2, then what is the a b c
=3+3
length and width of the rectangle? 1 a 1 b 1 c

Solution: 1 1 1
=6
1 a 1 b 1 c
Let the width of the rectangle be x m. The length of
the rectangle will be 2x m. Now, 2 2 2
= 12
Area, x × 2x = 450 1 a 1 b 1 c

2x2 = 450 9. If x
;x=?
x2 = 225
Solution:
x = 15
x
So, the width of the rectangle is 15 m and its length
is 30 m. 82
8 x
4. The length of a wire needed for fencing a 8
52

rectangular field is 96 m. What is area of the


32
rectangular field, if its length is twice its width? 8 8x
Solution: 3
x=
Let width of the field be x m. So, length is (2x) m. 2
Perimeter of the field is,
x2 1 2x 4
10. If ; then, what is the value of
2 (x + 2x) = 96, 6x = 96, x = 16 m x2 1 2x 5
Now, area of the field is x × 2x = 2x2 = 2 × 162 1
x ?
= 512 m2. x
5. If x * y = 3x – 4y + 2xy; then 4*5 is Solution:
Solution: x2 1 2x 4
4 * 5 = 3 (4) – 4 (5) + 2 (4) (5) = 12 – 20 + 40 = 32 x2 1 2x 5

a 1
6. If a ÷ b = a – b + ; then 8 ÷ 2 =? x x 4
b x
1 5
x x
Solution: x
8 1
8÷2=8–2+ = 6 + 4 = 10 Let x y
2 x
7. Two numbers x and y are such that their sum is y
equal to twice their difference. What will be the y
2xy
value of ? 5y – 10 = 4y + 8
x2 y2
y = 18
Solution:
1
x + y = 2 (x - y) x
x
x = 3y

________________________________________________________
568
Subhash Institute of Competitive Exams (SICE) Vijayawada
Vijaya Ph: 9030924949
________________________________________________________
Mathematics for Competitive Exams ( 101 )
Section III – Chapter 10: Algebra

POLYNOMIALS Simple-Intercept form


The general form of equation of the intercept form is
Polynomial consists of two words, ‘poly’, which means
written as (x/a) + (y/b) = 1; where (a, 0) is x-intercept of
‘many’ and ‘nomial’, which means ‘term’. So, polynomial
line and (0, b) is y-intercept of the line.
means ‘many terms’. It is an expression made by
combining constants and variables such that the powers Slope-Intercept form
of the variables always remain positive. It consist of Slope-Intercept form of linear equation is written as y =
variables (e.g. x, y, z, a, b, etc.), constants (e.g. 10, 2, ¾,
mx + c, where m is slope of the equation and c is the y-
etc.), and exponents (power that the variables bear).
intercept of the line. For example, y = 3x + 4. The graph
General form of a polynomial is p x a0 a1 x of this equation is shown below:

a2 x2 ........ an xn ; where x is a variable, a0, a1, a2,…., an


are constants, and n is a non-negative integer.

Degree of Polynomial
Degree of polynomial is defined as the highest degree
that any variable in the polynomial (in canonical form
consisting linear combination of different monomials)
carries. In polynomial x2 +2x + 1, variable x has a degree
of 2, in y – y3 + 2, variable y has a degree of 3 and so on.

Equation
An equation is formed when either two polynomials or On comparing with the simple form of linear equation (ax
one polynomial and a constant are equated. a
+ by + c = 0), we get, slope of a linear equation m .
b
Linear Equation
An algebraic equation with 1 as degree of polynomial is Some Important Examples:
y 1. What is the slope of line 3x + 4y = 7?
known as a linear equation. It means = constant. It
x
Solution: Rewrite the given expression in slope-
may consist of one or more variables. For example, 2x +
intercept form.
3 = 0 (one variable linear equation), y = 5/4 (one
variable linear equation), 3x + 4y = 5 (two variable linear 4y = – 3x + 7
equation), etc. are linear equations. 3 7
Y= – x
4 4
Simple form of linear equation
The simple form of linear equation is written as ax + by + Compare the above expression with generalized
form of slope-intercept equation that is y = mx + c.
c = 0, where a, b and c are constants and x and y are
So, the slope is,
variables. For example, 3x + 4y = 12 is a simple form of
a polynomial. 3
m= –
4
The graph of the equation is shown below:
2. What will be the x and y axes intercept of the line
5x + 7y = 70?
Solution: Rewrite the given equation in intercept
form.
5x 7y
1
70 70

x y
1
14 10

________________________________________________________
569
Subhash Institute of Competitive Exams (SICE) Vijayawada
Vijaya Ph: 9030924949
( 102 )
________________________________________________________
Mathematics for Competitive Exams

On comparing with general form that is (x/a) + (y/b) 3. Example: What will be the slope of the line passing
= 1, we get (14, 0) as x-intercept and (0, 10) as y- through (2, 7) and (5, 13)?
intercept. Solution: Here, x1 = 2, x2 = 5, y1 = 7 and y2 = 13.
Now, the slope is,
Two Points Form y2 y1
Point-Slope form of linear equation is written as m
x2 x1
y2 y1
y y1 x x1 where, (x1, y1) and (x2, y2) are 13 7
x2 x1 m
5 2
points through which line passes.
6
m 2
Point-Slope form 3

Point-Slope form of linear equation is written as Notes:


Two lines a1x + b1 y + c1 = 0 and a2x + b2 y + c2 = 0
y y1 m x x1 where, (x1, y1) and (x2, y2) are points
with slopes m1 and m2 respectively are parallel if m1
through which line passes and m is slope of the line. The = m 2.
graph of the equation is shown below: Two lines a1x + b1 y + c1 = 0 and a2x + b2 y + c2 = 0
with slopes m1 and m2 respectively are
perpendicular if m1 × m2 = –1
Two lines a1x + b1 y + c1 = 0 and a2x + b2 y + c2 = 0
will have infinite solutions (coincident lines) if
a1 b1 c1
.
a2 b2 c2
Two lines a1x + b1 y + c1 = 0 and a2x + b2 y + c2 = 0
will have no solution (inconsistent) if a1 b1 c1
.
a2 b2 c2
Two lines a1x + b1 y + c1 = 0 and a2x + b2 y + c2 = 0
will have exactly one solution if a1 b1
.
a2 b2
4. Example: How many solutions are possible for lines
with equation 3x + 4y + 6 = 0 and 12y + 9x + 30 = 0?
Some Important Examples:
1. Example: Write down the expression of line passing Solution: Here, a1 = 3, b1 = 4, c1 = 6, a2 = 9, b2 = 12
through points (1, 2) and (2, 5). and c2 = 30. So,

Solution: Here, x1 = 1, x2 = 2, y1 = 2 and y2 = 5. Now a1 b1 1 c1 1


a2 b2 3 c2 5
the expression for line is,
y2 y1 So, there is no solution as both are parallel lines.
y y1 x x1
x2 x1 5. Example: What will be the equation of line
5 2 1
y 2 x 1 3 x 1 perpendicular to y = x + 6 and passing through
2 1 4

y – 3x + 1 = 0 (1, 1)?

2. Example: Write down the expression of line passing Solution: Since the lines are perpendicular, slope of
through points (5, 2) and having a slope of 3. the line will be,
( 1)
Solution: Here, x1 = 5, y1 = 2 and m = 3. Now the m= =4
1
expression for line is,
4
y y1 m x x1 So the equation will be,
y 2 3 x 5 y 1 4 x 1

y – 3x + 13 = 0 y – 4x + 3 = 0

________________________________________________________
570
Subhash Institute of Competitive Exams (SICE) Vijayawada
Vijaya Ph: 9030924949
________________________________________________________
Mathematics for Competitive Exams ( 103 )
Section III – Chapter 10: Algebra

6. Example: What will be the equation of line parallel to Formulae


y = 5x + 2 and passing through (1, 1)? Certain formulae are used while factorising polynomials.
Solution: Since the lines are parallel, slope of the They are:
line will be,
Multiplying powers if bases are same is: x a x b x
a b
.
m=5
b
So the equation will be, Power to a power: x a xa b x ab .
y 1 5 x 1 Product property for power: xy k
xk yk .
y – 5x + 4 = 0 k 1
Negative power: x .
xk
Quadratic Equation
An algebraic equation with 2 as degree of polynomial is Roots of quadratic equation ax 2 bx c 0
2
y b b2 4ac
known as a quadratic equation. It means constant . are .
x 2 2a
For example, 2x2+ 3 = 0, y2 = 5/4, 3x2 + 4y2 = 5xy, etc. Sum and product of roots of quadratic equation
are quadratic equations. b c
ax 2 bx c 0 are and respectively.
a a
Factorisation of Polynomial Algebraic identities
The process of writing one polynomial as a product of
Common algebraic identities are as follows:
two or more polynomials is known as factorisation. For
example, x2 -5x + 6 = (x – 2) (x – 3). (a + b)2 = a2 + 2ab + b2 = (a – b)2 + 4ab
(a – b)2 = a2 – 2ab + b2 = (a + b)2 – 4ab
Remainder Theorem a2 – b2 = (a + b)(a – b)
According to this theorem, when an expression f(x) is
a2 + b2 = [(a + b)2 + (a – b)2]/ 2 = (a + b)2 – 2ab
divided by (x – a), the remainder is f(a).
(a + b)2 – (a – b)2 = 4ab
Example: What will be the remainder when x2 + 3x – 2 is
(a + b)3 = a3 + b3 + 3ab (a + b)
divided by (x – 1)?
(a – b)3 = a3 – b3 – 3ab (a – b)
Solution: Here, f(x) = x2
+ 3x – 2 is divided by (x – 1), a3 + b3 = (a + b) (a2 – ab + b2)
then the remainder will be f(1). So,
a3 – b3 = (a – b) (a2 + ab + b2)
Remainder is f(1) = 1 + 3 – 2 = 4 – 2 = 2. (a + b)2 – (a – b)2 = 4ab
(a + b + c)2 = a2 + b2 + c2 + 2(ab + bc + ac)
Factor Theorem
This theorem is an extension of the remainder theorem. Let’s take an example:
It is used to check whether an expression is a factor of Simplify the following expression:
another expression or not. According to this theorem, “If
(3.28 – 1.72)2 + 4 × 3.28 × 1.72 =?
(x – a) is a factor of f(x), then f (a) must be zero”.
Solution: From the above statement, it can be identified
Illustration: What are the factors of x2 – 3x + 2? that the statement is based on algebraic identity, which
is in the form of (a – b)2 + 4ab that can also be written
Solution: To find out factors, put f(x) = 0. So, as (a + b)2
x2 – 3x + 2 = 0 Using the above identity, we get
x2 – 2x – x + 2 =0 a = 3.28; b = 1.72
x(x – 2) – 1(x – 2) = 0 Therefore, (3.28 + 1.72)2 = (5)2 = 25
(x – 1)(x – 2) = 0
Since, f(1) = f(2) = 0, therefore (x – 1) and (x – 2) are
factors of x2 – 3x + 2.

________________________________________________________
571
Subhash Institute of Competitive Exams (SICE) Vijayawada
Vijaya Ph: 9030924949
________________________________________________________
Mathematics for Competitive Exams ( 104 )

PRACTICE QUESTIONS
Answer the following questions by selecting the most appropriate option.

1. Two numbers x and y are such that their sum is 9. If 2x – 3y = 5, then what is the value of
equal to twice their difference. What will be the 8x 3 27y 3 90xy 125 ?
2xy (1) 125 (2) 62.5
value of ?
2 2
(3) 250 (4) 25
3 6 10. If p = 20, then find the value of:
(1) (2)
25 25
3 2 .
1 3
(3) (4)
10 10 (1) 20 (2) 19
a b c (3) 4 (4) 21
2. If = 3, then what will be the
1 –a 1 –b 1 –c
3 1
2 2 2 11. If x = , then what is the value of
value of ? 3 1
1 –a 1 –b 1 –c
(1) 12 (2) 6 3x 2 ( 2 3 )x ?
(3) 3 (4) 24
(1) ( 2 3 (2) 2 3
x 2 + 1 – 2x 4 1 (3) 4 2 3 (4) 4 2 3
3. If 2 ; then, what is the value of ?
x + 1 + 2x 5 x
(1) 16 (2) 18 1
12. If x 3, then the value of x + 1 is
6

(3) 9 (4) 6 x
(1) 1 (2) 4
4. If a = 115, b = 113, c = –2, then what is the (3) 0 (4) 9
value of a3 + b3 + c3?
(1) 69000 (2) 12995 x 1 1
13. If , find the value of 1 .
(3) 38985 (4) 77970 x2 4x 1 13 x
(1) 11 (2) 17
1 3x (3) 12 (4) 13
5. If 3 = 7, find the value of 2
4x
1 13 8 3
(1) (2) 14. If 3a 8 b , then a = ?
19 19 8 3
3 5 (1) 2 (2) 0
(3) (4) (3) 1 (4) 3
5 3

1 1 15. For what value of k, will p4 + k + 18p2 be a perfect


6. If , then what is the value of x 3 + +5 ? square?
x x3
(1) 3 (2) 81
(1) 6 (2) 5
(3) 7 (4) 10 (3) 1 (4) 3

1 16. What will be the value of x 4 – 10x3 + 10x2 – 10x


7. If 2x + =4, then what is the value of +10 for x = 9?
3x
(1) 3 (2) 2
1
27x + ? (3) 1 (4) 0
8x
(1) 64 (2) 5 17. If a2bc = a4b = 2025, what is the value of 2c + 2?
(3) 107 (4) 189 (1) 15 (2) 20
(3) 25 (4) 30
x 1
8. If , then what is the value of 18. If a, b and c are in continued proportion and a = x 2
x2 5x 1 6
+ y2; b = y2 + z2; and c = z2 + x2, then which of the
1
x3 ? following is true?
x3
(1) y4 + z4 – x4 = x2z2 –y2z2 +x2y2
(1) 1298 (2) 1331
(2) y4 – z4 – x4 = x2z2 –y2z2 – x2y2
(3) 1333 (4) 1290
(3) y4 + z4 + x4 = x2z2 + y2z2 + x2y2
(4) y4 – z4 + x4 = x2z2 + y2z2 – x2y2

________________________________________________________
572
Subhash Institute of Competitive Exams (SICE) Vijayawada
Vijaya Ph: 9030924949
________________________________________________________
Mathematics for Competitive Exams ( 105 )
Section III – Chapter 10: Algebra

19. What will be the minimum value of (x – 5) 29. If x3 + 3x2 +3x = 7, then x is equal to
(x – 13)? (1) 2 (2) 3 6
(1) – 8 (2) 4 (3) 1 (4) –1
(3) 8 (4) – 16
1 1
30. If 2x 6, then 3x is equal to
20. If x 5 7 , then what is the value of 3x 2x
x 2 24x 8? (1) 4 (2) 8
(1) 2 (2) 4 (3) 9 (4) 12
(3) 8 (4) 16 1
31. If a + b + c = 1 and ab + bc + ca = then
21. If a2
+ b2
+ = 2(a – b – c) – 3, then the value of
c2 3
(a – b + c) is a : b : c is
(1) –1 (2) 3 (1) 1 : 2 : 2 (2) 2 : 1 : 2
(3) 1 (4) –2 (3) 1 : 1 : 1 (4) 1 : 2 : 1
22. If a = 25, b = 15, c = –10, then the value of 32. If a4 + a2b2 + b4 = 8 and a2 + ab + b2 = 4, then
3 3 3 the value of ab is
is (1) -1 (2) 0
(3) 2 (4) 1
(1) 30 (2) 15
2 2 2
(3) -30 (4) 15 33. If + + = 1 , then the value of
2 2 2
2
1 x –1 a2 b2 c2
23. If then the value of 2
is + 2 + 2
a 2
a +bc b +ca c +ab
1 (1) – 1 (2) 2
(1) a–1 (2) a – 1 (3) 0 (4) 1
2
1 34. If bc + ab + ca = abc, then the value of
(3) a+1 (4) a 1
2
b+c a+c a+b
+ +
24. If a, b, c are real and a2 + b2 + c2 = 2(a – b – c) – 3, bc a–1 ac b–1 ab c–1
then the value of a + b + c is
3
(1) 3 (2) 0 (1) (2) 1
(3) –1 (4) 1 2
1
1 1
(3) 0 (4)
25. If a2 + b2 + a2 + 2 = 4, then the value of a2 + b2 2
b
will be 1
35. What will be the value of x – if
1 x–4
(1) 1 (2) 1 2
2 2 1
x–4 + = 18 ?
1 x–4
(3) 2 (4) 2
2 (1) 14 (2) 16
(3) 15 (4) 8
1 2 1
26. If (x+ ) = 3, then x3 + 3 is equal to
x x 36. What is the value of “y2 – 2y + 4”, if y = 1 + 5 ?
(1) 3 (2) 2
(1) 8 5 (2) 6 2 5
(3) 1 (4) 0
(3) 8 (4) 6
2 1
27. If 2x+ =1, then the value of x3 + is 1 1
x x 37. If x 2, then the value of x 12 is
x x 12
13 11 (1) 2 (2) 0
(1) (2)
8 8 (3) 4 (4) - 4
11 13
(3) (4) 38. If m – 5, n = 2, then the value of (m3 – 125 n3 – 30
8 8
mn) is
1 1 (1) 8 (2) 7
28. If x 2, then the value of x 100 is
x x 100 (3) 9 (4) 6
(1) 2 (2) 0
(3) 1 (4) –2

________________________________________________________
573
Subhash Institute of Competitive Exams (SICE) Vijayawada
Vijaya Ph: 9030924949
( 106 )
________________________________________________________
Mathematics for Competitive Exams

39. Given that x3 + y3 = 72 and xy = 8 with x > y, then 45. (a2 + 2a)2 + 12(a2 + 2a) – 45 can be expressed as
the value of x – y is (1) (a 1)(a + 3)(a2 + 2a + 15)
(1) 4 (2) 4 (2) (a + 1)(a + 3)(a2 + 2a + 15)
(3) 2 (4) –2 (3) (a + 1)(a 3)(a2 + 2a + 15)
(4) (a 1)(a 3)(a2 + 2a + 15)
1 1
40. If x 1, then the value of x2 + 3x + + 7x + 1 1
x x2 46. If x = 3 + 2 2, then the values of x 3 + and
is x3
1
1 3 x 3– are respectively:
(1) (2) x3
2 7
(1) 234, 216 (2) 216, 234
(3) 1 (4) 2
(3) 198, 140 2 (4) 140 2 , 198
41. If x = 2, then the value of x3 + 27x2 + 243x + 631
is 47. Given that a + b + c = 2 and ab + bc + ca = 1, then
(1) 1233 (2) 1231 the value of (a + b)2 + (b + c)2 + (c + a)2 is
(3) 1321 (4) 1211 (1) 16 (2) 6
(3) 8 (4) 10
1 x 2 +3x+1 x3
42. If x + = 1, (x 0), then the value of is 48. If x2 + 4x + 3 = 0, then the value of is
x x 2 +7x+1 x 6 +27x 3 +27
(1) 2 (2) 0 1
1 (1) (2) 1
(3) (4) 1 2
2 1
(3) (4) 1
43. If a + b + c + = 0, find the value of 2
1 1 1 1 1 1 1 1
+ + + + + 49. If a 5, then the value of (a 3) 3
a3 b 3 c 3 (a+b) 3 (b+c) 3 (c+a) 3 a 3 (a 3) 3
(1) 3abc (2) 1 is
(3) 0 (4) 1 (1) 7 (2) 2
(3) 5 (4) 14
1 1
44. If x + y = 4 and + = 4, then the value of
x y 1 1 1
50. If a + =b+ =c+ where a b c 0, then
x3 + y3 is b c a
(1) 64 (2) 4 the value of a2 b 2 c 2 is
(3) 25 (4) 52 (1) 0 (2) 1
(3) abc (4) 1

________________________________________________________
574
Subhash Institute of Competitive Exams (SICE) Vijayawada
Vijaya Ph: 9030924949
________________________________________________________
Mathematics for Competitive Exams ( 107 )
Section III – Chapter 10: Algebra

ANSWERS AND EXPLANATIONS


1. (3) x + y = 2 (x – y) 1 9. (3) 2x – 3y = 5
6. (2)
x Cubing both sides:
x = 3y
Cubing both sides; 3 3
2xy 1
= =
2 2 2 2
10 3 1 1 1
x3 x x
2. (1) 8x 3 27 y 3 3 2 x 3 y 5 125
3 1
a b c 8x 3 27y 3 90xy 125
x3
1– a 1–b 1– c
=3+3 1 8x 3 27y 3 90xy 125 250
x3 + =0
x3
1 1 1 10. (4)
=6 1
1 – a 1 –b 1 – c 3
x + 3 +5 =5
x 3 p p2 3p 3 1
2 2 2
= 12 1 = 3
p3 3p 2 1 3p
1 – a 1 –b 1 – c 7. (4) 2x + =4
3x
x 2 + 1 – 2x 4 3
p 1
3
= p +1 = 20 +1 = 21
3. (2) Multiply both sides by 3/2
x 2 + 1 + 2x 5
1 11. (4)
1 3x + =6
2x
x 4 3 1
Cubing both sides: x=
1 5 3 1
x 1
27x + 3 1 3 1
1
8x =
Let 3 1 3 1
x 1 1
2x 2x 3 1
4 x=
3 1 2 3
1
27x + = 216 – 27 = 189
8x 1
5y – 10 = 4y + 8 y = 18 x= 2 3
2 3
8. (1) 6x x 2
5x 1
1
x x 2 11x 1 0 3x2 ( 2 3 )x

4. (4) Note that here, a + b + c = 0 1 = 3(2 3) ( 2 3 )( 2 3)


x 11 0
Therefore, a3 + b3 + c3 = 3abc x
6 3 3 2 3= 4 2 3
= 3 (115) ( 113) ( 2) = 77970. 1
x 11 1
1 x 12. (3) x 3
5. (1) 3x =7 x
4x Cubing both sides:
2
1 2
12x 2
1 28x 1 1 1 x 3
3 x
x3 x x
12x 2 28x 1 1
x2 2 3
3x 3 1 x2
x 33 1331
12x 2 29x 1 x3
1
x2 1
3x 3 1 x2
x 1298
28x 1 29x 1 x3

3x 1
=
57x 19

________________________________________________________
575
Subhash Institute of Competitive Exams (SICE) Vijayawada
Vijaya Ph: 9030924949
________________________________________________________
Mathematics for Competitive Exams ( 108 )

1 17. (2) a2bc = a4b c = a2 21. (3) Given a2 + b2 + c2 = 2 (a – b


Multiplying both sides by x , – c) – 3
x 2025 = 3 × 3 × 5 × 5 × 3× 3 =
we get: 34 × 25 (a – b + c) = ?
1 1 1 Comparing with a4b, we get a2 + b2 + c2 = 2 (a – b – c) – 3
x x2 x
x x2 x a = 3; b = 25 and c = a2 = 9 a2 + b2 + c2 = 2a – 2b – 2c – 1
1 1 1 –1–1
x3 x x Therefore, 2c + 2 = 2 × 9 + 2
x x3 x = 20. a2 – 2a +1 + b2 + 2b + 1 + c2
1 +2c + 1 = 0
x3 0 18. (1) If a, b and c are in continued
x3 (a – 1)2 + (b + 1)2 + (c + 1)2 = 0
proportion, then b2 = ac.
x 6
1 0 Therefore,
Therefore, a = 1, b = –1, c = –1
(y2 + z2)2 = (x2 + y2) (z2 + x2)
13. (2) 13 x x 2
4x 1 So, (a – b + c) = 1 – (–1) + (–1)
y4
+ + 2z4 y2z2 = x2z2 + x4 =1
x 2 17 x 1 0
+y2z2 +x2y2 22. (2) a 3 b 3 c 3 3abc = (a + b +
1 y4 + z4 – x4 = x2z2 –y2z2 +x2y2
x x 17x c) (a2 b 2 c 2 ab bc ca)
x
19. (4) (x – 5) (x – 13) = x2 – 18x We can reduce (a + b +c) (a2 +
1 +65 b2 + c2 – ab – bc – ca) as
x 17
x For a quadratic equation, ax2 + follows:
bx + c = 0, the minimum value 2 2
8 3 (a + b + c) (a + b2 + c2 – ab –
14. (1) 3a 8 b of the equation is calculated as 2
8 3
4 ac b2
. bc – ca)
Rationalising, we get: 4a
1
(a + b + c) (2a2 + 2b2 +2c2
8 3 8 3 8 3 Therefore, for the quadratic 2
8 3 8 3 8 3 equation, x2 – 18x +65 = 0, the 2ab – 2bc – 2ca)
minimum value will be
8 9 6 8 1
4(1)(65) ( 18)2 (a b c)
8 9 2
4(1)
6 8 17 3a 8 b (a b)2 (a c)2 (c a)2
260 324
By comparing LHS and RHS 4 a3 +b3 +c 3 -3abc
Therefore, =
3a = 6 and b = 17 2 2 2
64
16
a=2 4 1 2 2 2

2
15. (2) (p2)2+ k + 2 (9) p2 20. (2) x 5 7 2 2 2

If k = 92; (p2)2+ k + 2 (9) p2 = (p2 Squaring both sides, we get: 1


=
+ 9)2 2
x 5 7 2 35 = 12 2 35
Therefore, for k = 92; p4 + k + 1
= 25+15-10 = 15 (as a = 25,
18p2 will be a perfect square. Again squaring both sides, we 2
get: b = 15 and c = –10)
16. (3) x4 – 10x3 + 10x2 – 10x +10
x2 144 140 48 35 23. (1) Given that
= x4 – 9x3 – x3 + 9x2 + x2 – 9x –x
+10 284 48 35 1
2x a
= 94 – 9.93 – 93 + 9. 92 + 92 – Therefore, a
9.9 –9 +10 x 2 24 x 8 284 48 35 Let us assume that the value of
= 94 –94 – 93 + 93 + 92 – 92 –9 24(12 2 35) 8 a is 4.
+10 284 48 35 288 48 35 8 So,
=1 =–4+8=4

________________________________________________________
576
Subhash Institute of Competitive Exams (SICE) Vijayawada
Vijaya Ph: 9030924949
________________________________________________________
Mathematics for Competitive Exams ( 109 )
Section III – Chapter 10: Algebra

1 1 1 31. (3) a + b + c = 1
2x 2 a and b
2 a b
1
x
5 ab + bc + ca =
a= 1, b = 1 3
4
The value of a2 + b 2 = 1 + 1 = 2 a2 + b 2 + c 2

5
2 26. (4) = (a + b + c)2 2(ab + bc + ca)=
1
x 2
1 4 2 1
1 1 1 1
2 2 x3 x x2 –1 3 3
x x 1 5 5 x3 x x2
1
4 4 1 1
2
1
x x – 2–1 a=b=c=
3 x x 3
2
3 3–3 a:b:c = 1 : 1 : 1
By checking the given option, 0 32. (4) Given: a4 + b4 + a2b2 = 8 and
we get that,
a2 + b2 + ab = 4
1 2
a 1 27. (2) 2 x 1
2 x 4 4 2 2
8
2 2
= =2
1 4
4 1 1 1
2 x Add and subtract a2 b2 in the
x 2
3 numerator
2 1 1 1 1
3
x3 3x x = 4 4 2 2 2 2

24. (3) Given that x3 x x 2 2 2


=2

a2 b2 c2 2 a b c 3 1 1 3 11 2 2 2 2 2
x3 = = =2
a2 b2 c2 2 a b c 3 0 x3 8 2 8 2 2

2 2 2 2
a2 2a 1 b2 2b 1 1 =2
28. (1) x + =2 2 2
x
c2 2c 1 0
a 1
2
b 1
2
c 1
2
0
x2 2x + 1 = 0 (a2 + b2 – ab) =2
(x – 1)2 = 0 a2 + b2 = 4 –ab (since a2 + b2 +
Above equation is only possible ab = 4)
x=1
when: =2
1 1
a 1
2
0 x100 + = 1100 + ab = 1
x100 1100
b 1
2
0
=1+1=2 33. (4)
2 29. (3) + x3 3x2
+ 3x = 7 + x3 3x2 a2 bc b 2 ca
c 1 0
+ 3x + 1 = 7 + 1 = 8 (x + 1)3 a2 bc b 2 ca
Hence, = 23 x+1=2 x=1 c 2 ab
1
c 2 ab
a 1 1
30. (3) 2x + = 6 (given)
b 1 3x Simplifying the expression.
c 1 6x 2 1 a2 bc b2 ca c 2 ab
=6 1
Thus, the value 3x a2 bc b2 ca c 2 ab
2 2 2
a b c
a b c 6x 2 1 3 3
=6× 2 2
a bc b ca c ab 2

1 1 1 3x 2 2
bc ca ab
1
1 6x2 1 a bc b2 ca c 2 ab
=9
2x
1 1 For positive value,
25. (3) a2 + 2 + b2 + 2=0
a2 b2 6x 2 1
=9 a2 b2 c2
2x 2x 1
1
2
1
2
a2 bc b2 ca c2 ab
a + b =0 1
a b 3x + =9
2x

________________________________________________________
577
Subhash Institute of Competitive Exams (SICE) Vijayawada
Vijaya Ph: 9030924949
________________________________________________________
Mathematics for Competitive Exams ( 110 )

34. (2) Given that, 38. (1) 43. (3) a + b + c = 0


bc ab ca abc m 5n = 2 a+b= c, b + c = a, c + a
ab ca abc bc = b
On cubing both sides
a b c abc bc 1 1 1 1
(m 5n)3 = 23 a3 b3 c3 (a b)3
bc a 1
b c 1 1
a m3 125n3 15mn(m 5n) 3
=8 (b c) (c a)3
Similarly,
1 1 1 1
ac b 1 m3 125n3 30mn = 8
a c a3 b3 c 3 ( c)3
=
b 39. (3) 1 1
ab c 1 Given, x3 + y3 = 72 ( a) 3 ( b) 3
a b
c Let x = 4 ,y=2 1 1 1 1 1 1
= =0
Then the value of, a3 b3 c3 c3 a3 b3
43 + 23 = 64 + 8 =72
b c a c 44. (4)
xy = 8
bc a 1 ac b 1
x y
a b x y=4 2=2 4
xy
ab c 1 40. (1)
1 x y 4
1 xy 1
Given 1 4 4
35. (4) x
2 x2 3x 1 (x + y)3 = x3 + y3 + 3xy (x + y)
1 Expression,
x 4 x 2 7x 1
x 4 43 = x3 + y3 + 3 × 1× 4
2

x 4
2 1
2
Dividing numerator and x3 + y3 = 64 12 = 52
x 4 denominator by x, we get:
16 45. (1)
1
x 3 1
1 = x ( 1) (a2 + 2a)2 + 12(a2 + 2a) 45
x 4 16 4 1 x
x 4 x 7
x = a4 + 4a2 + 4a3 + 12a2 + 24a
1 1 3 4 1 45
x 4 4 8 =
x 4 1 7 8 2 = a4 + 16a2 + 4a3 + 24a 45
36. (3) 41. (1) = (a2 + 2a 3)(a2 + 2a + 15)
2
y 2y 4 Expression x3 + 27x2 + 243x +
2
= (a 1)(a + 3)(a2 + 2a + 15)
1 5 21 5 4 631
46. (3)
= x3 + 9(3x2) + 92(3x) + 93 – 93
1 5 2 5 2 2 5 4 1 1 3 2 2
+ 631
x 3 2 2 3 2 2 3 2 2
4 4 8 = (x + 9)3 – 729 + 631 = (2 +
9)3 – 98 = 3 2 2
37. (2)
1 = 1331 – 98 = 1233 1
Given, 2 x = 3 2 2 3 2 2 6
x
42. (3) Dividing numerator and
x2 + 1 = 2x denominator by x, we get: And
1
x2 2x + 1 = 0 1 x 3 2 2 3 2 2 4 2
2 3 x
x 3x 1 x
(x 1)2 = 0 = Now,
x 2 7x 1 1
7 1
3
1 1
x 1=0 x=1 x x x3 3 x
1 3 4 1 x x3 x
12 1 =
1 1 0 1 7 8 2 1
12
63 x3 3 6
x3

________________________________________________________
578
Subhash Institute of Competitive Exams (SICE) Vijayawada
Vijaya Ph: 9030924949
________________________________________________________
Mathematics for Competitive Exams ( 111 )
Section III – Chapter 10: Algebra

1 On putting x = 3 in the given 1 1


x3 216 18 198 50. (2) a + =b+
x3 expression: b c
1
3
1 1 3 1 1 b c
x x3 3 x 3 a–b= =
x x3 x 6 3 c b bc
= 3 27 3 27
3 1 b c
4 2 x3 3 4 2 27 bc =
1 a b
x3 36 36 27
1 3 1 1
x3 4 2 3 4 2 On putting x = 1 in the given b+ =c+
x3 c a
expression:
= 64 2 2 12 2 1 1
1
3 b–c=
a c
6 3
= 140 2 = 1 27 1 27 c a
1 = ac
47. (2) 1 ac
1 27 27 c a
(a + b + c)2 = a2 + b2 + c2 + 2(ab =
+ bc + ca) 49. (4) Subtracting 3 from both b c
sides, we get: 1 1
22 = a2 + b2 + c2 + 2× 1 a+ =c+
1 b a
a2 + b 2 + c 2 = 4 – 2 = 2 (a – 3) =2
a 3 1 1
c – a=
Now, b a
Cubing both sides, we get:
a b
(a + b)2 + (b + c)2 + (c + a)2 (a – 3)3
1 = ab
3 ab
a 3
= 2(a2 + b2 + c2) + 2(ab + bc + a b
ca) =
1 c a
3 a 3 = 23
=2×2+2×1=6 a 3 bc× ac × ab = a2b2c2
48. (4) x2 + 4x + 3 = 0 1 b c c a a b
(a – 3)3 3
3×2=8 = × × =1
a 3 a b b c c a
(x + 3)(x + 1) = 0
1
(a – 3)3 = 14
x = 3, 1 a 3
3

________________________________________________________
579
Subhash Institute of Competitive Exams (SICE) Vijayawada
Vijaya Ph: 9030924949
________________________________________________________
Mathematics for Competitive Exams ( 112 )

11 GEOMETRY

GEOMETRY
Geometry is a branch of Mathematics that deals with the measurement of various parameters
of geometric figures.

Important Terms in Geometry


Line
A line is a one-dimensional figure—it has length but no width or thickness. It can be extended
indefinitely in both directions; thus, a line is infinite. A finite line, that is, a line having two end
points is called a line segment. It is named after its end points, such as line segment AB
below:

SMART TIP

Remember that the shortest distance between any two points is always
a straight line.

Angle
Angle is the figure formed by two rays sharing a common endpoint. The common endpoint is
called the vertex of the angle. In the figure given below, the angle formed by rays BA and BC is
denoted by ABC or CBA and point B is the vertex of the angle.

Types of angles
Right angle: An angle equal to 90° is known as a right angle.

________________________________________________________
Subhash Institute of Competitive Exams (SICE) Vijayawada
Vijaya Ph: 9030924949
________________________________________________________
Mathematics for Competitive Exams ( 113 )
Section III – Chapter 11: Geometry

Acute angle: An angle less than 90° is called an acute BOC are complementary angles as their sum is equal
angle. to 90 .

Obtuse angle: An angle more than 90° and less than


180 is known as an obtuse angle. Adjacent Complementary Angles

Non-adjacent Complementary Angles

Supplementary angles: Two angles are supplementary if


Straight angle: An angle equal to 180° is called a
their sum is equal to 180°. As with complementary
straight angle.
angles, it is not necessary that two angles be adjacent. In
the given figure, AOC and BOC are supplementary
angles as their sum is equal to 180 .
Here, x = 180

Reflex angle: An angle more than 180° but less than


360° is called reflex angle.

Adjacent Supplementary Angles

Pairs of angles
Adjacent angles: Two non-overlapping angles are
adjacent if they have a common side and a common
vertex. In the following figure, 1 and 2 are adjacent
Non-adjacent Supplementary Angles
angles.
Linear pair of angles: Two angles that are adjacent and
supplementary are known as linear pair of angles. In the
given figure, 1 and 2 form a linear pair of angles.

Complementary angles: Two angles are complementary


if their sum is equal to 90°. It is not necessary that the
two angles be adjacent. In the given figure, AOC and
1+ 2 = 180

________________________________________________________
581
Subhash Institute of Competitive Exams (SICE) Vijayawada
Vijaya Ph: 9030924949
( 114 )
________________________________________________________
Mathematics for Competitive Exams

Vertically opposite angles: When two lines intersect, the Parallel Lines
angles that are opposite to each other are called Parallel lines are two lines that lie in the same plane but
vertically opposite angles. Vertically opposite angles are never intersect one another.
always equal.

In the given figure, 1 and 3 are vertically opposite


angles. Similarly, 2 and 4 are vertically opposite
angles. Line l1 and l2 above are said to be parallel and are
denoted as l1 l2.

Parallel lines and a Transversal


Two parallel lines AB and CD are intersected by a
transversal EF at points G and H respectively. It can be
observed in the given figure that four angles are formed
at the points G and H.
Here, 1= 3, 2= 4

Also, 1+ 2+ 3+ 4 = 360

SMART TIP

The sum of measures of an angle


around a point is always 360°.

Angle Bisector
A line bisects an angle if it divides the angle into two
equal parts. Two angles are congruent if their measures
are equal. Here, 1 = 3, 2 = 4, 5 = 7 and 6 = 8
(Vertically opposite angles)

1 = 5, 2 = 6, 3 = 7 and 4 = 8
(Corresponding angles)

In the given figure, line BD bisects the ABC. 3= 5 and 4= 6 (Alternate interior angles)

Thus ABD = CBD 1= 7 and 2= 8 (Alternate exterior angles)

Perpendicular Lines
When two lines intersect at a 90° angle, they are called
PLANE GEOMETRICAL FIGURES
perpendicular lines. All figures which occupy an area are called plane figures.
For example, triangles, quadrilaterals and circles are all
plane figures. All plane figures are two dimensional and
do not have any volume.

Triangle
A triangle is a three-sided closed plane figure. The sum
of three interior angles of any triangle is always equal to
In the given figure, line segment MN is perpendicular to
the line segment AB.

________________________________________________________
582
Subhash Institute of Competitive Exams (SICE) Vijayawada
Vijaya Ph: 9030924949
________________________________________________________
Mathematics for Competitive Exams ( 115 )
Section III – Chapter 11: Geometry

180°. Triangles can be categorised on the basis of sides Exterior Angle Theorem of a Triangle
as well as on the basis of angles. An exterior angle of a triangle is equal to the sum of the
opposite two interior angles of the triangle.
On the basis of sides, triangles can be classified as:

Equilateral triangle: A triangle in which all three


sides are equal is called an equilateral triangle. Each
angle of an equilateral triangle is equal to 60 .
Isosceles triangle: A triangle in which two sides are
equal is known as an isosceles triangle. In an
isosceles triangle, angles opposite to equal sides are
also equal.
Scalene triangle: A triangle in which all three sides
are different is called a scalene triangle. In a scalene Here,
triangle all three angles have different measures.
x a b
On the other hand, on the basis of angles, triangles
can be categorised as: Internal Angle Bisector Theorem

Acute angle triangle: A triangle in which all


three angles are less than 90° is known as an
acute-angled triangle.
Obtuse angle triangle: A triangle that has one
angle greater than 90° is called an obtuse-
angled triangle.
Right angle triangle: A triangle in which one
angle is equal to 90° is called a right-angled
triangle. If AD is the bisector of A in triangle ABC, then

Some Important Theorems on Triangles AB BD


Pythagoras Theorem AC DC
Pythagoras theorem applies only to right triangles and
Apollonius Theorem
states that in a right triangle, the square of the
Median of a triangle is the line segment that joins a
hypotenuse (the longest side) is equal to the sum of
vertex and the mid-point of the opposite side of the
squares of the other two sides.
vertex.

SMART TIP

If the two shorter sides of a right-


angled triangle are 3 and 4, the
hypotenuse will always be 5. These
are called Pythagorean triplets. This If AD is the median of triangle ABC then 2(AD2 + BD2)
also holds true for any multiples of = AB2 + AC2
these numbers such as 6, 8 and 10.

________________________________________________________
583
Subhash Institute of Competitive Exams (SICE) Vijayawada
Vijaya Ph: 9030924949
________________________________________________________
Mathematics for Competitive Exams ( 116 )

Similarity of Triangles congruent. In the given figure, AB = PQ, BC = QR and AC


Two triangles are similar if their corresponding angles = PR
are congruent and the ratio of their corresponding sides
is in proportion.

ABC PQR

Side-Angle-Side the (SAS) Congruence


If two sides and included angle of one triangle are equal
to the corresponding sides and the included angle of
In similar triangles ABC and PQR another triangle, then triangles are congruent.

AB BC AC
PQ QR PR

In two similar triangles ABC and PQR shown above

Ratio of corresponding sides = ratio of heights = ratio of


perimeters = ratio of medians = ratio of inradii = ratio of
circumradii = ratio of angle bisectors

2
area of ABC AB
And 2
area of PQR PQ

AA (Angle-Angle) Similarity
In two triangles, if two pairs of corresponding angles are
equal, the triangles will be similar.

SSS (Side-Side-Side) Similarity In the above figure AB = PQ, AC = PR and A= P


If the corresponding sides of two triangles are in the
ABC PQR
same ratio, the triangles will be similar.

SAS (Side-Angle-Side) Similarity Angle-Side-Angle (ASA) Congruence


If an angle of a triangle is equal to an angle of another If two angles and the included side of one triangle are
triangle and the sides including these angles are in the equal to two corresponding angles and the included side
same ratio, the triangles will be similar. of another triangle then two triangles are congruent. In
the following figure
Congruence of Triangles
BC = QR, B= Q and C= R
Two triangles are congruent if they have same shape and
size. If two triangles are congruent, then their
corresponding sides and angles are equal.

Conditions for Congruence of Triangles


Side-Side-Side (SSS) Congruence
When three sides of triangle are equal to corresponding ABC PQR
sides of another triangle, then the triangles are

________________________________________________________
584
Subhash Institute of Competitive Exams (SICE) Vijayawada
Vijaya Ph: 9030924949
________________________________________________________
Mathematics for Competitive Exams ( 117 )
Section III – Chapter 11: Geometry

Angle-Angle-Side (AAS) Congruence Here,


If two angles and a non-included side of one triangle are BAC
equal to the corresponding angles and the corresponding BOC = 90 + = 90 + x
2
side of another triangle, then two triangles are
congruent. ACB
Similarly AOB = 90 + and COA = 90 +
2
ABC
2
Circumcentre
Circumcentre of a triangle is the point of intersection of
A = P , B = Q and BC = QR perpendicular bisectors of its sides. Circumcentre is
equidistant from all three vertices of a triangle. In the
ABC PQR
given figure point O is the circumcentre of triangle ABC.
Right Angle-Hypotenuse-Side (RHS) Congruence
If the hypotenuse and one side of one right-angled
triangle are equal to the corresponding hypotenuse and
the side of another right angled triangle, then two
triangles are congruent.

In a right-angled triangle, circumcentre is the mid-point


of hypotenuse; in an acute-angled triangle, it is inside the
triangle; and in an obtuse-angled triangle, it is outside
the triangle. The radius of circle circumscribing the
AC PR and BC = QR triangle is called a circumradius.
ABC = PQR = 90
Centroid
ABC PQR
Centroid of a triangle is the point of intersection of its
Geometric centres of a triangle three medians. Centroid divides the medians in the ratio
2:1. In the triangle ABC, AD, BE and CF are the medians
Incentre
of triangle. In the given figure, point G is the centroid of
Incentre of a triangle is the point of intersection of its
triangle ABC.
angular bisectors. Incentre of a triangle is equidistant
from its three sides. The circle drawn with incentre as its
centre is called an incircle and the radius of this circle is
called an inradius. In the given figure, point O is the
incentre of triangle ABC.

AG BG CG 2
Here,
GD GE GF 1

________________________________________________________
585
Subhash Institute of Competitive Exams (SICE) Vijayawada
Vijaya Ph: 9030924949
________________________________________________________
Mathematics for Competitive Exams ( 118 )

Inradius and circumradius of a Triangle


If r be the inradius and R be the circumradius of a
triangle ABC then

area of ABC
r=
s Properties of Rectangle

a b c 1. Opposite sides are equal and parallel.


Where, s = = semi perimeter of a triangle
2 2. Diagonals are equal
3. Diagonals bisect each other.
abc
R= , where a, b and c are the sides
4 area of ABC Square
of triangle Square is a regular quadrilateral having all four sides
equal and each angle equal to 90°.
Key Points to Remember for Triangles
Sum of any two sides of triangle is greater than its
third side.
Sides opposite to equal angles of a triangle are
equal.
If the sum of squares of two smaller sides is more
than the square of the largest side, then the triangle
is an acute-angled triangle.
Properties of a Square
If the sum of the squares of two smaller sides is less
than the square of the largest side, then the triangle 1. Diagonals are congruent.
is an obtuse-angled triangle. 2. Diagonals bisect each other at 90°.
In equilateral and isosceles triangles, the altitude
3. Diagonals bisect the opposite angles.
from the vertex bisects the base.
In any triangle, the larger angle has a longer side
Parallelogram
opposite to it.
A quadrilateral whose opposite sides are parallel is
In a triangle, the line segment joining the midpoints
of two sides is parallel to and half of the third side. called a parallelogram.
A median divides the triangle into two equal areas.
The sum of three altitudes of a triangle is less than
its perimeter.
If a triangle is drawn in such a manner that one of
the sides of a parallelogram is the base of the
triangle and the third vertex of the triangle lies on
the opposite parallel side, then the area of the Properties of a Parallelogram
triangle is half the area of the parallelogram. 1. Opposite angles of a parallelogram are equal.

Quadrilateral 2. Opposite sides of a parallelogram are equal.


Quadrilateral is a four-sided closed plane figure. The sum 3. Diagonals of a parallelogram bisect each other.
of four interior angles of a quadrilateral is 360°. 4. Sum of the adjacent angles of a parallelogram
Let us now discuss the properties of different types of is 180°.
quadrilaterals.
Rhombus
Rectangle Rhombus is a parallelogram in which all four sides are
A rectangle is a quadrilateral in which every angle is equal.
equal to 90°.

________________________________________________________
586
Subhash Institute of Competitive Exams (SICE) Vijayawada
Vijaya Ph: 9030924949
________________________________________________________
Mathematics for Competitive Exams ( 119 )
Section III – Chapter 11: Geometry

Each interior angle of a regular polygon of n sides


n 2
= 180
n
Sum of exterior angles of a polygon of n sides = 360
Each exterior angle of a regular polygon of n sides
360
=
Properties of a Rhombus n
1. Opposite angles are equal. n n 3
Number of diagonals in a polygon of n sides =
2
2. Sum of adjacent angles is 180°.
3. Diagonals bisect each other at 90°. Area of some commonly used polygons
4. Diagonals bisect the opposite angles. A regular hexagon can be divided in to six equilateral
triangles.
Trapezium
Area of a regular hexagon of side ‘a’ = 6 × area of
A quadrilateral having a pair of parallel opposite sides is
called a trapezium. If non-parallel sides of a trapezium equilateral triangle of side ‘a’
are equal, it is called an isosceles trapezium. 3 2 3 3 2
6 a = a
4 2

2
Area of regular octagon of side ‘a’ = 2a 1 2

Circle
A circle is a set of points in a plane which are at a equal
Polygon
A polygon is a plane figure that is bound by a finite distance from a fixed point in a plane. The fixed point is
number of sides. The minimum number of sides in a called the centre of circle.
polygon is 3. Important concepts with relation to circles
If all sides (in length) and all angles are equal, it is called circumference

a regular polygon.

The following table lists different types of polygons and


the number of sides they have:

Polygon Number of Sides


Triangle 3
Quadrilateral 4
Pentagon 5
Hexagon 6
Heptagon 7
Octagon 8
Nonagon 9 Chord: A chord is a segment whose end points lie on the
Decagon 10 circle.

Angles of a Polygon Diameter: The diameter is the line segment connecting


The sum of interior angles of a polygon of n sides two points on the circle that passes through the centre of
the circle. The diameter is the longest chord in a circle.
= (n – 2) × 180

________________________________________________________
587
Subhash Institute of Competitive Exams (SICE) Vijayawada
Vijaya Ph: 9030924949
________________________________________________________
Mathematics for Competitive Exams ( 120 )

Radius: The distance between the centre of a circle and


any point on the circle is called the radius. It is basically
half of the diameter.

Circumference: The distance around the outer boundary


of a circle is called its circumference. It is same as the
perimeter of the circle.
4. Angles subtended by the same chord in alternate
Circumference, C = 2 r segment are equal
Tangent: A tangent is a line that touches the circle at any
one point on its circumference. A line drawn tangent to a
circle is perpendicular to the radius at the point of
tangency.

Arc: An arc is a portion of the circumference of a circle.


An arc that is exactly half the length of the circumference
is called a semicircle.

Theorems on Circle Here, ACB and ADB are subtended by the chord
1. The perpendicular from the centre of a circle to a AB therefore If ACB = x then ADB is also equal
chord bisects the chord; or we can say that the to x.
perpendicular bisector of a chord passes through 5. The angle subtended by an arc at the centre is
the centre of a circle. double the angle subtended by it at any point on the
remaining part of the circle.

If OC AB then AC = BC

2. Equal chords of a circle are equidistant from the


centre. Here, AOB = 2 ACB

6. Angle in a semicircle is 90 . In the figure given below


AB is the diameter of circle and ACB = 90

If AB = CD then OE = OF and vice versa.

3. When two circles intersect, then the line joining their


centres bisects their common chord at right angle. In
7. Products of intercepts of two intersecting chords are
the figure PQ AB and AC = BC
equal.

________________________________________________________
588
Subhash Institute of Competitive Exams (SICE) Vijayawada
Vijaya Ph: 9030924949
________________________________________________________
Mathematics for Competitive Exams ( 121 )
Section III – Chapter 11: Geometry

Length of traverse common tangents

Here, AE × BE = CE × DE

8. Lengths of tangents drawn from an external point


are equal. 2
AB = CD = distance between centres R r

Number of Common Tangents for Two Circles


No common tangent can be drawn for two circles if
one circle lies completely inside the other circle.
Only one common tangent can be drawn for two
circles touching each other internally.
Two common tangents can be drawn for two circles
intersecting each other at two distinct points.
In this figure, P is the external point from where two Three common tangents can be drawn for two circles
tangents PA and PB are drawn. Here, PA is equal touching each other externally.
to PB. Four common tangents can be drawn for two non-
touching circles.
9. The angle between a chord and tangent at the end
point of the chord is equal to the angle subtended by Secant of a Circle
the same chord in alternate segment. This theorem A line that intersects a circle at two distinct points is
is known as alternate segment theorem. called secant of circle.

Theorems on Secant
1. Square of the length of the tangent from an external
point is equal to the product of intercepts of a
secant passing through this point.

Here, ABD = ACB

Length of Direct Common Tangents

Here PT2 = AP × PB

2. If two secants are drawn from an external point, then


the product of one secant and its external segment
is equal to the product of other secant and its
external segment.
2
P1Q1 = P2Q2 = distance between centres R r

________________________________________________________
589
Subhash Institute of Competitive Exams (SICE) Vijayawada
Vijaya Ph: 9030924949
________________________________________________________
Mathematics for Competitive Exams ( 122 )

Some Important Examples:


1. The interior angle of a regular polygon exceeds its
exterior angle by 108°. The number of sides of the
polygon is
Solution:
Each interior angle of a regular polygon of n sides
n 2 180
=
n
Each exterior angle of a regular polygon of n sides
Here PA and PC are two secants then PA × PB = PC × PD
360
=
Cyclic Quadrilateral n
Cyclic quadrilateral is a quadrilateral that can be n 2 180 360
= 108
inscribed in a circle. In a cyclic quadrilateral, the sum of n n
the opposite angles is equal to 180 . n = 10
2. The ratio of each interior angle to each exterior
angle of a regular polygon is 3 : 1. The number of
sides of the polygon is
(1) 7 (2) 9
(3) 8 (4) 6
Solution:
Number of sides of polygon = p
In cyclic quadrilateral ABCD, A+ D= B+ C = 180 According to the question,

The exterior angle of a cyclic quadrilateral is equal to its (2 4) 90 360 3


/
p p 1
interior opposite angle.
2 4 90 3
360 1
2 4 3
4 1
2p 4 = 12
2p = 16
p=8
3. The measure of an angle whose supplement is
Here CBE is the exterior angle at vertex B therefore
three times as large as its complement is
ADC = CBE
Solution:
SMART TIP Let the required angle be A°
According to the question:
For a given perimeter, a circle has the
maximum area among all the plane 180 – A = 3 (90 – A)
figures. 180 – A = 270 – 3A
2A = 90
A = 45°

________________________________________________________
590
Subhash Institute of Competitive Exams (SICE) Vijayawada
Vijaya Ph: 9030924949
________________________________________________________
Mathematics for Competitive Exams ( 123 )
Section III – Chapter 11: Geometry

4. The diameter of each wheel of a car is 70 cm. If 7. In a right angled triangle, the product of two sides
each wheel rotates 400 times per minute, then the is equal to half of the square of the third side i.e.,
22 hypotenuse. One of the acute angles must be
speed of the car (in km/hr) is Take
7 Solution:
Solution: Let b is as the base, p is as the perpendicular and h
Distance covered by wheel in one rotation = is as the hypotenuse of the right-angled triangle.
22 Now, as per the question,
2× × 35 = 220 cm
7
1 2
h = bp
Distance covered by car in one minute = 400 × 220 2
= 88,000 cm = 0.88 km
h2 = 2bp …(1)
Distance covered by car in one hour = 60 × 0.88 =
As in right-angled triangle,
52.8 km
h2 p2 b2
Speed of car = 52.8 km/h
From (1)
5. A tangent is drawn to a circle of radius 6 cm from
a point situated at a distance of 10 cm from the 2bp = p2 + b2
center of the circle. The length of the tangent will (b – p)2 = 0
be b=p
Solution: Since two sides of a right-angled triangle are equal,
triangle is isosceles. So, each acute angle = 45
8. Quadrilateral ABCD is circumscribed about a
circle. If the lengths of AB, BC and CD are 7 cm,
8.5 cm and 9.2 cm, respectively, then the length
(in cm) of DA is
OQ is radius of circle = 6 cm, OP = 10 cm Solution:
And PQ is a tangent,
OQ is perpendicular on PQ
So in right-angled triangle OPQ

PQ (OP 2 OQ2 ) (102 62 ) (100 36)

(64 8 cm
6. If the number of vertices, edges and faces of a
a + b = 7 …..(1)
rectangular parallelepiped are denoted by v, e and
f, respectively, the value of (v – e + f ) is b + c = 8.5 …..(2)
Solution: c + d = 9.2 …..(3)
In a rectangular parallelepiped Adding (1), (2) and (3), we get,
Vertices = v = 8 a + 2(b + c) + d = 24.7
Edges = e = 12 a + 2 × 8.5 + d = 24.7
Faces = f = 6 a + d = 24.7 – 17 = 7.7 cm
So v e + f = 8 – 12 + 6 = 2 DA = 7.7 cm

________________________________________________________
591
Subhash Institute of Competitive Exams (SICE) Vijayawada
Vijaya Ph: 9030924949
________________________________________________________
Mathematics for Competitive Exams ( 124 )

PRACTICE QUESTIONS
Answer the following questions by selecting the most appropriate option.

1. In a triangle PQR, the length of PQ and PR are (x 9. XY and XZ are tangents to a circle, ST is another
+ y) and x 2 + y 2 , respectively. What will be the tangent to the circle at the point R on the circle,
which intersects XY and XZ at S and T,
measure of PRQ if the length of side RQ is respectively. If XY = 15 cm and TX = 9 cm, then RT
2xy ? is
(1) 30 (2) 60 (1) 4.5 cm (2) 6 cm
(3) 120 (4) 90 (3) 7.5 cm (4) 3 cm
2. A triangle is formed by joining the mid points of 10. Two circles touch externally. Their sum is 130
all the three sides of an equilateral triangle. What sq cm and the distance between their centres is
is the ratio of the area of the triangle formed to 14 cm. The radius of the smaller circle is
the area of the equilateral triangle? (1) 5 cm (2) 2 cm
(1) 1 : 4 (2) 1 : 3 (3) 3 cm (4) 4 cm
(3) 1 : 2 (4) 2 : 5
11. ABC is a cyclic triangle and the bisectors of
3. What is the maximum number of common ABC, ABC and BCA meet the circle at P, Q,
tangents that can be drawn between two circles? and R, respectively. Then the angle RQP is
(1) 1 (2) 2 B B
(3) 3 (4) 4 (1) 90°– (2) 90°+
2 2
4. ABC is a triangle and the sides AB, BC and CA are A C
(3) 90°– (4) 90°+
produced to E, F and G, respectively. If CBE = 2 2
ACF = 130 then the value of GAB is 12. If PQRS is a rhombus and SPQ = 500 then RSQ
(1) 100 (2) 90 is
(3) 80 (4) 130 (1) 65° (2) 75°
5. The ratio of each interior angle to each exterior (3) 55° (4) 45°
angle of a regular polygon is 3 : 1. The number of 13. In a rhombus ABCD, A = 600 and AB = 12 cm.
sides of the polygon is Then the diagonal BD is
(1) 7 (2) 9 (1) 10 cm (2) 12 cm
(3) 8 (4) 6
(3) 2 3 cm (4) 6 cm
6. If two supplementary angles differ by 44°, then
14.
one of the angle is
such that BD:DC = 4:5. If the area of
(1) 68° (2) 72°
cm2, then the area of DC is
(3) 102° (4) 65°
(1) 90 cm2 (2) 75 cm2
7. The measures of two angles of a triangle are in (3) 50 cm2 (4) 60 cm2
the ratio 4 : 5. The sum of these two measures is
15. The length of the portion of the straight line 3x +
equal to the measure of the third angle. Find the
4y = 12 intercepted between the axes is
smallest angle.
(1) 7 (2) 4
(1) 40°
(3) 3 (4) 5
(2) 50°
(3) 10° 16. A tangent is drawn to a circle of radius 6 cm from
(4) 90° a point situated at a distance of 10 cm from the
center of the circle. The length of the tangent will
8. If two medians BE and CF of a triangle ABC be
intersect each other at G and if BG = CG, BGC = (1) 8 cm (2) 5 cm
600, BC = 8 cm, then the area of the triangle ABC (3) 4 cm (4) 7 cm
is
(1) 96 2 17. Two chords of length a unit and b unit of a circle
(2) 48 2 make angles 60° and 90° at the centre of the
(3) 48 cm2 circle, respectively. The correct relation is
(4) 64 2 3
(1) b = 2a (2) b = a
2
(3) b = 2 a (4) b = 3 a

________________________________________________________
592
Subhash Institute of Competitive Exams (SICE) Vijayawada
Vijaya Ph: 9030924949
________________________________________________________
Mathematics for Competitive Exams ( 125 )
Section III – Chapter 11: Geometry

18. The measure of an angle whose supplement is 27. Internal bisectors of Q and
three times as large as its complement is intersect at O. If ROQ = 96 , then the value of
(1) 45° (2) 75°
RPQ is
(3) 60° (4) 30°
(1) 12 (2) 6
19. A square is inscribed in a quarter-circle in such a (3) 36 (4) 24
manner that two of its adjacent vertices lie on the
two radii at an equal distance from the centre, 28. If the measure of three angles of a triangle are in
while the other two vertices lie on the circular the ratio 2 : 3 : 5, then the triangle is
arc. If the square has sides of length x, then the (1) equilateral (2) isocscels
radius of the circle is (3) obtuse angled (4) right angled

(1) 2x (2)
2x 29. If the sum and difference of two angles are 22/9
radian and 36 , respectively, then the value of the
16x 5x smaller angle in degree taking the value of as
(3) (4) 22/7 is
4 2
(1) 60 (2) 48
20. Two poles of height 7 m and 12 m stand on a (3) 52 (4) 56
plane ground. If the distance between their feet is
12 m, the distance between their top will be 30. In two triangles, ABC and PQR, BC=QR , AB=PQ and
(1) 13 m (2) 19 m extended m ABD = 120° and extended M PQS
(3) 17 m (4) 15 m
- 120° By which condition, are the two triangles
21. In a parallelogram PQRS, angle P is four times of congruent?
angle Q, then the measure of R is (1) Angle – Side – Side (2) Angle – Side – Angle
(1) 72° (2) 144° (3) Side – Side – Side (4) Side – Angle – Side
(3) 36° (4) 130° 31. If an equilateral triangle ABC is inscribed in a
22. circle with centre at O, then BOC, COA and
BE intersect at right angles. If the lengths of AD AOB are respectively
and BE are 9 cm and 12 cm, respectively, then (1) 120 , 120 , 120 (2) 60 , 60 , 60
the length of AB (in cm) is (3) 80 , 120 , 160 (4) 50 , 60 , 70
(1) 10 (2) 10.5
(3) 9.5 (4) 11 32. Find the area of the triangle formed by the graph
9C
23. The three angles of a triangle are F= + 32 with the F-axis and C-axis.
5
6x 2x
(x + 15 ), 6 and 30 the triangle is 2460 2860
5 3 (1) sq. unit (2) sq. unit
9 9
a /an
2560 2760
(1) scalene (2) isosceles (3) sq. unit (4) sq. unit
(3) right angled (4) equilateral 9 9

24. If the number of vertices, edges and faces of a 33. If in a triangle ABC, AB is smaller than AC, then
rectangular parallelepiped are denoted by v, e and AC – AB is
f, respectively, the value of (v – e + f ) is 1
(1) AC (2) < BC
(1) 4 (2) 2 2
(3) 1 (4) 0 (3) > BC (4) = BC
25. The area of the triangle formed by the graphs of 34. Radii of two circles are 6.3 cm and 3.6 cm. If they
the equations x = 0, 2x + 3y = 6 and x + y = 3 is touch each other internally, then the distance
(1) 3 sq. unit (2) 1 sq. unit between their centres is
(1) 2.7 cm (2) 3.7 cm
(3) 1 sq. unit (4) 4 sq. unit (3) 10.1 cm (4) 9.1 cm
26. Among the equations, 35. There is a circle with the radius 5 cm and the
x + 2y + 9 = 0; 5x – 4 = 0; 2y – 13y = 0, 2x – 3y = 0, perpendicular distance from the centre of the
the equation of the straight line passing through circle to the chord of the circle is 3 cm. Then the
the origin is length of the chord is
(1) 2x – 3y = 0 (2) 5x – 4 = 0 (1) 5 cm (2) 8 cm
(3) x + 2y + 9 = 0 (4) 2y – 13 = 0 (3) 4 cm (4) 6 cm

________________________________________________________
593
Subhash Institute of Competitive Exams (SICE) Vijayawada
Vijaya Ph: 9030924949
( 126 )
________________________________________________________
Mathematics for Competitive Exams

36. In a ABC, A – B=20° , B– C=32° . Then A 39. The value of a sector in a pie-chart is equal to
is Angle of the sector
(1) × Total value of the data
(1) 90 360
(2) 750 Angle of the sector
(2) × Total value of the data
(3) 80 180
(4) 84 Angle of the sector
(3) × 100
37. A point D is taken from the side BC of a right- 180
angled triangle ABC, where AB is hypotenuse. Angle of the sector
(4) × 100
Then 360
(1) AB2 + CD2 = BC2 + AD2
40. In the figure, AB II CD and EF intersects them. The
(2) CD2 + BD2 = 2 AD2
value of x is
(3) AB2 + AC2 = 2 AD2
(4) AB2 = AD2+BD2
38. The angle subtended at the centre of a circle by
an arc length equal in length to the radius of the
circle is defined as
(1) 90 degrees
(2) one grade
(3) one degree (1) 14 (2) 28
(4) one radian (3) 24 (4) 18

________________________________________________________
594
Subhash Institute of Competitive Exams (SICE) Vijayawada
Vijaya Ph: 9030924949
________________________________________________________
Mathematics for Competitive Exams ( 127 )
Section III – Chapter 11: Geometry

ANSWERS AND EXPLANATIONS


1. (4) Since, PQ2 = x2 + y2 +2xy BAC = 180 50 50 BG = GC
and PR2 + RQ2 = x2 + y2 + 2xy = 80
GBC = GCB = 60
Therefore, PQR is a right angled GAB = 180 80 = 100 Area of GBC
triangle with PQ as the 5. (3) Number of sides of polygon
hypotenuse and PRQ is 90 3
=p = 82 16 3 sq.cm
4
2. (1) According to the question,
Area of ABC
(2 4) 90 360 3
/
p p 1 = 3 16 3 48 3 sq.cm
9. (2)
2 4 90 3
360 1 The tangents drawn from an
exterior point to a circle are
2 4 3 equal.
4 1
All the four triangles are XY = XZ = 15
congruent to each other. Areas 2p 4 = 12
of triangles ADE, DBF, DEF and 2p = 16
EFC are equal. So, the ratio is,
p=8
DEF DEF 1
ABC 4 DEF 4 6. (1) According to the question,

3. (4) Circles whose centers are at Let supplementary angles = x TZ = XZ TX = 15 9 = 6 cm


a distance more than the sum and 180 x
So, RT = TZ = 6 cm
of their radii will have a total of 180 x x = 44
4 tangents in common as 10. (3)
shown in the figure: 180 2x = 44
PP' = r1 + r2 = 14cm
180 44 136
x= = 68
2 2
7. (1) Two angles of triangle be 4x
and 5x.
r12 + r22 = 130
4. (1) According to question
CBE = 130 2(4x + 5x)=180 r12 + r22 = 130

ABC = 180 130 = 50 18x = 180 r12 + (14 r1)2 = 130

ACF = 130 x = 10 r12 + 196 + r12 28r1 = 130

ACB = 180 130 = 50 Smallest angle = 4 × 10 = 40 2r12 28r1 + 66 = 0

8. (2) BGC = 60 r1 2 14r1 + 33 = 0


(r1 11)(r1 3)=0
r1 = 11 cm or 3 cm
r2 =3 cm or 11 cm
Radius of smaller circle
= 3 cm

________________________________________________________
595
Subhash Institute of Competitive Exams (SICE) Vijayawada
Vijaya Ph: 9030924949
________________________________________________________
Mathematics for Competitive Exams ( 128 )

11. (1) In a rhombus ABCD, Intersection point on x -axis =


(4, 0)
BAD = 60°
OP = 4 and OQ = 3
AB = AD = 12 cm
ABD = ADB = 60° PQ (OP 2 OQ2 ) (42 32 )

ABD is an equilateral PQ (16 9) 25 5 units


triangle.
16. (1)
BD = AB = AD = 12 cm
14. (2)
BQP = BAP
A
BQP =
2
BQR = BCR
OQ is radius of circle = 6 cm, OP
C = 10 cm
BQR =
2
And PQ is a tangent,
A C
BQP + BQR = Let, AE BC OQ is perpendicular on PQ
2 2
1 So in right-angled triangle OPQ
1 Area of ABD BD AE
= RQP = (180 B) = 2
2 1 PQ (OP 2 OQ2 ) (102 62 )
Area of ADC CD AE
( A = B = C = 180) 2 (100 36)
B 60 BD
RQP = 90° 64 8 cm
2 ADC CD
12. (1) In the rhombus PQRS, 17. (3)
60 5
ADC 75 cm2
PQ = QR = RS = SP 4
15. (4)

SPQ = 50°
PQS = PSQ = Radius = r
Given, = OP = OQ = OR = OS
2 2 Straight line 3x + 4y = 12
130
POQ = 90° ;
65 Now putting x = 0 in equation,
2 PQ = b, RS = a
we get
PSR = 180° 50° = 130° From OPQ,
3 × 0 + 4y = 12
RSQ = 130° 65° = 65° OP2 + OQ2 = PQ2
y=3
13. (2) r2 + r2 = b2
So point of intersection on y -
axis = (0, 3) b2
r2
2
Now put y = 0 in 3x + 4y = 12
b
3x + 4 × 0 = 12 r …(i)
2
x=4

________________________________________________________
596
Subhash Institute of Competitive Exams (SICE) Vijayawada
Vijaya Ph: 9030924949
________________________________________________________
Mathematics for Competitive Exams ( 129 )
Section III – Chapter 11: Geometry

In ORS, 20. (2) 6 2


x 1 = 180 51
ROS = 60°; 5 3

OR = RS (15 18 10)
x = 129
15
ORS = OSR = 60°
43x = 129 × 15
r=a …(ii)
From equation (i) and (ii) we get x = 3 × 15 = 45
P + Q = 180°
b
Now angles of triangle are
a 4 Q + Q = 180°
2 (x + 15 ) = 45 + 15 = 60
5 Q = 180°
b a 2 6x
180 + 6 = 54 + 6 = 60
 Q 36  5
18. (1) 5

Let the required angle be A° R = 180° 36° = 144° 2x


+ 30 = 30 + 30 = 60
3
According to the question: 21. (4)
Since all angles are equal
180 – A = 3 (90 – A) Let radius of sphere = r unit
Therefore, it is an equilateral
180 – A = 270 – 3A 4 triangle.
Then, 3 2

2A = 90 3
24. (2) In a rectangular
A = 45° r = 3 unit parallelepiped

19. (4) So diameter = 2r = 6 unit = 6 Vertices = v = 8


cm.
OT= OU and TU = x unit Edges = e = 12
22. (1)
Faces = f = 6
So v – e + f = 8 – 12 + 6 = 2
25. (2)

Since centroid divides each


l median in 2:1

From OTU, 2(OT) 2 = TU2 2 2


AG = × (AD) = × (9) = 6
3 3
cm
x 2 2
OT = And BG = × (BE) = × (12)
2 3 3
QT = Diagonal of square TUQR = = 8 cm Putting x = 0 in 2x + 3y = 6 we
In ABG, AGB = 90 get y = 2

OQ (OT 2 2
TQ ) Coordinates of point of
AB = (AG2 BG2 ) = (62 82 )
intersection on y – axis = (0, 2)
2
x 2 = (36 64)
2x Now putting y =0 in 2x + 3y = 6
2
= cm we get x = 3

x2 5x 2 6x 2x Point of intersection on
2 x2 23. (4) x + 15 + +6 + x –axis = (3, 0)
2 2 5 3
+ 30 = 180 In x + y = 3
5x
OQ Putting x = 0 we get y = 3
2

________________________________________________________
597
Subhash Institute of Competitive Exams (SICE) Vijayawada
Vijaya Ph: 9030924949
________________________________________________________
Mathematics for Competitive Exams ( 130 )

And on putting y = 0 we get x So 2n + 3n + 5n = 180 31. (1)


=3
10n = 180
So the required area of triangle
n = 18
formed by the graph of equation
So angles of triangle = 18 ×
= Area of ( OAC OAB)
2,18× 3,18 × 5
1 1 3
= (3 × 3) – (3 × 2) = sq. So angles of triangle = 36, 54 BOC + AOB + COA = 360
2 2 2
and 90 (angles around a point)
units
Hence it is a right angled In
1
=1 sq. unit triangle.
2 OB = OC = OA (radius of circle)
29. (3) Let two angles be A and B
26. (1) BC = AB = CA (sides of
So equilateral triangle)
Point of origin (0, 0) = (x, y)
22
Put the value of x = 0, y = 0 in A+B=( ) radian
equation 9 congruent.
22 180
2x – 3y = 0 = × BOC = AOB = COA
9
2×0–3×0=0 BOC = AOB = COA
22 180 7
= 20 7 140 = 120
0=0 (9 22)
32. (3) Putting C = 0 in the given
So equation 2x – 3y = 0 is a A + B = 140 …(i) equation, we get
straight line which passes
through the origin. and F = 32
27. (1) A – B = 36 …(ii) Putting F = 0, we get
From equation (i) and (ii), we get 160
C=
A = 88 , B = 52 9

30. (4) Thus the given line intersects


coordinate axes at (0, 32) and
( 160/9, 0)

In
In OQR
AB = PQ, BC = QR (given)
OQR + ORQ + QOR = 180
ABD = PQS = 120
1 1
( PQR) + ( PRQ) + 96 ABD + ABC = 180 (linear
2 2
pair)
= 180 Area of
ABC = 60 coordinate axes and the given
1
( PQR + PRQ) = 84 line
2 Similarly, PQR = 60
PQR) + PRQ = 168 Two sides and their included 1 160 2560
= 32 sq. unit
angle in two triangles is the 2 9 9
In PQR ,
same, thus by side angle side 33. (2) Difference between any two
QPR= 180 168 = 12 congruence sides of a triangle is less than
28. (4) Let angles of triangle = 2n, its third side
3n and 5n AC – AB < BC

________________________________________________________
598
Subhash Institute of Competitive Exams (SICE) Vijayawada
Vijaya Ph: 9030924949
________________________________________________________
Mathematics for Competitive Exams ( 131 )
Section III – Chapter 11: Geometry

34. (1) Distance between the From (4) + (3) we get 39. (1) Value of a sector in a pie
centres of two circles touching chart
internally = difference of radii 3 B = 192 B = 64
Angle of the sec tor
= 6.3 – 3.6 = 2.7 cm Thus, A = 64 + 20 = 84 = × Total
360
35. (2) 37. (1) By Pythagoras theorem, value of the data
In 40. (2)
(AD)2 = AC2 + CD2

AC = 2AB

AB = OA 2 OB 2 52 32 AC2 = AD2 CD2 ….. (1) In the given figure


= 4 cm
In AGE = CHF = 4x + 12
AC = 2 × 4 = 8 cm (corresponding angles)
(AB)2 = AC2 + BC2
36. (4) 4x + 12 + 2x = 180 (Linear
AB2 = AD2 + BC2 [From (1)]
A B = 20 …..(1) pair)
(AB)2 + (CD)2 = AD2 + BC2
B C = 32 …..(2) 6x = 168
38. (4) The angle subtended at the
A+ B+ C = 180 …..(3) centre of the circle by an arc x = 28
whose length is equal to the
From (2) (1) we get
length of radius of that circle is
2 B ( A+ C) = 12 …..(4) one radian.

________________________________________________________
599
Subhash Institute of Competitive Exams (SICE) Vijayawada
Vijaya Ph: 9030924949
________________________________________________________
Mathematics for Competitive Exams ( 132 )

12 MENSURATION

MENSURATION
Mensuration is a branch of mathematics in which we deal with the calculation of length,
breadth, height, area, volume, etc.

2-D Figures
S. No. Name Figure Perimeter Area Nomenclature

Right angled 1 b = base


1. AB + BC + CA ×b×h
triangle 2 h = height

Equilateral 3 2 a = side of
2. 3a a
triangle 4 triangle

a = length of
two equal
Isosceles b
3. 2a + b 4a
2
b
2
sides and b =
triangle 4 length of the
third side

s=
s(s a)(s b)(s c)
semiperimeter
Scalene Where s =
4. a+b+c a, b and c are
triangle a b c sides of
2 triangle

________________________________________________________
Subhash Institute of Competitive Exams (SICE) Vijayawada
Vijaya Ph: 9030924949
________________________________________________________
Mathematics for Competitive Exams ( 133 )
Section III – Chapter 12: Mensuration

S. No. Name Figure Perimeter Area Nomenclature

l = length
b = breadth
5. Rectangle 2(l + b) lb d = diagonal

= l 2 b2

a = side
6. Square 4a a2
d = diagonal = a 2

b = base
7. Parallelogram 2(AB + BC) bh
h =height

a and b are parallel


AB + BC + CD + 1
8. Trapezium (a b)h sides
AD 2
h = height

a = side
1
9. Rhombus 4a d1 d 2 d1, d2 are diagonals
2

10. Circle 2 r r2 r = radius

2
11. Sector r r = radius
360

________________________________________________________
601
Subhash Institute of Competitive Exams (SICE) Vijayawada
Vijaya Ph: 9030924949
( 134 )
________________________________________________________
Mathematics for Competitive Exams

S. No. Name Figure Perimeter Area Nomenclature

2 R = external radius
12. Disc or ring
2
R r
r = internal radius

Points to remember 1
Area of trapezium = × (7x + 4x) × 18 = 693 cm2
2
Distance covered in one revolution of a wheel is
equal to its circumference. x=7
If a circle is inscribed in a square, then the side of Larger side of trapezium = 7 × 7
the square is the diameter of the circle. = 49 cm
If a square is inscribed in a circle, then the diameter 3. Nine equal squares are placed side by side to
of the circle is the diagonal of the square. make a single rectangle whose perimeter is 320
cm. What is the area of each square?
1
Area of any quadrilateral = × one diagonal × (sum Solution:
2
Let x be the side of the square
of perpendiculars on that diagonal from opposite
vertices) Length of rectangle formed by placing nine square
side by side = 9
Another formula for calculating area of quadrilateral
Breadth of rectangle = x
1
= × Product of diagonals × sine of angle between Perimeter of rectangle = 2(9x + x) = 20
2
20x = 320 x = 16
them.
Area of square = 162
Some Important Examples: = 256 cm2
1. A cow is tied to one corner of a square plot of side
4. A rectangular park with length 18 cm and width
24 m by a rope of 14 m. Find the area which
12 cm is surrounded by a path of 3 cm. Find the
cannot be grazed by the cow?
area of the park.
Solution:
Solution:
Area of plot = 576 m2
Length of the outer rectangle = 18 + 3 + 3 = 24 cm
Area that can be grazed by cow
Breadth of the outer rectangle = 12 + 3 + 3 = 18 cm
90 22
= 14 14 = 154 m2 Area of the outer rectangle = 24 × 18 = 432 cm2
360 7
Area of the inner rectangle = 18 × 12 = 216 cm2
Area that cannot be grazed = 576 154 = 422 m2
Area of the path = 432 – 216 = 216 cm2
2. Parallel sides of a trapezium are in ratio 7:4 and
5. Diagonals of a rhombus are 30 cm and 16 cm. Find
the perpendicular distance between the parallel
the perimeter of rhombus.
sides is 18 cm. If the area of a trapezium is 693
cm2 find the larger side of the parallel side. Solution:
Solution: Side of rhombus = 15 2 8 2 = 17 cm
Let 7x and 4x be the parallel sides of the trapezium. Perimeter of rhombus = 17 × 4 = 68 cm

________________________________________________________
602
Subhash Institute of Competitive Exams (SICE) Vijayawada
Vijaya Ph: 9030924949
________________________________________________________
Mathematics for Competitive Exams ( 135 )
Section III – Chapter 12: Mensuration

6. If the ratio of the areas of two squares is 9:1, the Solution:


ratio of their perimeters is: 22
Circumference of circle = 2 42
Solution: 7
The ratio of the perimeter of the two squares whose = 264
areas are in the ratio 9:1 is According to the given problem,
9 3 Perimeter of rectangle = Circumference of circle
=
1 1 2(6x + 5x) = 264
Thus, 22x = 264
Ratio is 3:1 x = 12
7. A circular wire of radius 42 cm is cut and bent in Small side = 5x = 5 × 12 = 60 cm
the form of a rectangle whose sides are in the
ratio of 6:5. The smaller side of the rectangle is:

3-Dimensional figures
S.No Name Figure Volume Curved/lateral Total surface Nomenclature
surface area area

l = length
1. Cuboid lbh 2(lh + bh) 2(lb + bh + lh) b = breadth
h = height

3 2 2
2. Cube a 4a 6a a = side

r = radius of base
3. Cylinder r 2h 2 rh 2 r (r h) h = height of
cylinder

4 3
4. Sphere r 4 r2 4 r2 r =radius
3

Hemi- 2 3
5. r 2 r2 3 r2 r = radius
sphere 3

________________________________________________________
603
Subhash Institute of Competitive Exams (SICE) Vijayawada
Vijaya Ph: 9030924949
( 136 )
________________________________________________________
Mathematics for Competitive Exams

S.No Name Figure Volume Curved/lateral Total surface Nomenclature


surface area area

r = radius
1 2 h = height
6. Cone r h rl r (l r)
3
l = slant height

R = radius of base
of frustum
r = radius of top of
Frustum of frustum
1 R r l
7. a cone r2 r R R2 h R r l l = slant height of
3 R2 r2
frustum
l=
2 2
h R r

(Perimeter of Lateral surface


Right its base) × area + 2(area of
8. Area of base × height
Prism
height base)

1
(perimeter Lateral surface
Right 1 2
9. area of base height of base slant area + area of
pyramid 3 base
height)

Euler’s formula Solution: Let r be the radius of cone

In any regular solid, Volume of cylinder = volume of cone


1
Number of faces + Number of vertices = Number of × 82× 2 = × × r2 × 6
3
edges + 2
SMART TIPS r = 8 cm
2. If the surface area of two spheres are in the ratio
Remember that a cylinder is made up
4 :9, then the ratio of their volumes will be
of two circles and one curved
rectangle. Solution: Let r and R be the radii of sphere 1 and
sphere 2, respectively

Some Important Examples: r2


1. A cylinder with base radius 8 cm and height 2 cm R2
is melted to form a cone of height 6 cm. The radius 4 r2
of the cone will be 9 R2

________________________________________________________
604
Subhash Institute of Competitive Exams (SICE) Vijayawada
Vijaya Ph: 9030924949
________________________________________________________
Mathematics for Competitive Exams ( 137 )
Section III – Chapter 12: Mensuration

r 2 Area of square base = 10 × 10 = 100 cm2


R 3 Total surface area = 260 + 100 = 360 cm 2
r3 4. The portion of a ditch 48 m long, 16.5 m wide and
=
R3 4 m deep that can be filled with stones and earth
3
available during excavation of a tunnel, cylindrical
2 in shape, of diameter 4 m and length 56 m, is
3
22
Take
= 8:27 7
3. Base of a right pyramid is a square of side 10 cm. Solution: Volume of ditch = 48 × 16.5 × 4
If the height of the pyramid is 12 cm, then its total = 3168 m3
surface area is 22
Volume of tunnel = × 22× 56 = 704
Solution: 7
Portion of ditch that can be filled with stone and
704 2
earth = =
3168 9
5. The radii of two solid iron spheres are 1 cm and 6
cm, respectively. A hollow sphere is made by
melting the two spheres. If the external radius of
Height of pyramid = AB = 12 cm the hollow sphere is 9 cm, then its thickness (in
cm) is
10
BC = = 5 cm Solution: Let r be the internal radius of hollow
2
sphere.
AC = 122 52 = 13 cm
According to the question,
1
Area of ADE = AC DE 4 4
2 3
r3 = 3 3

3 3
1
= × 13 × 10 = 65 cm2 r3 = 93 (13 + 63) = 512
2
r = 8 cm
Lateral surface area = 4 ×Area of ADE = 4 × 65 =
260 cm2 Thickness of hollow sphere = 9 – 8 = 1 cm

________________________________________________________
605
Subhash Institute of Competitive Exams (SICE) Vijayawada
Vijaya Ph: 9030924949
( 138 )
________________________________________________________
Mathematics for Competitive Exams

PRACTICE QUESTIONS
Answer the following questions by selecting the most appropriate option.

1. The portion of a ditch 48 m long, 16.5 m wide and 8. The area of the parallelogram whose length is 30
4 m deep that can be filled with stones and earth cm, width is 20 cm and one diagonal is 40 cm is
available during excavation of a tunnel, (1) 100 15 cm2 (2) 150 15 cm2
cylindrical in shape, of diameter 4 m and length
(3) 200 15 cm2 (4) 300 15 cm2
22
56 m, is Take
7 9. The area of a circle is 324 sq cm. The length of
1 1 its longest chord (in cm) is
(1) Part (2) Part (1) 28 (2) 32
2 4
(3) 36 (4) 38
2 1
(3) Part (4) Part 10. The area of a rhombus is 256 sq cm. and one of
9 9
its diagonals is twice the other in length. Then the
2. If a hemisphere is melted and four spheres of length of its largest diagonal is
equal volume are made, the radius of each sphere (1) 16 cm (2) 24 cm
will be equal to (3) 32 cm (4) 48 cm
(1) 1/2 of the radius of the hemisphere
(2) 1/6th of the radius of the hemisphere 1
11. If the side of a square is x 1 units and its
(3) Radius of the hemisphere 2
(4) 1/4th of the radius of the hemisphere 3 x
diagonal is units, then the length of the side
2
3. The perimeter of a rhombus is 60 cm and one of
of the square would be
its diagonals is 24 cm. The area (in sq cm) of the
1
rhombus is (1) unit (2) 2 units
(1) 432 (2) 216 2
(3) 108 (4) 206 4
(3) units (4) 1 unit
3
4. A spherical ball of radius 1 cm is dropped into a
conical vessel of radius 3 cm and slant height of 6 12. For an equilateral triangle, the ratio of the in-
cm. The volume of water (in cm3) that can just radius and the outer-radius is
immerse the ball is (1) 1 : 2 (2) 1 : 3
4 (3) 1 : 2 (4) 1 : 3
(1) (2)
3 3
13. If a and b are lengths of the sides of a right
5
(3) (4) 3 triangle whose hypotenuse is 10 and whole area
3 is 20, then the value of (a + b)2 is
5. The in-radius of a triangle is 6 cm, and the sum of (1) 180 (2) 160
the length of its sides is 50 cm. The area of the (3) 140 (4) 120
triangle (in sq cm) is 14. The circumference of a triangle is 24 cm and the
(1) 50 (2) 56 circumference of its in-circle is 44 cm. Then the
(3) 150 (4) 300 22
area of the triangle is (taking )
6. One of the angles of a right-angled triangle is 7
15°, and the hypotenuse is 1 m. The area of the (1) 84 sq cm (2) 68 sq cm
triangle (in sq cm) is (3) 56 sq cm (4) 48 sq cm
(1) 1200 (2) 1215
(3) 1220 (4) 1250 15. If the length of each of the two equal sides of an
isosceles triangle is 10 cm, and the adjacent
7. If the height of a cylinder is 4 times its angle is 45°, then the area of the triangle is
circumference, the volume of the cylinder in (1) 12 2 sq cm (2) 15 2 sq cm
terms of its circumference c is
(3) 20 2 sq cm (4) 25 2 sq cm
(1) 4 c3 (2) 2 c3
2c3 c3 16. The length of the diagonal of a rectangle with
(3) (4) sides 4 m and 3 m would be
(1) 7 m (2) 14 m
(3) 12 m (4) 5 m

________________________________________________________
606
Subhash Institute of Competitive Exams (SICE) Vijayawada
Vijaya Ph: 9030924949
________________________________________________________
Mathematics for Competitive Exams ( 139 )
Section III – Chapter 12: Mensuration

17. The height of a cone is 30 cm. A small cone is cut 25. A large solid sphere is melted and moulded to
off at the top by a plane parallel to the base. If its form identical right circular cones with the base
1 radius and height the same as the radius of the
volume is th of the volume of the given cone, sphere. One of these cones is melted and
27
then at what height above the base is the section moulded to form a smaller solid sphere. Then the
made? ratio of the surface area of the smaller to the
(1) 19 cm (2) 20 cm surface area of the larger sphere is
(3) 12 cm (4) 15 cm (1) 1 : 34 /3 (2) 1 : 23 /2
18. ABCD is a trapezium with AD and BC as parallel (3) 1 : 32/3 (4) 1 : 24 /3
sides. E is a point on BC. The ratio of the area of
26. A conical cup is filled with ice-cream. The ice-
ABCD to that of AED is
cream forms a hemispherical shape on its open
AD BE
(1) (2) top. The height of the hemispherical part is 7 cm.
BD EC The radius of the hemispherical part equals the
AD BE AD BC height of the cone. Then the volume of the ice-
(3) (4)
AD CE AD 22
cream is
19. If the surface area of a sphere is 346.5 cm2, then 7
22 (1) 1078 cubic cm (2) 1708 cubic cm
its radius taking is (3) 7108 cubic cm (4) 7180 cubic cm
7
(1) 7 cm (2) 3.25 cm 27. A lawn is in the form of a rectangle having its
(3) 5.25 cm (4) 9 cm breadth and length in the ratio 3 : 4. The area of
20. The height of the right pyramid whose area of the the lawn is 1 hectare. The breadth of the lawn
base is 30 m2 and volume is 500 m3 is 12
(1) 50 m (2) 60 m is
(3) 40 m (4) 20 m (1) 25 metres (2) 50 metres
(3) 75 metres (4) 100 metres
21. In an equilateral triangle of side 24 cm, a circle is
inscribed, touching its sides. The area of the 28. A right circular cone is 3.6 cm high and radius of
its base is 1.6 cm. It is melted and recast into a
remaining portion of the triangle is ( 3 = 1.32) right circular cone with radius of its base as 1.2
(1) 98.55 sq cm (2) 100 sq cm cm. Then the height of the cone (in cm) is
(3) 101 sq cm (4) 95 sq cm (1) 3.6 (2) 4.8
22. The base of a right prism is an equilateral (3) 6.4 (4) 7.2
triangle. If the lateral surface area and volume 29. The area of a rectangle is thrice that of a square.
are 120 cm2 and 40 3 cm 2 respectively, then the The length of the rectangle is 20 cm and the
side of the base of the prism is 3
breadth of the rectangle is times that of the
(1) 4 cm (2) 5 cm 2
(3) 7 cm (4) 40 cm side of the square. The side of the square, in cm,
is
23. If the perimeter of a rhombus is 2p unit and the
(1) 10 (2) 20
sum of lengths of diagonals is m unit, then the
(3) 30 (4) 60
area of the rhombus is
1 2 1 30. If the surface area of two spheres are in the ratio
(1) m p sq unit (2) mp 2 sq unit
4 4 4 :9, then the ratio of their volumes will be
1 2 1 2 (1) 4 : 9 (2) 16 : 27
(3) (m p 2 ) sq unit (4) (p m2 ) sq unit
4 4 (3) 8 : 27 (4) 16 : 9
24. A ball of lead 4 cm in diameter is covered with 31. The volume of a conical tent is 1232 cu.m and the
gold. If the volumes of the gold and lead are area of its base is 154 sq. m. Find the length of
equal, then the thickness of gold the canvas required to build the tent, if the
canvas is 2 m in width.
given 3 2 1.259 is approximately
22
(1) 5.038 cm (2) 5.190 cm (Take )
7
(3) 1.038 cm (4) 0.518 cm
(1) 270 m (2) 272 m
(3) 276 m (4) 275 m

________________________________________________________
607
Subhash Institute of Competitive Exams (SICE) Vijayawada
Vijaya Ph: 9030924949
________________________________________________________
Mathematics for Competitive Exams ( 140 )

32. Assume that a drop of water is spherical and its 34. The diagonals of a rhombus are 12 cm and 16 cm
diameter is one-tenth of a cm. A conical glass has respectively. The length of one side is
a height equal to the diameter of its rim. If 32,000 (1) 8 cm (2) 6 cm
drops of water fill the glass completely, then the (3) 10 cm (4) 12 cm
height of the glass, in cm, is
35. A rectangular block of metal has dimensions 21
(1) 1 (2) 2
cm, 77 cm and 24 cm. The block has been melted
(3) 3 (4) 4
into a sphere. The radius of the sphere is
33. The total number of spherical bullets, each of
(Take as 22 )
diameter 5 decimeter, that can be made by 7
utilizing the maximum of a rectangular block of (1) 21 cm (2) 7 cm
lead with 11 metre length, 10 metre breadth and (3) 14 cm (4) 28 cm
5 metre width is (assume that > 3)
(1) equal to 8800 (2) less than 8800
(3) equal to 8400 (4) greater than 9000

________________________________________________________
608
Subhash Institute of Competitive Exams (SICE) Vijayawada
Vijaya Ph: 9030924949
________________________________________________________
Mathematics for Competitive Exams ( 141 )
Section III – Chapter 12: Mensuration

ANSWERS AND EXPLANATIONS


1. (3) Volume of ditch = 48 × 16.5 5. (3) 1
Area of PQR =
× 4 = 3168 m3 2
22 1 3 1 3 1 1 2
Volume of tunnel = × 22× 56 = = m
7 2 2 2 2 2 8
= 704 = 1250 cm2
Portion of ditch that can be 7. (4) Let the radius of cylinder = r
filled with stone and earth
Therefore, height of cylinder = 4
704
= ×2 r=8 r
3168
Area of required triangle = Area
2 Volume of cylinder = r2h =
= of POQ + Area of POR +
9 c 3

Area of QOR r2 × 8 r = unit3


2. (1) Let r and R be the radii of
the hemisphere and sphere, 1
= (PQ × OT + QR × OU + PR 8. (2) Area of parallelogram = 2 ×
respectively. 2 Area of triangle
× OS)
2 4 By Heron’s formula, area of
Then 3
= 4 3
1
3 3 = × 6 × (PQ + QR + PR) triangle
2
r3 = 8R3 20 30 40
1 45 20
r = 2R = × 6 × 50 = 150 cm2 2
2
45 30 45 40
r 6. (4)
R=
2
= 45 25 15 5
3. (2)
= 75 15 cm2
Therefore, area of parallelogram
= 150 15 cm2
Sin 15 = sin (45 30 )
9. (3) Area of circle = 324 cm2
= sin 45 cos 30 cos 45 sin = r2
30
r2 = 324 r = 18 cm
1 3 1 1 3 1 Length of its longest chord =
60 =
Side of rhombus = = 15 cm 2 2 2 2 2 2 Length of diameter = 2 × radius
4
= 36 cm
Cos 15 = cos (45 30 )
BE = 152 122 10. (3) Let the length of shorter
= cos 45 cos 30 + sin 45 sin diagonal of rhombus be l
BE = 9 cm 30
Length of longer diagonal of
Diagonal BD = 9 × 2 = 18 cm
1 3 1 1 3 1 rhombus = 2l
=
1 2 2 2 2 2 2
Area of rhombus = × 24 × 18 1
2 Area of rhombus = 2 =
= 216 cm2 3 1 2
PQ = PR × sin 15 = m 256 cm2
2 2
4. (2) Volume of water that can
immerse the ball = Volume of l = 16 cm
3 1
4 QR = PR × cos 15 = m Length of longer diagonal = 2
the ball = × ×1×1×1 2 2
3 = 32 cm
4
= cm3
3

________________________________________________________
609
Subhash Institute of Competitive Exams (SICE) Vijayawada
Vijaya Ph: 9030924949
________________________________________________________
Mathematics for Competitive Exams ( 142 )

1 Let r be the radius of in-circle. Similarly, the height of the


11. (4) Side of square = (x + 1)
2 smaller cone be h and the larger
OT = OU = OS = r
units cone be H.
2 r = 44 r = 7 cm As cone is cut parallel to the
Diagonal of square = 2 side
Therefore, the required area of base, both the cones are
1 3 x
= x 1 1 similar.
2 2 triangle = 
2 Thus,
+1=3– 1
24 7 h r
2 …… (1)
=1 H R
= 84 cm2
1 Now, according to the question:
Side of square = (1 + 1) 15. (4)
2
1 2
= 1 unit 3 rh 1
1 R2H 27
12. (3) In an equilateral triangle, all 3
the sides are equal. r 2h 1
Let the length of side of R2H 27
equilateral triangle be ‘a’ units. After putting the values from 1,
In-radius of an equilateral h3 1
Side a H3 27
triangle =
2 3 2 3 In PQR 30 30 30
h3 ----(Putting H = 30)
27
Outer radius of an equilateral 1
PO = PQ sin 45 = 10 × h 10
Side a 2
triangle = So, the height above the section
3 3 = 5 2 cm is made = 30 – 10 = 20 cm
a a 1
Required ratio = : Required area = × QR × 18. (4)
2 3 3 2
= 1:2 1
PO = 10 5 2 = 25 2 cm2
13. (1) Let the base and 2
perpendicular of right-angled 16. (4) All the angles of a rectangle
triangle be a and b, respectively. are of 90°, therefore a diagonal
divide a rectangle into two equal
a2 + b2 = 100
right-angled triangles.
1 1
and = 20 ab = 40 Length of diagonal h AD BC
2 Area of ABCD
= 2
= 4 2 32 Area of AED 1
(a + b)2 = a2 + b2 + 2ab AD h
=5m 2
= 100 + 2(40) = 180 1 1
17. (2) AD h BC h
14. (1) 2 2
1 1
AD h AD h
2 2
BC
=1+
AD
AD BC
AD

19. (3) Surface area of sphere


2

Let O be the centre of the in-


2 = 346.5
circle of PQR 346.5 7
Let the radius of the small cone r2
According to the question, PQ + be r and the larger cone be R. 4 22
r 5.25 cm
QR + PR = 24 cm

________________________________________________________
610
Subhash Institute of Competitive Exams (SICE) Vijayawada
Vijaya Ph: 9030924949
________________________________________________________
Mathematics for Competitive Exams ( 143 )
Section III – Chapter 12: Mensuration

20. (1) Let h be the height of 23. (3) Let the side be ‘x’ and the Thickness of gold = R–r
pyramid. diagonals be ‘a’ and ‘b’.
= 23 2 2
1 So, according to the question:
Then, 500 = × 30 × h = 2.518–2
3 a b m
0.518 cm
h = 50 m a b
2
m2 …… (i)
25. (4) Let the radius of the large
21. (1)
P solid sphere = r
Also, 4x 2P gives x
2
2 2 So,
2 a b
x 4 3
2 2 Volume of the sphere r
a2
b 2 3
x2
4 1 3
Volume of the cone r
3
P2 a2 b2
4 4 Thus,
P2 a2 b2 …… (ii)
The volume of new moulded
In the above figure, 3
Now, 1 r
small sphere = 1
AM = AC 2 MC 2 Area of rhombus 3 4 3

1
= 24 2 122 = a b Thus, radius of the new sphere
2
= 576 144 r
a b
2
a2 b2
= 1
= 432 1 4 3
=
2 2
As centroid divides median in According to question,
the ratio of to 2 : 1, [---(Putting values from (i) and (ii)]
Surface area of smaller sphere
432 12 3 Surface area of large sphere
So, OM = 1 m2 P 2
3 3 = 2
2 2 r
4 1
1 2 4 3 1
Area of remaining portion of = m P 2 sq unit
triangle = Area of equilateral 4 4 r2 2 4 /3
trangle – Area of circle 24. (4) So, the ratio of surface area of
smaller sphere with larger
3 22 12 3 12 3 Let the radius of the lead ball be
24 2 sphere is 1 : 24/3.
4 7 3 3 r and the gold foil be R.
249.41 150.86 26. (1)
According to the question,
98.55
Volume of gold = Volume of lead Volume of ice-cream
So, the area of remaining ball 2 22 1 22
73 72 7
portion of triangle is 98.55 sq. 3 7 3 7
cm. 4 4 2156 1078
R3 r3 r3 =
3 3 3 3
22. (1)
R3 – r3 = r3 = 1078 cubic cm
Lateral surface area Perimeter of base Height
Volume Area of base Height R3 = 2r3 27. (1) Let the breadth and the
length of the rectangle be 3x
120 Perimeter of base R3 = 2(2)3 and 4x, respectively
40 3 Area of base R3 = 16
3a Area = 1/12 hectare
3
3 a2 1 hectare = 10,000 m2
4
3 4 R 23 2
a 4 cm Area = × 10,000 m2
3 3 So,

________________________________________________________
611
Subhash Institute of Competitive Exams (SICE) Vijayawada
Vijaya Ph: 9030924949
________________________________________________________
Mathematics for Competitive Exams ( 144 )

1 (2 / 3)3 h = 2r
3x × 4x = × 10,000
12 Volume of water in the conical
= 8:27
10,000 glass = Volume of 32,000
x2 = 31. (4) Let r and h be the radius and spherical balls filled in the glass
144
the height of the cone,
x = 100/12 m respectively 4
r 2h = 32,000 ×
3
100 Then, = 154
Breadth = 3x = 3 ×
12 1 3
r2 20
= 25 m 22
4
28. (3) According to the question, r 2
= 49 × r2 r = 32,000 ×
3
Volume of cone before recasting r=7m 1 3
= Volume of cone after
Volume of conical tent = 1232 20
recasting
cu. m
Let h be the height of new cone =8
1 2
1 1 × h r = 2 cm
2
× 3.6 = (1.2)2 × h 3
3 3 = 1232 h = 2 × 2 = 4 cm
2.56 3.6 h = 24 m 33. (2) Volume of the spherical
h=
1.44 bullet
Slant height (l)
h = 6.4 cm = × (2.5)3 cu. dm
= 25 m
29. (1) Let the side of the square = Volume of the rectangular block
a Area of curved surface =
= 110 × 100 × 50 cu.dm
3 22 Number of bullets
Breadth of rectangle = × = × 7 × 25
2 7
=
side of square = = 550 sq. m

According to question, Length of canvas = 110 100 50 26400


= 4 =
3
Area of rectangle = 3 × area of
3
square
550 26400
3a = If > 3 than < 8800.
20 × = 3 × a2 2
2
= 275 m 34. (3) Since ABCD is a rhombus,
a = 10 cm therefore, its diagonals bisect
32. (4) The diameter of the
30. (3) Let r and R be the radii of each other at 90.
spherical drop = cm
sphere 1 and sphere 2,
respectively Radius of spherical drop =
1
r2 cm
20
R2
Volume of 32,000 spherical
4 r2
4 1 3 OA = OC = 6 cm; OB = OD = 8
9 R2 balls = 32,000 × ×
3 20 cm
r 2 cu. cm
R 3 (AB)2 = (OA)2 + (OB)2
Let h and r be the height and
3 the radius of the glass, (AB)2 = (6)2 + (8)2
= r respectively
R3 (AB)2 = 36 + 64

________________________________________________________
612
Subhash Institute of Competitive Exams (SICE) Vijayawada
Vijaya Ph: 9030924949
________________________________________________________
Mathematics for Competitive Exams ( 145 )
Section III – Chapter 12: Mensuration

3
AB = 100 4 r= 9261 cm
Now, 21 × 77 × 24 = ×
3
= 10 cm = 21 cm
22 3
35. (1) Volume of the rectangular r
7
block = Volume of the sphere
r 3 = 9261
Let r be the radius of the sphere

________________________________________________________
613
Subhash Institute of Competitive Exams (SICE) Vijayawada
Vijaya Ph: 9030924949
( 146 )
________________________________________________________
Mathematics for Competitive Exams

13 TRIGONOMETRY

TRIGONOMETRY
Trigonometry is a branch of mathematics that deals with the study of relationships between
sides and angles of a triangle.

MEASUREMENT OF ANGLES
Sexagesimal System
In this system, angles are measured in degrees, minutes and seconds. A right angle is divided
into 90 equal parts and each part is called a degree. 60 degree is denoted by 60 .
1 = 60 (60 minutes)
1 = 60 (60 seconds)

Circular System
In this system, an angle is measured in radians. A radian is the angle subtended at the centre
of a circle by an arc equal to the radius of that circle. One radian is denoted by 1 c.
radians or c = 180

Relation between Arc and Angle


If s is the length of an arc of circle with radius r, then the angle subtended by this arc at the
centre of circle is given by:

s
=
r

Or s = r

________________________________________________________
Subhash Institute of Competitive Exams (SICE) Vijayawada
Vijaya Ph: 9030924949
( 147 )
________________________________________________________
Mathematics for Competitive Exams Section III – Chapter 13: Trigonometry

TRIGONOMETRIC RATIOS
Trigonometric ratios of an acute angle in a right-angled
triangle provide the relationship between its sides and
angles. The longest side of a right-angled triangle is
called hypotenuse. Hypotenuse is always opposite to the
right angle. Let us consider a right angled triangle ABC,
right angled at B. In this triangle, AC is the hypotenuse;
AB, the side opposite to angle , is called the opposite The trigonometric ratios in this right angled ABC are as
side; and the third side BC is called the adjacent side. follows:

BC BC AB BC
sin = , cos = , tan = ,
AC AC AC AB

AC AC AB
cosec = , sec = and cot =
BC AB BC

Trigonometric Identities
cos2 + sin2 =1
1 + tan2 = sec2
1 + cot2 = cosec2
There are six trigonometric ratios in this right angled
Always remember that (sin )n = sinn , where n 1.
ABC as given below:
Similarly (cos )n = cosn . This rule is applicable for the
side opposite to AB remaining trigonometric functions also.
sin = =
hypotenuse AC
But (sin ) 1 sin 1 , which has entirely different
side adjacent to BC meaning. The same rule applies to the remaining
cos = =
hypotenuse AC
trigonometric functions.
side opposite to AB
tan = = Some Important Deductions
side adjacent to BC
sec2 tan2 =1
hypotenuse AC
cosec = =
side opposite to AB (sec + tan ) (sec tan ) = 1

hypotenuse AC 1
sec = = sec + tan = and sec tan
side adjacent to BC sec
1
side adjacent to BC =
cot = = sec
side opposite to AB
Similarly,
It can be observed that
1 1 1 1
cosec = , sec = , cot = cosec + cot = and cosec cot
sin cos tan cosec
1
Caution: The assignment of opposite side and adjacent =
cosec
side depends on the choice of the acute angle. If BAC =
, then

________________________________________________________
615
Subhash Institute of Competitive Exams (SICE) Vijayawada
Vijaya Ph: 9030924949
________________________________________________________
Mathematics for Competitive Exams ( 148 )

Values of Trigonometric ratios: Trigonometric ratios of (90 – ) in terms of for


all values of .
Angle 0° 30°or 45°or 60°or 90° or /2
sin (90 ) = cos
/6 /4 /3

Ratio cos (90 ) = sin

sin 0 1 1 3
1 tan (90 ) = cot
2 2 2

cosec (90 ) = sec


cos 1 3 1 1 0
2 2 sec (90 ) = cosec
2
tan 0 1 1 3 Not defined cot (90 ) = tan
3
Trigonometric ratios of (90 + ) in terms of for
cosec Not 2 1
2 2 all values of .
defined 3
sin (90 + ) = cos
sec 1 2 2 2 Not defined
3
cos (90 + ) = sin

cot Not 3 1 1 0 tan (90 + ) = cot


defined 3
cosec (90 + ) = sec

Trigonometric Ratios in Terms of ( ) sec (90 + ) = cosec


sin ( )= sin
cot (90 + ) = tan
cos ( ) = cos
Trigonometric ratios of (180 ) in terms of for
tan ( )= tan all values of .
sin (180 ) = sin
cosec ( ) = cosec
cos (180 ) = cos
sec ( ) = sec
tan (180 ) = tan
cot ( ) = cot
cosec (180 ) = cosec
Trigonometric Ratios in different Quadrants
sec (180 ) = sec

cot (180 ) = cot

Trigonometric ratios of (180 + ) in terms of for


all values of .
sin (180 + ) = sin

cos (180 + ) = cos

tan (180 + ) = tan

cosec (180 + ) = cosec

sec (180 + ) = sec

cot (180 + ) = cot

________________________________________________________
616
Subhash Institute of Competitive Exams (SICE) Vijayawada
Vijaya Ph: 9030924949
________________________________________________________
Mathematics for Competitive Exams ( 149 )
Section III – Chapter 13: Trigonometry

Trigonometric ratios of (270 ) in terms of for MAXIMUM AND MINIMUM VALUE OF


all values of . TRIGONOMETRIC FUNCTIONS
sin (270 ) = cos
Maximum value of sin =1
cos (270 ) = sin
Maximum value of cos =1
tan (270 ) = cot
Minimum value of sin = 1
cosec (270 ) = sec Minimum value of cos = 1
sec (270 ) = cosec tan and cot can be any real number value.
cot (270 ) = tan sec and cosec cannot have any value between
1 and 1.
Trigonometric ratios of (270 + ) in terms of for
all values of . Maximum value of a sin + b cos = a2 b2

sin (270 + ) = cos Minimum value of a sin + b cos = a2 b2

cos (270 + ) = sin


Use of AM-GM inequality
tan (270 + ) = cot a tan
a tan (by AM-GM inequality)
2
cosec (270 + ) = sec
Minimum value of a tan + b cot = 2 ab
sec (270 + ) = cosec

cot (270 + ) = tan Sine Rule


In any triangle ABC, where a, b and c are the sides of a
Trigonometric ratios of (360 ) in terms of for triangle opposite to angles A, B and C, respectively and R
all values of . is the circumradius of triangle. We have:
sin (360 ) = sin
a b c
2R
cos (360 ) = cos sin A sin B sin C

tan (360 ) = tan Cosine Rule


cosec (360 ) = cosec In any triangle ABC, where a, b and c are the sides of a
triangle opposite to angles A, B and C, respectively.
sec (360 ) = sec
b 2 c 2 a2
cot (360 ) = cot cos A =
2bc

Periodic Formulae c2 a2 b 2
cos B =
For a positive integer n 2ca

sin (2 n + ) = sin a2 b 2 c 2
cos C =
2ab
cos (2 n + ) = cos

tan ( n + ) = tan APPLICATIONS OF TRIGONOMETRY


cosec (2 n + ) = cosec Trigonometry is used to find height and distances which
sec (2 n + ) = sec cannot be measured by ordinary means. It can be used
to find height of tower, height of cloud, inclination of a
cot ( n + ) = cot
mountain, width of river etc. Let us review some basic
Here, means angle of radian which is equal to 180 . terms required to solve height and distance problems.

________________________________________________________
617
Subhash Institute of Competitive Exams (SICE) Vijayawada
Vijaya Ph: 9030924949
( 150 )
________________________________________________________
Mathematics for Competitive Exams

Angle of Elevation By squaring both sides and then adding them, we


This is the angle formed between the horizontal plane get
and the line of sight and when you look upward at an sin2 + cos2 + 2sin cos + sin2 + cos2
object. 289
– 2sin cos = x2 +
169
289
x2 = 2 –
169
7
x=
13
3. Find the value of sin5 sec40 sin50 sec85 .
Solution: sin5 sec40 sin50 sec85
Angle of Depression = sin5 sec40 sin (90 – 40 ) sec (90 – 5 )
This is the angle formed between the horizontal line and = sin5 sec40 cos40 cosec5
the observer’s line of sight, when you look down at an
=1
object.
1
4. What will be the value of sin if
sin
1
sin2 ?
sin2
Solution: We know,
2
1
sin
sin
1 1
sin2 2 sin ,
sin2 sin
2 2 4
Some Important Examples: Therefore,
1. If cos + sec = 2, then find the value of 1
sin 2
cos400 + sec400 . sin
Solution: cos + sec =2 5. 2 2 4

1 Solution: 2
cos + =2
cos – cos2 2

cos2 + 1 = 2cos So, sin2 4 2 2 = cos2 2

(cos 1)2 = 0
cos = 1 = cos90 6. The minimum value of 2 Sin 2 + 3 Cos2 is
2 2 2 2
= 90 Solution: 2sin + 3cos = 2 – 2 cos + 3cos

cos400 + sec400 = (cos90 )400 + (sec90 )400 = = 2 +cos2 , (cos2


1+1=2 So minimum value = 2 + 0 = 2
2. Find the value of sin – cos , if sin + cos 7. If sinA + cosA = 0, then tanA + cotA = ?
17 Solution:
= .
13 sinA cos A 0
Solution: Let sin – cos = x and sin + cos sinA cos A
17 sinA
= 1
13 cos A
tanA 1

________________________________________________________
618
Subhash Institute of Competitive Exams (SICE) Vijayawada
Vijaya Ph: 9030924949
________________________________________________________
Mathematics for Competitive Exams ( 151 )
Section III – Chapter 13: Trigonometry

1 1 H H
cot A 1 tan60
tanA 1 BC 10 3
Now,
tan A cot A H
3
10 3
1 1 2
H = 30 m
8. If tan . tan 2 = 1, then the value of sin22 +
10. A 10 m long ladder is placed against a wall. It is
tan22 is equal to
inclined at an angle of 30° to the ground. The
Solution: tan . tan 2 =1 distance (in m) of the foot of the ladder from the
1 wall is (Given )
tan = = cot 2
Solution:
tan = tan (90° – 2 )
= 90° – 2
= 30°
2
3 2 3 3
sin2 60°+ tan2 60° = + 3 =3+ = 3
2 4 4 PR = ladder = 10 meter
9. The angle of depression from the top of a tower to QR =distance of the foot of the ladder from the wall
a man standing at 10 3 m away from its foot is PRQ = 30°
60 degrees. What is the height of the tower? From PQR
Solution: cos 30° = QR / PR = QR / 10

QR = 5
= 8.660 meter

From the figure,

________________________________________________________
619
Subhash Institute of Competitive Exams (SICE) Vijayawada
Vijaya Ph: 9030924949
( 152 )
________________________________________________________
Mathematics for Competitive Exams

PRACTICE QUESTIONS
Answer the following questions by selecting the most appropriate option.

1. For any value of from 0 to 90 , if A = cos2 + 9. Which of the following is true if A = tan26 tan
sin4 , then 62 and B = 2cot64 cot 28 ?
(1) A > 1 (2) 0 < A < 1 (1) A = B (2) B = 2A
(3) A (4) 0 < A B
(3) A = (4) A = –B
2
2. For 0 , if 2sin + 2cosec = 4, then
find out the value of sin5 + cos5 . 10. If cos2 a + cos2 b = 2, then find the value of tan5 a
2 + sec7 b.
(1) 1 (2) (1) 1 (2) 0
8
9 3 1 3 1 (3) 2 (4) Not defined
(3) (4)
32 2 11. If is an acute angle and 9cos2 + 5sin2 = 6,
3. Find out the value of sin2 10 + sin2 20 + sin2 30 then find the value of cot .
(1) 1 (2) 0
+ … + sin2 80 + sin2 90 .
1 1
2 3 1 (3) (4)
(1) 2 + 3 (2) 2 2
2
3 1 12. Find the value of cos21 + cos25 + cos29 + … +
(3) 5 (4)
4 cos285 + cos289 .
45 23
4. Find the value of tan 5 , if tan 4 × tan 6 = 1. (1) (2)
2 2
1
(1) (2) 1 37 59
3 (3) (4)
2 2
2 3 5
(3) (4)
5 2 13. Simplify: cot 22 (cot 68 cos2 18 +
1
5. For 0 90 , if sin a + cos b = 2 then find )
tan 68 sec 2 72
2a-b
the value of sin . 4
2 (1) (2) 0
3
1 1
(1) 1 (2) (3) 1 (4) 4
2 3
1
(3) (4) 0 14. What is the value of 2 cos – sin , if 2sin +
2
3
6. If sin + cosec = 2, then find the value of cos = ? (where 0 < < 90 )
2
sin400 + cosec400 . (1) 0 (2) 1
(1) 1 (2) 2 1
(3) 3 2 (4) 0 (3) 2 (4)
1
2
sin52°
7. What is the value of + 2tan12 tan32
cos38° sin cos
15. If , then (sin2 cos2 ) is equal to
tan45 tan58 tan78 – 4(sin235 + sin255 )? sin cos
(1) 1 (2) 0 2
(3) –1 (4) 2 (1)
5
sin2 1
8. Find the value of , if = 3 and 0 < (2)
tan2 – sin2 5
. 4
(3)
(1) 90 (2) 45 5
(3) 60 (4) 30 3
(4)
5

________________________________________________________
620
Subhash Institute of Competitive Exams (SICE) Vijayawada
Vijaya Ph: 9030924949
________________________________________________________
Mathematics for Competitive Exams ( 153 )
Section III – Chapter 13: Trigonometry

16. If sin x + sin y = 0, 0 x 90 and 0 y 90 , 24. Find the value of a sin + b cos , if a cos –b
then find cos x + cos y. sin = c.
1 (1) ± a2 b 2 c 2 (2) ± a 2 b 2 c 2
(1) 2 (2)
2 (3) ± a 2 b 2 c 2 (4) ± a 2 b 2 c 2
1
(3) (4) 0 cosec + cot
2 25. Find cos if, .
cosec – cot
17. If tan = 1 then is equal 1 3
(1) (2)
cos2 – 2sin3 7 sin 2 4
to 1
13 11 (3) (4) 1
(1) (2) 4
8 6
(3) 2 (4) –1 26. If sec + tan = 1 + , then cos is equal to
1
18. If is an acute angle and sin2 + sin4 = 1, (1) 3 (2)
3
then find the value of cot2 + cot4 .
4
(1) 4 2 (2) 8+2 2 (3) (4) Not defined
1 3 3
(3) 0 (4) 1
19. Find the value of sin3 sec42 sin48 sec87 . 27. What is the value of (cos2 35 + cos2 55 )?
(1) 0 (2) 1 (1) 0 (2) 1
1
1
(3) (4) 2 2
3 (3) 64 2 (4) Not defined

3 28. Find the value of A, if sin (A + B) = cos (A – B) =


20. If tan = , then find the value of
4 3
and (A + B) is an acute angle. Also A > B > 0 .
4 sin – 2 cos 2
4 sin – 2 cos (1) 15 (2) 30
2 (3) 45 (4) 60
(1) 5 (2)
5 29. Find the maximum value of (3sin + 2cos ).
5 1 (1) 17 (2) 2 3
(3) (4)
2 5 (3) 2 2 (4) 13
21. If sin + cosec = 2, then find the value sin100 1 1
+ cosec100 . 30. Find the value of x if x = +
1 sec 2 y 1 cos 2
(1) 0 (2) 1
(1) 0 (2) 1
(3) 2 (4) –1
1
(3) (4) 1
sin 2
22. Which of the following is equal to +
1 – cosec
31. If 0° < A < 90 , the simplified form of the given
cosec expression Sin A Cos A (Tan A – Cot A) is
?
1 – sin (1) 1 Cos2 A (2) 1 – 2 Sin2 A
(1) sin + cosec –1 (3) 2Sin A 1
2 (4) 1
(2) sin + cosec +1
(3) sin – cosec +1 32. The angle of elevation of a tower from a distance
(4) sin – cosec –1 of 100 m from its foot is 30°. The height of tower
is
1 (1) 50/ (2) 50
23. If cosec = a + and (0 < < 90 ) then
4a (3) 100/ (4) 100
(cosec + cot ) is equal to
33.
(1) 2a
the value of Cos2
(2) a
(1) 1/ (m2 + n2)
1
(3) (2) m2/(m2 + n2)
a (3) n2/(m2 + n2)
1 (4) 0
(4)
2a

________________________________________________________
621
Subhash Institute of Competitive Exams (SICE) Vijayawada
Vijaya Ph: 9030924949
________________________________________________________
Mathematics for Competitive Exams ( 154 )

34. 2 2 4 38. If 5 sin = 3, the numerical value of


(1) None (2) sin /cos2 sec q – tan q
(3) cos2 /sin (4) 1 sec q + tan q
35. A 10 m long ladder is placed against a wall. It is 1 1
(1) (2)
inclined at an angle of 30° to the ground. The 3 2
distance (in m) of the foot of the ladder from the 1 1
(3) (4)
wall is (Given 4 5
(1) 7.32 (2) 8.16
(3) 8.66 (4) 8.26 39. A kite is flying at a height of 75m from the
ground. The string makes an angle (where Cot =
36. The value of Sin222 + Sin268 + Cot230 is 8/15 ) with the level ground. Assuming that there
(1) 3/4 (2) 4 is no slack in the string, the length of the string is
(3) 5/4 (4) 3 equal to
(1) 75m (2) 85m
37. If be acute angle and tan ( 4 50 ) = cot (3) 40m (4) 65m
(50 ), then the value of in degree is:
(1) 30 (2) 40 40. If Sec + Tan = p, (p 0) then Sec is equal to
(3) 20 (4) 50 (1) (p + 1/p), p 0 (2) ½ (p + 1/p), p 0
(3) 2(p 1/p), p 0 (4) (p 1/p), p 0

________________________________________________________
622
Subhash Institute of Competitive Exams (SICE) Vijayawada
Vijaya Ph: 9030924949
( 155 )
________________________________________________________
Mathematics for Competitive Exams Section III – Chapter 13: Trigonometry

ANSWERS AND EXPLANATIONS


1. (4) In A = cos2 + sin4 , the tan 4 = cot 6 8. (4)
power of cos and sin are
tan 4 = tan (90 – 6 ) sin2
=3
even, so A is always greater
tan2 sin 2
than 0. 4 = 90 – 6
At =0 ,A=1+0=1 sin2
=9 =3
1
sin2 1
3 1 13 tan 5 = tan 5(9 ) = tan 45 cos 2
At = 30 , A =
4 16 16 =1
cos 2
1 1 3 3
At = 45 , A = 5. (1) sin a + cos b = 2 1 cos2
2 4 4
sin a cot =  (Since 0 <
1 9 13
At = 60 , A = a = 90 and b = 0 < 90 )
4 16 16

At = 90 , A = 0 + 1 = 1 2a-b cot = cot 30


2 2 = 30
0<A
2. (1) 2sin + 2cosec =4 9. (2) A = tan26 tan62
=1
sin + cosec =2 B = 2cot 64 cot 28 = 2cot
6. (2) sin + cosec =2
(90 – 26 ) cot (90 – 62 ) = 2
1
sin + =2 1 tan 26 tan 62 = 2A
sin sin + =2
sin 10. (1) cos2 a + cos2 b = 2
sin =1
sin2 + 1 = 2sin
a=b=0
= 90
(sin 1)2 =0 tan5 a + sec7 b = tan5 (0 ) +
sin5 + cos5 = (sin 90 )5 + sec7 (0 ) = 05 + 17 = 1
sin = 1 = sin90
(cos 90 )5 = 1 + 0 = 1
= 90 11. (1) 9cos2 + 5sin2 =6
3. (3) sin210 + sin220 + sin230
+ … + sin280 + sin290 sin400 + cosec400 = 5cos2 + 5sin2 + 4
(sin90 )400 + (cosec90 )400 cos2 =6
= sin210 + sin220 + sin230 +
=1+1=2 5 + 4 cos2 =6
sin240 + sin250 + sin260 +
sin270 + sin280 + sin290 sin52° 4 cos2 =1
7. (3) + 2tan12 tan32
cos38°
= sin210 + sin220 + sin230 + 1
tan45 tan58 tan78 – cos =
sin2 40 + sin2 (90 – 40 ) + 2
4(sin235 + sin255 )
sin2 (90 – 30 ) + sin2 (90 –
20 ) + sin2 (90 – 10 ) + sin2 sin52° = 45
= + 2tan12
90 cot = cot 45 = 1
tan32 tan45 tan(90 – 32 )
= sin210 + sin220 + sin230 + 12. (2) cos21 + cos25 + cos29 +
tan(90 – 12 ) – 4(sin235 +
sin240 + cos240 + cos230 + … + cos285 + cos289
sin2(90 35 ))
cos220 + cos210 + sin290
sin52°
In the given expression we can
=4+1=5 = + 2tan12 tan32 see that the angles are 1 , 5 ,
4. (2) tan 4 × tan 6 =1 tan45 cot32 cot12 ) – 9 , 13 , …, 81 , 85 , 89

1
4(sin235 + cos235 ) Let the number of terms in the
tan 4 = expression be n
tan6 = 1 + 2 – 4 = –1
a + (n – 1)d = tn
1 + (n – 1)4 = 89

________________________________________________________
623
Subhash Institute of Competitive Exams (SICE) Vijayawada
Vijaya Ph: 9030924949
( 156 )
________________________________________________________
Mathematics for Competitive Exams

n = 23 sin + cos = 3sin 4sin c


20. (1)
3cos 4sin cos
cos21 + cos25 + cos29 +
… + cos285 + cos289 4cos = 2sin On dividing both numerator and
denominator by cos , we get
= cos21 + cos289 + cos25 + tan =2
cos285 + … + cos237 + 4tan 2 3 2
sin2 cos2 = =5
cos253 + cos241 + cos249 + 4tan 2 3 2
cos2 (tan2 1)
cos245 21. (3) sin + cosec =2
= cos21 + sin21 + cos25 + tan2 1
= sin + =2
tan2 1
sin25 + … + cos241 + sin241
+ cos245 22 1 3 sin2 + 1 = 2sin
= =
= [1 + 1 + 1 + … + 1](11 times) + 22 1 5
(sin 1)2 = 0
2
1 4
sec4 – tan4 = sin =1
2 15
16. (1) sin x + sin y = 0 = 90
23
= sin100 + cosec100 = (sin
2 x=y=0
90 )100 + (cosec 90 )100
13. (3) cot 22 (cot 68 cos218 + cos x + cos y = cos0 + cos0
=1+1=2
1 =1+1=2
)
tan68 sec2 72 sin
17. (3) tan = 1 = tan45 22. (2)
= cot 22 cot 68 cos218 + 1 cosec
cot 22
= 45
1
tan68 sec2 72 8 cos sin sin
= =
cos 3 3 1 1 sin
= cot 22 tan 22 cos218 + 1
8 5 sin
cot 22
cot 22 sec2 72 2 2 sin2
1
3
1
3
7 =
2 sin
= cos218 + cos272 2 2 2
= sin272 + cos272 = 1 sin2
13 13 =
sin
3 = 2 = 2
14. (4) 2sin + cos = and let 1 1 7 1 2 14
2 sin3
=
2 2 2 2 2 2 sin
2 cos sin be x
=2
On squaring both sides and (sin 2

18. (4) sin2 + sin4 =1 =


then adding them, we get sin

4sin2 + cos2 + 4sin cos sin4 = 1 – sin2 = cos2


= sin + cosec +1
+ 4 cos2 + sin2 cos 2
9 sin2 = 23. (1) cosec =a+
4sin cos = + x2 sin2
2
cot2 = sin2 cot = cosec2 – 1 =
9
5(sin2 + cos2 )= + x2 cot2 + cot4 = sin2 +
2 1 1
sin4 =1 a2 1
16a2 2
9
x2 = 5 – 19. (2) sin3 sec42 sin48 sec87
2 1 1
= a2 =a–
1 = sin3 sec42 sin (90 – 42 ) 16a2 2
x= ( 0 < < 90 )
2 sec (90 – 3 )
(cosec + cot )=a+ +a
sin cos = sin3 sec42 cos42 cosec3
15. (4) 3 – = 2a
sin cos =1

________________________________________________________
624
Subhash Institute of Competitive Exams (SICE) Vijayawada
Vijaya Ph: 9030924949
________________________________________________________
Mathematics for Competitive Exams ( 157 )
Section III – Chapter 13: Trigonometry

24. (2) Let a sin + b cos =x On solving (1) and (2), we get cos cos
and a cos – b sin = c A = 45 and B = 15
On squaring and adding both 29. (4) Maximum value of (a sin + on squaring both sides
the equations, we get b cos )= a 2
b 2

a2 sin2 + b2 cos2 + 2ab Maximum value of 3sin + 2 2 2 2


sin cos + a2cos2 +
2cos = 2 2
b2sin2 – 2ab sin cos = x2
2 2 2 2 2
+ c2 1
30. (2) sec y
a2 (sin2 + cos2 ) + b2 (sin2 cos y 2 2 2 2

+ cos2 ) = x2 + c2 Therefore,
2 n2
x2 = a2 + b2 – c2 1 1 2 2
x
1 cos2 y 1
1
x = ± a2 b 2 c 2 cos 2 y 34. (4) Sin 2

cosec cot 5 1 cos2 y – sin2 2


25. (3) x 1
cosec cot 3 1 cos2 y 1 cos2 y So, cos2 4 2

3cosec + 3cot = (cos2 2


31. (3) sinA. cosA (tanA cotA)
5cosec – 5cot
= cos2 2 = cos2
sinA cos A
8cot = 2cosec = sinA.cosA sin2
cos A sinA
cot 1 35. (3)
cosec 4 sin2 A cos 2 A
= sinA.cosA
sin A.cos A
1
cos = = sin2 A – cos 2 A
4
= 2 2

26. (3) sec + tan = 1 + 3


…(1) = sin2A 1 + sin2A
PR = ladder = 10 meter
sec – tan = = 2sin2A 1
1 1- 3 3-1
QR =distance of the foot of the
= = (2) 32. (3) From triangle PQR, ladder from the wall
1+ 3 1+ 3 1- 3 2
PQ PRQ = 30°
tan30 =
On solving (1) and (2), we get QR
From PQR
1 3 3
sec = cos 30° = QR / PR = QR / 10
4

4
cos =
1 3 3 QR = 5
27. (2) (cos2 35 + cos2 55 ) = 8.660 meter
= cos2 35 + cos2 (90 – 35 ) 36. (2) sin2 22 + sin2 68 + cot2
1 PQ
30
= cos2 35 + sin2 35 = 1 3 100
= sin2 22 + sin2 (90 22 ) +
100
28. (3) sin(A + B) = = sin60 PQ = meter ( 2
3
(A + B) = 60 -(1) = sin2 22 + cos2 22 + 3
33. (3)
=1+3=4
cos(A –B) = = cos 30 cos cos
n and m
sin cos 37. (1) tan(4 50 ) = cot(50 )
(A – B) = 30 …(2)
tan(90 ) = cot

________________________________________________________
625
Subhash Institute of Competitive Exams (SICE) Vijayawada
Vijaya Ph: 9030924949
________________________________________________________
Mathematics for Competitive Exams ( 158 )

tan (4 50 ) = tan {90 3 PR = 75cosec = 75


1
(50 )} 1 sin 5 cot2 )
1 sin 3
(4 50 ) = {90 (50 )} 1 = 75 2
5
5 3 2 1 = 75 × (17/15) = 85 m
4 50 = 90 50 +
5 3 8 4
3 = 90 40. (2) Sec + tan = p, ( p 0 )
39. (2) ………(i),
= 90 / 3 = 30
sec2 – tan2 =1
38. (3) 5 sin = 3,
(sec + tan ) (sec – tan )
sin = 3/5 =1
sec – tan p(sec – tan ) = 1
Expression
sec tan
P = Location of kite (sec – tan ) = 1/p ……….(ii)
1 sin
cos cos PR = Length of string on adding equation (i) and (ii)
= 1 sin
PQ = 75 meter 1
cos cos 2sec =p+
p
In
1 1
PR sec = p ,p
= cosec 2 p
75

________________________________________________________
626
Subhash Institute of Competitive Exams (SICE) Vijayawada
Vijaya Ph: 9030924949
( 159 )
________________________________________________________
Mathematics for Competitive Exams

14 STATISTICS & PROBABILITY

STATISTICS
Statistics offers certain measures of central tendency or summary measures that attempt to
describe a whole set of data with a single value that represents the middle or the centre of its
distribution. There are three main measures of central tendency: the mode, the median and
the mean. Each of these measures describes a different indication of the typical or central
value in the distribution.

Mean
Referred as arithmetic mean sometimes, the mean is the average of all numbers of a
particular set. To calculate the mean, we need to add together all of the numbers in a set and
then divide the total sum by total number of values.
Sum of observations
Mean =
Number observations

Ex. In the set, (100, 98, 105, 90 and 102) the mean will be calculated by,
100 98 105 90 102 495
Mean = 99
5 5

Mode
The mode is the number with the highest frequency within a given set of numbers. Consider
the set, (90, 104, 98, 98, 105, 92, 102, 100, 110, 98, 210 and 115). Here, the number 98
occurs three times and hence it is the mode of the distribution.

Median
Median is the central value of the distribution such that the number of values less than the
median is equal to the number greater than the median. The median can be easily calculated
by sorting the data from the smallest to the largest and counting the middle value.
Ex. Find the median of the following observations in a data set: 5, 7, 6, 1, 8, 10, 12, 4, and 3.
Arranging the data, in ascending order, we get 1, 3, 4, 5, 6, 7, 8, 10, 12.
Now, since the “middle score” is 6, it is the median.

________________________________________________________
Subhash Institute of Competitive Exams (SICE) Vijayawada
Vijaya Ph: 9030924949
( 160 )
________________________________________________________
Mathematics for Competitive Exams

Note: If the total number of elements in a set is even (iii) The A.M. of these differences known as deviations is
then there will be two observations which fall in the the Mean Deviation of the respective set of data.
middle. The median in this case is computed as the d
Mean Deviation =
arithmetic mean of the two middle values. n

The position of the median can be calculated using Ex. Calculate the Mean Deviation of the following
the formula: observations; 2, 4, 7, 8 and 9.
2 4 7 8 9
N 1 We know the A.M of this data will be
Position of median = 5
2
=6
where N = number of items.
Now, calculate the deviations,
Suppose the Arithmetic Mean, Median and Mode of
a particular set of observations are AM, M i and Mo, X |d|
then the relative magnitude of the three is 2 4
represented as, 4 2
AM < Mi < Mo or AM > Mi >Mo 7 1
This implies that the median is always between the 8 2
arithmetic mean and the mode.
9 3

Dispersion: d = 12
Dispersion is the extent to which values in a distribution
d 12
differ from the average of the distribution. The Putting this in the formula, Mean Deviation = =
n 5
distributions that we need to know for the RRB NTPC
= 2.4
exams include Range, Mean Deviation and Standard
Deviation. Standard Deviation
Standard Deviation is the positive square root of the
Range
mean of squared deviations from the mean. The mean of
The range is the difference between the highest (H) and
these squared deviations is called the variance and the
lowest (L) values within a set of numbers. To calculate
positive square root of the variance is the standard
range, subtract the smallest number from the largest
number in the set.
calculated by following these simple steps:
Range = H – L
Steps:
Mean Deviation: (i) Calculate the A.M. of the values in the set.
Mean Deviation is the arithmetic mean of the differences (ii) Now, calculate the deviations of the given values
of the values from their average. The average used is from mean.
either the arithmetic mean or median of the respective (iii) Square these deviations now and calculate the sum
set. Mean Deviation can be calculated by following these of these squares.
simple steps: (iv) Insert the calculated sum in the formula to get the
standard deviation.
Steps:
(i) Calculate the A.M. of the values in the set d2
=
(ii) Now, calculate the difference between each value n

and the A.M. Keep in mind that all differences will be Ex. Calculate the standard deviation of the following
considered positive, denoted as |d|. values:
5, 10, 25, 30, 50

________________________________________________________
628
Subhash Institute of Competitive Exams (SICE) Vijayawada
Vijaya Ph: 9030924949
________________________________________________________
Mathematics for Competitive Exams ( 161 )
Section III – Chapter 14: Statistics & Probability

We know the A.M of this data will be events will be 1. Therefore, the required probability will
5 10 25 30 50 be 1/6.
= 24
5
The following are two very important rules of probability:
X d(deviation) d2
AND rule: Suppose the probability of event A is P(A) and
5 -19 361
that of event B is P(B). Now, if these two events are to
10 -14 196
occur together, i.e. if we want to calculate the probability
25 1 1 of occurrence of event A and that of event B, we need to
30 6 36 multiply the individual probabilities of A and B.
50 26 676
Therefore, the probability of occurrence of A and B will be
2= 1270 P(A) × P(B).

d2 Illustration: If the probability of A passing an exam is


= 254
n 1/4 and that of B passing the same exam is 2/3, what is
= 15.937 the probability that both A and B will pass the exam?
Note: Standard Deviation is Independent of Origin. Solution: The required probability is 1/4 × 2/3 = 2/12 =
1/6. The individual probabilities of the two events have
PROBABILITY been multiplied using the AND rule as asked in the
question, the probability of occurrence of both the events
Probability is one of the most important mathematical
together was asked.
concepts having wide implications in different real-life
scenarios. OR rule: When the probabilities of two events are given
and we need to calculate the probability that at least one
Probability simply refers to the chance of occurrence of
of them occurs, we need to add the individual
an event. In other words, probability is a likelihood that
probabilities of the two events. Therefore, if the
something (defined as an ‘event’ in the language of
probability of event A is P(A) and that of event B is P(B),
mathematics) may or may not occur. For example, one
then the probability that at least one of the two events
might say that India has a 50% probability of winning
occurs is P(A) + P(B).
cricket matches against Australia. This statement means
that India is likely to win 1 out of every 2 matches played Illustration: Suppose the probability of horse-1 winning a
against Australia. race is 1/2 and that of horse-2 winning the same race is
3/5. What is the probability that either horse-1 or horse-
In questions related to probability, you need to ask
2 wins the race?
yourself “Probability of what?” In other words, you need
to first identify the event, the occurrence of which needs Solution: The required probability is 1/2 + 1/3 = 5/6
to be determined.
SMART TIP
In the simplest form, probability of an event is calculated
You can solve a probability-related
on the basis of the total number of cases, i.e. total question by following two simple
number of probable events, and the total number of steps:
favourable cases, i.e. total number of occurrences of
STEP 1: Find out the probability of
events, for which we are determining the probability. For individual events by determining the
example, suppose 6 balls of different colours are in a total number of cases and total
bag. Now, suppose one ball is to be randomly picked number of favourable cases.
from the bag. If we want to calculate the probability of The probability of the favourable
picking a ball with a particular colour, the total number of event = Total number of favourable
probable events will be 6 and the number of favourable cases/Total number of cases

________________________________________________________
629
Subhash Institute of Competitive Exams (SICE) Vijayawada
Vijaya Ph: 9030924949
( 162 )
________________________________________________________
Mathematics for Competitive Exams

STEP 2: Find out the AND or OR If A and B are dependent events, P(A B) = P(A) ×
relationship of the individual events
B A
to find out the final probability. P P B P
A B

Mutually Exclusive Events


The events that cannot happen at the same time are
Conditional probability
mutually exclusive events. A
When A and B are dependent events, then P
B
Ex: Occurrence of head and tail in the toss of a coin are denotes the probability of the happening of event A when
two mutually exclusive events, as we can get either head
B
or tail. B has already occurred. Similarly, P denotes the
A
For mutually exclusive events, P(A or B) = P(A) + P(B) probability of the happening of B when A has already
Illustration: There are 3 blue marbles, 5 white marbles occurred.
and 10 green marbles in a box. If one marble is picked at
Important tips
random, what is the probability of getting a blue marble
For Mutually exclusive events and non-mutually
or a green marble?
exclusive independent events, we must use the
Solution: concept of AND and OR as discussed in
Total number of marbles = 3 + 5 + 10 = 18 Permutations and Combinations.
In conditional probability questions (that is, non-
Number of possible outcomes = 18
mutually exclusive dependent events), most of
Let A be the event of getting a blue marble and B be the the questions are framed in a manner in which
event of getting a green marble. The event of getting a
A B
blue marble or a green marble are mutually exclusive; P is given and P is to be calculated. So,
B A
therefore,
A
Required probability = P(A or B) = P(A) + P(B) we first find P(A B) by using P in such cases
B
3 10 13 B
= and then use it to find P .
18 18 18 A
For non-mutually exclusive events, P(A or B) = P(A) + P(B)
P(A and B)
Illustration: Two trains P and Q are scheduled to reach
Probability of the occurrence of events A and B, P(A and New Delhi railway station at 10.00 AM. The probability
B) = P(A) × P(B) that train P and train Q will be late is 7/9 and 11/27
respectively. The probability that train Q will be late,
Independent events
given that train P is late, is 8/9. Find the probability that
If happening of one event (say A) does not affect the
neither train will be late on a particular day.
happening of another event (say B), then A and B are
said to be independent events. Solution

Dependent events Probability that train P will be late


If happening of one event (say A) affects the happening 7
of another event (say B), then A and B are dependent = P(P) =
9
events. Probability that train Q will be late
If A and B are independent events, P(A B) = P(A) × P(B) 11
= P(Q) =
27

________________________________________________________
630
Subhash Institute of Competitive Exams (SICE) Vijayawada
Vijaya Ph: 9030924949
________________________________________________________
Mathematics for Competitive Exams ( 163 )
Section III – Chapter 14: Statistics & Probability

Probability of Q will be late in case P is late data interpretation, with conditional probability 0.15. An
applicant surveyed is found to be strong in data
Q P(P Q)
P interpretation. Find the probability that the applicant is
P P(P)
also strong in quantitative aptitude.
[Using conditional probability]
Solution
7 8 56
P(P Q) PP Q
9 9 81 Using Baye’s Theorem :
Probability of either P or Q or both being late P(LR) = 0.8
P(P) P(Q) P(P Q) P(QA) = 0.2
7 11 56 40 DI
P 0.87
9 27 81 81 QA
Hence, the probability that neither of the trains will DI
P 0.15
40 41 LR
be late 1
81 81 Therefore probability that an applicant is good in
Data interpretation given that he is good in
Bayes' Theorem Quantitative aptitude
Bayes' theorem is a direct application of conditional
probabilities. This theorem is used to find the conditional DI
P(QA) P
DI QA
probability of an event P(A | B), say, when the "reverse" P
LR DI DI
conditional probability P(B | A) is the probability that is P(QA) P P(LR) P
QA LR
known.
DI 0.2 0.87
Let A1, A2, ... , An be a set of mutually exclusive events P 0.6
LR 0.2 0.87 0.8 0.15
that together form the sample space S. Let B be any
event from the same sample space, such that P(B) > 0. Points to Remember
Then, Total number of outcomes in a toss of 2 coins = 2 2
=4
P( Ak B )
P Ak |B
P A1 P B|A1 P A 2 P B|A 2 P An P B|An Total number of outcomes in a toss of n coins = 2n
Total number of outcomes in a throw of 2 dices = 6 2
Note: Invoking the fact that P(Ak B ) = P(Ak )P(B | Ak ),
= 36
Baye's theorem can also be expressed as
Total number of outcomes in a throw of n dices = 6n
P A k P B|A k
P A k |B
P A 1 P B|A 1 P A 2 P B|A 2 P A n P B|A n Some Important Examples:
1. How many words with or without meaning can be
Illustration: A survey was conducted to test relative
formed by using the letters of the word ‘MIXTURE’
aptitudes in quantitative and logical reasoning of B.Tech so that the vowels are never together?
applicants. It is perceived (prior to the survey) that 80
Solution: There are 7 distinct letters in the word
percent of B.Tech applicants are extremely good in
MIXTURE. Total number of words which can be
logical reasoning, while only 20 percent are extremely formed by using these letters = 7! = 5040
good in quantitative aptitude. Further, it is believed that
Number of words in which vowels are together = 5! ×
those with strong quantitative knowledge are also sound 3! = 720
in data interpretation, with conditional probability as high
Required number of the words = 5040 – 720
as 0.87. However, some B.Tech applicants who are
= 4320
extremely good in logical reasoning can be also good in

________________________________________________________
631
Subhash Institute of Competitive Exams (SICE) Vijayawada
Vijaya Ph: 9030924949
( 164 )
________________________________________________________
Mathematics for Competitive Exams

2. In how many ways can a cricket team of 11 players Number of ways in which vowels come together = 6!
be selected out of 14 players if 4 particular players × 2! = 1440
are always included? Number of ways in which vowels do not come
Solution: Since 4 players are certain in the team together = 5040 – 1440 = 3600
therefore selection of 7 players is required from the 6. Hemant speaks the truth in 90% cases while Anil
remaining 10 players. in 80% cases. Find the probability that at least one
10 9 8 7! is lying.
Required number of ways = 10
C7 =
7! 3! Solution: P(none is lying) = 0.90 × 0.80 = 0.72
= 120
P(at least one lying) = 1 – P(none is lying)
3. A polygon has 35 diagonals. The number of sides
= 1 – 0.72 = 0.28
in this polygon is
7. A box contains balloons of three colours. There
Solution: Number of diagonals in a polygon of n
were 7 red, 8 yellow and 5 green balloons in that
n n 3
sides = box. Three balloons are drawn randomly. What is
2
the probability that neither of the balloons drawn
nn 3 is green?
= 35
2 Solution: The probability that neither of the balloons
n2 – 3n – 70 = 0 drawn is green is,
(n + 7)(n – 10) = 0 15
C3 15 14 13 91
20
n = 10 C3 20 19 18 228

4. There are 5 different books on Physics, 4 different 8. A bag contains 13 white and 7 black balls. Two
books on Chemistry and 2 different books on balls are drawn at random. What is the probability
Mathematics. How many ways are there to select that they are of the same colour?
a book? Solution: Total number of cases = 20
C2 =
Solution: Since each book is different therefore total 20 19 18!
= 190
number of ways to select a book = 5 + 4 + 2 = 11 2! × 18!
Or 11C1 = 11 Number of favourable cases
5. In how many different ways can the letters of the 13 12 11! 7 6 5!
= 13C2 + 7C2 = +
word ‘THERAPY’ be arranged so that the vowels 2! 11! 2! 5!
never come together? = 78 + 21 = 99
Solution: Number of permutations of the word 99
Required probability =
‘THERAPY’ = 7! = 5040 190

________________________________________________________
632
Subhash Institute of Competitive Exams (SICE) Vijayawada
Vijaya Ph: 9030924949
________________________________________________________
Mathematics for Competitive Exams ( 165 )
Section III – Chapter 14: Statistics & Probability

PRACTICE QUESTIONS
Answer the following questions by selecting the most appropriate option.

1. The marks obtained by 17 students in a 10. What will be the median of following numbers:
mathematics test (out of 100) are given below:
2, 3, 5, 4, 3, 2, 4, 6, 7, 3, 2, 5, 7
91, 82, 100, 100, 96, 60, 82, 76, 79, 90, 46, 64, (1) 3 (2) 4
72, 68, 66, 48, 49. (3) 3.5 (4) 4.5
What is the mean for this data? 11. What will be the mode of the following numbers
(1) 65 (2) 70 is:
(3) 75 (4) 80
10, 30, 50, 30, 10, 20, 20, 10, 20, 30, 30, 40, 20,
2. Consider the data: 2, 3, 9, 16, 3, 16, 3, 9. What is 20, 40, 40, 40, 50, 10, 40, 50, 50, 30, 30
the mode of these observations? (1) 10 (2) 20
(1) 2 (2) 9 (3) 30 (4) 40
(3) 16 (4) 3
12. The mode of the following observations is 13.
3. Find the mean of the data: What will be the value of x.
2, 8, 6, 5, 4, 5, 6, 3, 6, 4, 9, 1, 5, 6, 5 11, 13, 15, 11, 15, 11, 13, 11, 14, 12, 12, 14, 13,
(1) 3 (2) 4 12, 15, 14, 12, 13, 12, 12, x, 13, 15, 13, 14
(3) 5 (4) 6 (1) 11 (2) 12
(3) 13 (4) 14
4. The following observations have been arranged in
ascending order: 13. Find the median of the following data:
26, 29, 42, 53, x, x + 2, 70, 75, 82, 93 53, 34, 33, 57, 29, 55, 75, 55, 25, 27, 35, 45, 53,
45, 52, 33, 57, 33, 28, 92, 33, 33, 45, 52, 75, 33,
If the median is 65, what will be the value of x.
17, 13, 25, 33
(1) 50 (2) 54
(1) 33.5 (2) 34
(3) 64 (4) 70
(3) 34.5 (4) 36
5. The range of the data:
14. Find the mean deviation of the following data:
18, 35, 16, 6, 20, 17, 15, 22, 12, 30, 32, 10, 19,
8, 11, 5 is Size (x) 1 3 5 7 9 11 13 15
(1) 10 (2) 15 Frequency (f) 3 3 4 14 7 4 3 4
(3) 25 (4) 30
(1) 2.85 (2) 2.90
6. The class mark of the class 80 – 100 is (3) 2.95 (4) 3.00
(1) 20 (2) 10
(3) 90 (4) 180 15. What will be the standard deviation for the
following data?
7. The mean of 10 numbers is 20. If one number is
excluded, their mean becomes 19. The excluded 57, 64, 43, 67, 49, 59, 44, 47, 61, 59
number is: (1) 8.85 (2) 7.90
(1) 11 (2) 29 (3) 6.90 (4) 8.13
(3) 20 (4) 21 16. Calculate variance of the following data:
8. If mean of x – 3, x + 1 and x + 5 is 11, then what Class interval Frequency
is the value of third number in the sequence?
(1) 11 (2) 13 4-8 3
(3) 15 (4) 17 8-12 6
9. The mean of 10 observations is 13. If 4 is 12-16 4
subtracted from each number, then what will be 16-20 7
new mean?
(1) 9 (2) 7 (1) 18 (2) 19
(3) 1 (4) – 27 (3) 20 (4) 21

________________________________________________________
633
Subhash Institute of Competitive Exams (SICE) Vijayawada
Vijaya Ph: 9030924949
( 166 )
________________________________________________________
Mathematics for Competitive Exams

17. The range of the data 14, 6, 12, 17, 21, 10, 4, 3 is 27. A card is drawn from a pack of 52 cards. Find the
(1) 21 (2) 17 probability that it is neither a king nor a jack.
(3) 18 (4) 11 11 11
(1) (2)
18. The mode of the data 23, 26, 22, 29, 23, 29, 26, 26 13
29, 22, 23 is 1 9
(3) (4)
(1) 23 and 29 (2) 23 only 4 26
(3) 29 only (4) 26 only 28. A card is drawn from a pack of 52 cards. Find the
19. The median of the data 40, 50, 99, 68, 98, 60, 94 probability of getting a club and king.
is 1 7
(1) (2)
(1) 40 (2) 60 52 52
(3) 68 (4) 99 1 3
(3) (4)
20. What is the mean of first five prime numbers? 13 26
(1) 5.8 (2) 6.2 29. In a class there are 18 boys and 14 girls. Two
(3) 6.8 (4) 5.6 students are selected for prize. What is the
21. The odds against an event are 9:4. Find the probability of the selection of one boy and one
probability of occurrence of the event. girl?
4 9 5 63
(1) (2) (1) (2)
9 13 12 124
4 5 15 37
(3) (4) (3) (4)
13 9 32 62

22. A number is selected at random from first 200 30. It four coins are tossed, then the probability of
natural numbers. Find the probability that the getting at least one as tail is
number drawn is a perfect cube. 1 5
(1) (2)
3 2 3 16
(1) (2)
25 15 1 1
(3) (4)
1 7 12 8
(3) (4)
40 75 31. Probability for Gagan to fail in an exam is 0.15
23. A bag contains 3 red, 5 yellow and 4 green balls. and for Prakash to pass the exam is 0.88, then
Three balls are drawn out. What is the probability find the probability that either Gagan or Prakash
that none of the three is yellow-coloured? fails in the exam.
(1) 7/44 (2) 9/44 (1) 0.75 (2) 0.15
(3) 13/44 (4) 3/44 (3) 0.36 (4) None of these
24. There are 12 boys and 16 girls in a particular 32. An urn contains 4 yellow, 6 green and 5 red balls.
class. If the teacher calls out a particular Three balls are drawn out at random. Find the
enrolment number, what is the probability that probability that none of the balls drawn is green.
the enrolment number will be that of a girl? (1) 12/65 (2) 1/5
(1) 1/4 (2) 1/5 (3) 2/7 (4) 17/60
(3) 3/12 (4) 4/7 33. If the probability of selection of Anita, Yogesh and
25. Two dice are thrown simultaneously What is the 3 1 5
Vijay in a company is , and , respectively.
probability that the sum of the numbers of two 7 3 8
dice is more than 10? Find the probability of selection of at least two
1 2 persons.
(1) (2)
12 15 37
(1)
1 1 84
(3) (4)
8 11 17
(2)
26. In a throw of two dice the probability of getting a 84
sum of at most 4 is 47
(3)
7 5 84
(1) (2) 55
36 18 (4)
5 84
(3) (4) None of these
36

________________________________________________________
634
Subhash Institute of Competitive Exams (SICE) Vijayawada
Vijaya Ph: 9030924949
________________________________________________________
Mathematics for Competitive Exams ( 167 )
Section III – Chapter 14: Statistics & Probability

34. What is the probability that in a group of four 35. Three dice are thrown simultaneously. What is the
persons all the four are born in the same month? probability of getting a multiple of 2 on all the
1 1 three dice?
(1) (2)
12 43 1 1
(1) (2)
1 1 18 72
(3) (4)
124 123 1 1
(3) (4)
24 36

________________________________________________________
635
Subhash Institute of Competitive Exams (SICE) Vijayawada
Vijaya Ph: 9030924949
________________________________________________________
Mathematics for Competitive Exams ( 168 )

ANSWERS AND EXPLANATIONS


1. (2) To find the mean, putting 7. (2) The mean of 10 numbers is 11. (3) Arranging the given data in
these values in the formula we 20. ascending order, we get
get,
Sum of 10 numbers = 10 × 20 10, 10, 10, 10, 20, 20, 20, 20,
91 82 100 100 96 60 = 200 20, 30, 30, 30, 30, 30, 30, 40,
82 76 79 90 46 64 40, 40, 40, 40, 50, 50, 50, 50
Mean of 9 numbers = 19
72 68 66 48 49 Here,
Sum of 9 numbers = 9 × 19 =
17
171 10 is 4 times,
1190 Excluded number = 200 – 171 20 is 5 times,
= 70
17 = 29
30 is 6 times,
2. (4) The mode of these 8. (3) According to the question,
observations is 3, since it 40 is 5 times
occurs most frequently in these x 3 x 1 x 5 3x x
50 is 4 times
11 = =
set of observations. 3 3
x = 10 As 30 occurs maximum number
3. (3) To find the mean, putting of times, so 30 is the mode of
these values in the formula we Third number = x + 5 = 10 + 5 given data.
get, = 15
12. (3) 11 occurs 4 times.
2 8 6 5 4 5 6 3 9. (1) Mean of 10 observations
6 4 9 1 5 6 5 = 13 12 occurs 6 times.
=5
15 Let these ten observations are 13 occurs 6 times.
4. (3) Number of observations (n) x1, x2, x3, …, x4. 14 occurs 4 times.
= 10, which is even. Therefore, Therefore, x1 + x2 + x3 + …+ x4 15 occurs 4 times.
median = Average of the two = 130
middle terms X occurs 1 time.
(x1 – 4) + (x2 – 4)+ (x3 – 4) +
x x 2 …+ (x4 – 4) = 130 – 4 × 10 Here, 12 and 13 both occur 6
65 = times. The mode is 13 and so it
2
(x1 – 4) + (x2 – 4)+ (x3 – 4) + should occur maximum number
65 = x + 1 …+ (x4 – 4) = 90 of times.
x = 64 (x 1 4) (x 2 4) (x 3 4) ... (x 4 4) Hence, the value of x should be
5. (4) Highest value in the given 10 13.
data = 35 90
= 13. (3) Here, number of terms is 30,
10
Lowest value in the given data which is even number.
=5 Therefore, new mean = 9
Now, Median =
Range = Highest value – Lowest 10. (2) Arranging the given data in 1 n n
ascending order, we get th term
Value = 35 – 5 = 30 2 2 2
6. (3) Here, lower limit = 80 and 2, 2, 2, 3, 3, 3, 4, 4, 5, 5, 6, 7, 7
1
upper limit = 100 (15th + 16th term)
Here number of terms is 13, 2
Class mark which is an odd number. 1
= (34 + 35) = 34.5
Lower Limit Upper Limit 2
= n 1
2 Median = th term
2 f i xi
80 100 14. (3) Mean = xm = =
= 90 13 1 fi
2 = th term
2 3 9 20 98 63 44 39 60
42
7th term = 4 =8

________________________________________________________
636
Subhash Institute of Competitive Exams (SICE) Vijayawada
Vijaya Ph: 9030924949
________________________________________________________
Mathematics for Competitive Exams ( 169 )
Section III – Chapter 14: Statistics & Probability

f i xi 40, 50, 60, 68, 94, 98, 99 26. (4) Favourable outcomes = (1,
Mean Deviation =
fi 1), (1, 2), (1, 3), (2, 1), (2, 2), (3,
n 1 1) = 6
3(7) 3(5) 4(3) 14(1) Median = th term = 4th
2
7(1) 4(3) 3(5) 4(7) term = 68. Total number of outcomes in
= throw of two dice = 36
42
20. (4) Mean of the first five prime
6
= 21 15 12 14 7 12 15 28 62 numbers =
2 3 5 7 11 Required probability =
42 5 36
= 2.95 = 5.6 1
=
15. (4) Mean = (xm ) = 6
4
21. (3) Required probability = 27. (2) Number of king cards = 4
57 64 43 67 49 59 44 47 61 59 4 9
10 4 Numbers of jack cards = 4
= 55 =
13
Number of cards which are
2
xi xm 22. (3) Number of perfect cubes neither kings nor jacks = 52 –
Varianc 2) =
n from 1 to 200 = {1, 8, 27, 64, (4 + 4) = 44
125} = 5
2 2
9 2
12 2
12 2
62 42 44
Required probability =
6 2
4 2
11 2
8 2 5 52
= Required probability =
10 200 11
=
1 13
=
= 66.2 40
13
23. (1) Number of favourable cases 28. (1) P (club card) =
2 52
= 7C3 = 35
= 8.13 4
Total number of cases = 12C3 P (king card) =
f i xi 52
= 220
fi P (A and B) = P (A).P(B)
16. (2) Mean = xm =
35
Required probability =
3 6 6 10 4 14 7 18 220 13
= Required probability = ×
20 7 52
= 13 =
44 4 1
2
=
f i xi – xm 52 52
24. (4) Here, total number of cases
Variance 2) = n = 12 + 16 = 28 29. (2) Total number of outcomes =
32C2 = 496
3( 7)2 6( 3)2 4( 1)2 7(5)2 Total number of favourable
=
20 cases = 16 Favourable outcomes = 18C1 ×
14C1 = 18 × 14 = 252
= 19 Therefore, the probability that
17. (3) Highest value in the given the called out enrolment 252
Required probability =
data = 21 number is of a girl will be 16/28 496
= 4/7 63
Lowest value in the given data =
=3 25. (1) Total number of outcomes in 124
a single throw of two dice = 62 = 30. (2) At least one tail can occur in
Range = Highest value – Lowest 36
Value = 21 – 3 = 18 any of the following ways
Number of favourable outcomes HHHH, THHH, HTHH, HHTH,
18. (1) Here, 23 and 29 both occur = (5, 6), (6, 5), (6, 6) = 3
at three times in the set while HHHT
22 and 26 occur only twice. 3 Required probability =
Required probability =
Hence, both 23 and 29 are the 36 4 4 4 4 4
1 1 1 1 1
mode of this set. 1 + + +
= 2 2 2 2 2
19. (3) Arranging data in ascending 12 4
1 5
order for these seven =5× =
2 16
observations, we get

________________________________________________________
637
Subhash Institute of Competitive Exams (SICE) Vijayawada
Vijaya Ph: 9030924949
( 170 )
________________________________________________________
Mathematics for Competitive Exams

31. (4) Probability for Prakash to fail 33. (1) P (selection of at least two 1
in the exam = 1 – 0.88 = 0.12 persons) = P (selection of two Required probability =
124 1

persons) + P (selection of three 1


P(A B) = P(A) + P(B) – P(A =
persons) 123
B)
3 1 3 4 1 5 35. (2) Total number of outcomes in
Required probability = 0.15 + = × × + × × +
0.12 – 0.15 × 0.12 = 0.252 7 3 8 7 3 8 a single throw of the three dices
3 2 5 3 1 5 = 63 = 216
32. (1) Total number of cases = 15C3 × × +
7 3 8 7 3 8
= 455 Number of favourable outcomes
74 37 = (2, 2, 2), (4, 4, 4), (6, 6, 6) = 3
Number of favourable cases = = =
168 84
9C3 = 84 (This is because all the 3
Required probability =
3 balls to be drawn must be of 34. (4) Probability that in a group of 216
either yellow or red colour.) n persons all are born in same 1
=
1 72
Required probability = 9C3 /15C3 month =
12n 1
84 12
= =
455 65

________________________________________________________
638
Subhash Institute of Competitive Exams (SICE) Vijayawada
Vijaya Ph: 9030924949
( 171 )
________________________________________________________
Mathematics for Competitive Exams

15 DATA INTERPRETATION

DATA INTERPRETATION
‘Data’ refers to the collected information and ‘interpretation’ refers to determining the
conclusion. So, the process of determining conclusions and significance from the collected
information is known as data interpretation.
SMART TIP

Percentage, profit and loss, ratio and average are some useful aspects of
data interpretation.

The questions based on the representation of the provided data can be classified in the
following categories:
(i) Tabulation
(ii) Line graph
(iii) Bar graph
(iv) Pie chart

Let us take up tabulation in brief.

Tabulation
The most common way of presenting information systematically, i.e. arranged in horizontal
rows and vertical columns, in front of people is through tables. Data presented in the form of
tables is easily understandable and usable for further statistical analysis. In a table, the first
row and the first column are generally used to give the titles and help readers to understand
the data and the units used for the same.

________________________________________________________
Subhash Institute of Competitive Exams (SICE) Vijayawada
Vijaya Ph: 9030924949
________________________________________________________
Mathematics for Competitive Exams ( 172 )

Let us take up the following examples: LINE GRAPHS


Directions (Q. 1–3): Study the given table carefully and A line graph or a line chart is a graphical representation
answer the questions that follow: of the change in two variables over a period. A line graph
The following table gives the number of manufactured is created by connecting various data points. Each data
(M) and defective (D) refrigerators (in hundreds) point is obtained as a result of plotting a point when we
produced by five units of a company over the years. are given the values of two variables (one independent
variable and one dependent variable).
Unit

I II III IV V
In line graph questions, the candidates are provided with
certain data in the form of a line graph. The candidate is
Year M D M D M D M D M D
expected to analyse the data to answer the questions.
2006 64 11 55 7 86 28 66 11 56 8

2007 59 8 42 10 55 14 73 14 46 11 At times, questions give certain tables and you are


2008 60 8 58 8 65 6 78 20 44 5 required to derive a graph from the given data. Let us
2009 75 10 78 5 67 10 64 9 58 3 understand how to draw a line graph from the given
2010 80 21 82 3 92 12 58 17 68 10 table:

1. Find the ratio between the total number of


Year Annual Income in Thousand Rupee
refrigerators manufactured by Unit I to that of Unit
A B C D E
V for all the years together.
2010 900 800 700 750 830
Solution: Total number of refrigerators
2011 1000 950 700 770 850
manufactured by Unit I = 33800
2012 1050 1000 800 800 900
Total number of refrigerators manufactured by Unit V
2013 1100 1100 900 950 1050
= 27200
2014 1150 1200 1000 1000 1100
So, the ratio is 33800:27200 = 169:136
2. Out of the refrigerators manufactured by Unit II, Let us create one multiple line chart that shows the
during which year were the smallest percentage of income of all persons over the years.
refrigerators found defective?
1400
Solution:
2006 = 12.72% 1200
2007 = 23.81%
2008 = 13.79% 1000

2009 = 6.41% A
800 B
2010 = 3.66%
C
Out of the refrigerators manufactured by Unit II, the 600
percentage of defective refrigerators was least in D
2010. E
400
3. Find the average number of defective refrigerators
produced by Unit III for all the given years. 200
Solution: Total number of defective refrigerators
from Unit III = 2800 + 1400 + 600 + 1000 + 1200 = 0
7000 2010 2011 2012 2013 2014

7000
Required average = = 1400
5

________________________________________________________
640
Subhash Institute of Competitive Exams (SICE) Vijayawada
Vijaya Ph: 9030924949
________________________________________________________
Mathematics for Competitive Exams ( 173 )
Section III – Chapter 15: Data Interpretation

Let’s try to understand Line Graphs with an example: 3. What is the percent of marks obtained by Ajay in
the exams of April, Oct and Dec taken together?
Directions (Q. 1–5): Study the graph below and answer (1) 74 (2) 76
the following questions based on it. (3) 78 (4) 80
In a school, monthly exams are conducted. The given line Solution: (2) Marks obtained in April, Oct and Dec =
graph represents the aggregate marks obtained by a 360 + 375 + 405 = 1140
student Ajay during the entire session (2015-2016), i.e. % of marks obtained

from April 2015 to March 2016. (Maximum Marks = 1140
= 100 =76%
500) 1500
4. In which exams the marks obtained are less than
420
the marks obtained in previous exams and by
what percentage?
410 (1) Oct, 6.25% (2) Aug, 6.50%
(3) Oct, 6.50% (4) Aug, 7.25%
400
Solution: (1) It is clear from the graph that Oct
390
2015 got less marks than the previous exam.
400 375
Percentage decrease in marks 100
380 400
= 6.25%
370
5. In which exam was the percentage increase in
360 marks the highest over its previous exams?
(1) Aug
(2) June
(3) Dec
(4) Feb
1. The total marks obtained in August 2015 is what
percent of the total marks obtained in June Solution: (3) Percentage increase from April to June
2015? 385 360
(1) 102.2% (2) 103.8% 100
360
(3) 104.5% (4) 105%
= 6.94%
Solution: (2) Total marks in Aug 2015 = 400
Percentage increase from June to August
Total marks in June 2015 = 385 400 385
100 =3.89%
x% of 385 = 400 385

400 Percentage increase from Oct to Dec


x 100
385 405 375
= 100 = 8%
375
= 103.8%
Percentage increase from Dec to Feb
2. What are the average marks obtained by Ajay
410 405
during exams held in 2015? = 100 = 1.2%
(1) 400 (2) 390 405
(3) 385 (4) 380 Therefore, in December, the percentage increase in
Solution: (3) Average marks in 2015 marks was the highest.

360 385 400 375 405 Bar Graph


5 A bar graph or a bar chart is a graph that presents the
1930
grouped data with the help of rectangular bars. These
5 bars are either horizontal or vertical and their lengths are
proportional to the values that they represent. There are
= 385
two axes in the graph in which one represents particular

________________________________________________________
641
Subhash Institute of Competitive Exams (SICE) Vijayawada
Vijaya Ph: 9030924949
( 174 )
________________________________________________________
Mathematics for Competitive Exams

categories being compared and other axis shows a


Distribution of Total Expenditure of a
discrete value. Those bar graphs in which clustered Company (in ` lakh)
groups of more than one bar are presented are known as
30
grouped bar graphs. 25
20
An example of a bar graph is given below. 15
10
The below graph shows the model-wise sale of
5
refrigerators manufactured by a company during 0
different years:

80
70
60 400 Ltr Model
50 1. Total amount of expenditure is how many times of
40 250 Ltr Model expenditure on salary and infrastructure?
30 Solution: Total expenditure = ` (25 + 17.5 + 20 +
20 Double Door
Model 15 + 10 + 25 + 27.5) lakhs = ` 145 lakhs
10
Expenditure on salary and infrastructure = ` (25 +
0
1995 1996 1997 1998 25) lakhs = ` 50 lakhs
Therefore, total expenditure is 2.9 times the
From the above bar graph, we can obtain: expenditure on salary and infrastructure.
2. Find the ratio of total expenditure of infrastructure
i. Percentage contribution of each model in the total
and taxes to the total expenditure of transport and
sales of a company in four years
interest on loan.
ii. Relative increase or decrease in the share of each
Solution: Total expenditure of infrastructure and
model
taxes
iii. Sales trend of different models over various years, = ` (25 + 15) lakhs = ` 40 lakhs
etc
Total expenditure of transport and interest on loan
In questions based on bar graphs, either a simple or a
= ` (17.5 + 27.5) lakhs = ` 45 lakhs
cumulative or a grouped bar graph is given, providing
Required ratio of total expenditure of infrastructure
different information in the context of the same thing
and taxes to the total expenditure of transport and
and five questions related to that graph are asked. interest on loan = 40:45 = 8:9
SMART TIP 3. The expenditure on taxes is what percent of the
The data given in the bar graph tells total expenditure on advertisement, interest on
us so many things but interpreting all loans and research and development.
the data is just pointless. So, interpret
only what is required. Solution: Expenditure on taxes = ` 15 lakh
Total expenditure on advertisement, interest on
For example, loans and research and development = ` (20 + 27.5
+ 10) lakhs = ` 57.5 lakhs
Directions (Q. 1–3): Study the following bar graph
15
carefully and answer the given questions: Required percentage = 100 = 26.1%
57.5
The bar graph given below shows the percentage
distribution of the total expenditure of a company under
various expense heads during 2010.

________________________________________________________
642
Subhash Institute of Competitive Exams (SICE) Vijayawada
Vijaya Ph: 9030924949
________________________________________________________
Mathematics for Competitive Exams ( 175 )
Section III – Chapter 15: Data Interpretation

Pie Chart There are two approaches of developing a pie chart.


A pie chart is a data presentation tool in which the data These are:
is represented as the parts of a circle. A pie chart is
Percentage Approach: In this approach, any part or
divided into a number of segments in which each
segment in the pie chart is calculated as part of
segment represents a certain portion of the total value.
100%. For example, in the above pie chart, numbers
The total value (100%) is represented by a circle. The
of students who pass in different institutes have
length of the arc, the value of the central angle in the
been presented as a part of 100% (which equals to
given segment of pie chart and the area of the given
1100 students.) Therefore, you can easily say what
segment is proportionate to the value of the given
percentage of the total number students who
segment in the total value.
passed belong to which particular institute by having
Let us have a look at the following information: a look at the pie chart.

Total number of students = 1100 Degree Approach: In this approach, any part or
segment in the pie chart is calculated as part of 360
The number of students who passed from different
degree. For example, in the following pie chart,
institutes are shown in the following table:
different modes of transportation have been
Institute Number of Students depicted using the degree approach:
Passed
Modes of Transportation
Eternity Business School 275
Others ,
(EBS)
18 Degree
Sky-high Business School 180
(SBS) Private
Car, 72 Metro,
Popular Business School 200 144
(PBS) Degree
Degree
Life-skill Business School 120
(LBS) Motorcycle
Famous Business School 325 54 Degree
(FBS)
Bus , 72
Now, if we present this information in the form of a pie Degree
chart, it will look like the following:

No. of Students Passing from Different


Let us understand Pie Charts with an example:
Institutes
Directions (Q. 1–5): The pie chart below shows the
sector-wise break up of exports in 2015.
EBS
FBS 25%
30%

SBS
LBS
16%
11%
PBS
18%
The pie chart given below shows the sector—wise break
It can be seen from the above pie chart that it visually up of imports in 2015.
represents data in a way that is easy to understand and
compare.

________________________________________________________
643
Subhash Institute of Competitive Exams (SICE) Vijayawada
Vijaya Ph: 9030924949
( 176 )
________________________________________________________
Mathematics for Competitive Exams

3. Value of software exported to Germany in 2015 is


what percent of the value of defence equipment
imported from USA in 2015?
Solution: Total value of India’s export to Germany in
2015 = ` 500 crores
Value of Software’s export by India to Germany in
2015 = 8% of ` 500 crores = ` 40 crores
Total value of India’s import from USA in 2015
= ` 900 crores
1. If fresh fruit accounts for 25% of the
miscellaneous items exported in 2015, then Value of defense equipment import from USA to
exported fruits are what percent of bulk drugs India in 2015 = 20% of ` 900 crores = ` 180 crores
imported in the same year? Required percentage =   = 22.22%
Solution: Value of India’s export among various 4. Find the average value of India’s import among
trading groups in 2015 = ` (600 + 300 + 1200 + various trading groups in 2015.
800 + 900 + 500 + 1600) crores = ` 5900 crores
Solution: Value of India’s import among various
Value of miscellaneous items exported in 2015 = trading groups in 2015 = ` (800 + 200 + 900 + 800
16% of ` 5900 crores = ` 944 crores + 1100 + 300 + 1300) crores = ` 5400 crores
Value of fresh fruits exported in 2015 = 25% of `
Required average = ` crores = ` 771.43 crores
944 crores = ` 236 crores
Value of India’s import among various trading groups 5. Find the difference between the value of India’s
in the year 2015 = ` (800 + 200 + 900 + 800 + import and export among various trading groups in
1100 + 300 + 1300) crores = ` 5400 crores 2015.

Value of bulk drugs imported in 2015 = 15% of ` Solution: Value of India’s import among various
5400 crores = ` 810 crores trading groups in 2015 = ` (800 + 200 + 900 + 800
+ 1100 + 300 + 1300) crores = ` 5400 crores
Required percentage =   = 29.14% Value of India’s export among various trading groups
2. What is the value of textile export to China in in 2015 = ` (600 + 300 + 1200 + 800 + 900 + 500
2015? + 1600) crore = ` 5900 crore
Solution: Total value of export to China in 2015 Required difference = ` (5900 – 5400) crores
= ` 800 crores = ` 500 crore
The value of textile export to China in 2015 = 10% of
` 800 crores = ` 80 crores

________________________________________________________
644
Subhash Institute of Competitive Exams (SICE) Vijayawada
Vijaya Ph: 9030924949
________________________________________________________ ( 177 )
Mathematics for Competitive Exams Section III – Chapter 15: Data Interpretation

PRACTICE QUESTIONS
Answer the following questions by selecting the most appropriate option.

Directions (Q. 1–3): Answer the questions with the help 4. The demand of company B is what percentage of
of a bar chart given below. the production of company F?
(1) 50% (2) 60%
This bar chart shows the number of runs (3) 80% (4) 70%
scored by six players in a test match.
5. What is the ratio of the companies having more
demand than production to those having more
production than demand?
(1) 1 : 2 (2) 2 : 1
(3) 3 : 2 (4) 2 : 3
6. The production of company A is approximately
what percentage of the demand of company C?
(1) 50% (2) 60%
(3) 65% (4) 55%
7. What is the difference between the average
demand and the average production of the
companies (in lakh tonnes)?
1. What percentage of runs did Aditya score more [Approximately]
than Raj? (1) 325 (2) 200
(1) 20% (2) 80% (3) 275 (4) 250
(3) 75% (4) 25%
Direction (Q. 8 –10): The pie chart given below shows
2. What is the difference between the sum of runs
scored by two least scorers and two highest the distribution of the number of clients of a Company
scorers? in six different cities. Study the chart and mark the
(1) 152 (2) 160 appropriate answers.
(3) 118 (4) 130
Distribution of number of clients of a company
3. If no extra runs were scored other than
mentioned, then approximately what percent of Hyderabad
, 52.2o Delhi, 66o
total runs was scored by Aditya?
(1) 30% (2) 25%
Kolkata,
(3) 35% (4) 35% 28.8o
Mumbai,
Directions (Q. 4–7): In the following, the graph shows 30o
the demand and production of different companies.
Study the graph and answer the questions.
Bangalore,
108o Chennai,
75o

Total number of clients = 25200

8. What percentage of clients of the company is


from Kolkata?
(1) 15 (2) 8
(3) 17 (4) 21
9. What is the average number of clients from Delhi,
Mumbai and Bangalore?
(1) 4275 (2) 6150
(3) 5180 (4) 4760

________________________________________________________
645
Subhash Institute of Competitive Exams (SICE) Vijayawada
Vijaya Ph: 9030924949
________________________________________________________
Mathematics for Competitive Exams ( 178 )

10. What is the difference between the numbers of


clients from Kolkata and Chennai?
(1) 3234 (2) 3761
(3) 2986 (4) 2864
Directions (Q. 11–13): The following pie-chart shows
the sources of funds to be collected by the National
Highways Authority of India (NHAI) for its Phase II
projects. Study the pie-chart and answer the following
three questions.

14. What was the speed of the motorist between 12


noon and 1 pm?
(1) 160 km/h
(2) 60 km/h
(3) 80 km/h
(4) 140 km/h
15. What is the average speed of the motorist during
2 pm and 4 pm?
(1) 80 km/h
(2) 30 km/h
(3) 60 km/h
11. If the toll is to be collected through an outsourced (4) 40 km/h
agency by allowing a maximum 10% commission,
how much amount should be permitted to be 16. What was the highest speed that was attained by
collected by the outsourced agency so that the the motorist during the entire journey?
(1) 20 km/h
project is supported with `4,910 crores?
(2) 40 km/h
(1) `5,401 crores
(3) 60 km/h
(2) `5,827 crores
(4) 80 km/h
(3) `5,316 crores
(4) `6,213 crores Directions (Q. 17–20): Study the given graph and
12. If NHAI could receive a total of `9,695 crores as answer the following questions.
External Assistance, by what percent
(approximately) should it increase the market
borrowing to arrange for the shortage of funds?
(1) 7.5%
(2) 6%
(3) 4.5%
(4) 8%
13. The central angle corresponding to market
borrowing is
(1) 52°
(2) 137.8°
(3) 192.4°
(4) 187.2°
Directions (Q. 14–16): Study the following line graph
and answer the following questions by selecting the
correct option:
The given line graph represents the relationship between
the time taken and the distance covered by a motorist 17. What was the approximate percentage increase
in export value from the years 1995 to 1999?
for a journey of 7 hours.
(1) 233.3 (2) 200
(3) 375 (4) None of these

________________________________________________________
646
Subhash Institute of Competitive Exams (SICE) Vijayawada
Vijaya Ph: 9030924949
________________________________________________________
Mathematics for Competitive Exams ( 179 )
Section III – Chapter 15: Data Interpretation

18. What is the difference (in lakh) between the Directions (Q. 25–26): Study the following table and
boxes exported in years 1997 and 1998? answer the questions based on it.
(1) 400 (2) 352
(3) 375 (4) 330 The given table shows the marks obtained by the five
students of a class in three different subjects.
19. If in the year 1998, the boxes were exported at
the same rate per box as that in 1997, what Marks Obtained in Three Subjects
would be the value (in crore `) of export in 1998?
(1) 1000000 (2) 10 Students English Maths Science
(3) 1000 (4) 100000 A 351.6 155.5 54.2

20. The value of an aluminium box was minimum in B 407.9 134.3 42.6
(1) 1998 (2) 1999 C 380.1 149.9 38.9
(3) 1995 (4) 1996 D 439.7 160.5 50.3
Directions (21–24): The following graph shows E 485.9 203.3 65.8
production (in thousands) of two types (P and Q) of
25. The marks obtained in English by D are
vehicles by a factory over the years 2009 to 2014. approximately what per cent of marks obtained in
Study the graph and answer the questions. English by A?
(1) 75% (2) 85%
(3) 116% (4) 125%
26. Identify the student who obtained maximum per
cent increase in maths marks as compared to the
previous one?
(1) A (2) B
(3) C (4) E
Directions (Q. 27 - 30): The table given below shows
the ratio between literate and illiterate persons and
the population of seven villages of a district. Study the
table and select the most appropriate answers.
Village Literate: Illiterate Population
21. The production of Type Q vehicles in 2010 was
approximately what percent of Type P vehicles in Nathupur 9:2 2354
2014?
Dhankot 4:1 2540
(1) 75 (2) 54.5
(3) 45.5 (4) 60 Basai 7:2 2709
22. The ratio of total production of Type P vehicles to Badshahpur 13:4 5100
total production of Type Q vehicles over the years
is Kadipur 5:3 3248
(1) 5 : 8 (2) 8 : 5
(3) 41 : 48 (4) 48 : 41 Pataudi 5:2 4921

23. In how many years was the production of Type P Bilaspur 13:6 3990
vehicles of the company more than the average
production of this type of vehicles in the given 27. If 42% of the illiterates of Badshahpur are males,
years? then what is the number of illiterate women in
(1) 2 (2) 4 this village?
(3) 5 (4) 3 (1) 632 (2) 598
(3) 546 (4) 696
24. The total production of Type P vehicles in the
years 2009 and 2011 is what percent of total 28. Which village has the maximum percentage of
production of Type Q vehicles in 2010 and 2014? literates?
(1) 75 (1) Nathupur
(2) 80 (2) Dhankot
(3) 81.25 (3) Badshahpur
(4) 69.25 (4) Kadipur

________________________________________________________
647
Subhash Institute of Competitive Exams (SICE) Vijayawada
Vijaya Ph: 9030924949
( 180 )
________________________________________________________
Mathematics for Competitive Exams

29. By what percentage is the number of literates in Directions (Q. 36–38): Study the following Pie-chart
Pataudi more than the number of literates in carefully to answer these questions.
Nathupur?
(1) 55.78 (2) 82.50 Total Number of Passengers = 8,500
(3) 69.33 (4) 93.25 Percentage of Passengers
30. What is the difference between the number of
illiterates of Kadipur and the number of literates
of Basai?
(1) 889 (2) 673
(3) 691 (4) 804
Directions (Q. 31–35): Study the bar graph and answer
the questions given below.
The bar graph shows the number of fans, tables and
chairs (in thousands) sold by four shopkeepers A, B, C
and D in a year:
36. What was the approximate average number of
passengers in train-S, train-M and train-L
together?
(1) 1521 (2) 1641
(3) 1651 (4) 1671
37. If in train-R, 34 percent of the passengers are
females and 26 percent are children, what is the
number of males in that train?
(1) 306 (2) 316
(3) 308 (4) 318
38. Number of passengers in train-Q is approximately
31. Which shopkeepers sold the maximum percentage what percentage of the total number of
of number of tables? passengers in train-A and train-R?
(1) D (2) B (1) 90 (2) 86
(3) A (4) Both C and D (3) 75 (4) 80
32. How many articles were sold by all four Directions (Q. 39–40): Study the following table
shopkeepers together?
(1) 468000 (2) 226000 carefully to answer the questions that follow:
(3) 474000 (4) 552000 Number of flights cancelled by five different airlines in six
33. Which shopkeeper sold the highest number of different years
chairs?
(1) A (2) B Airlines P Q R S T
(3) C (4) D Year

34. Which articles were the maximum purchased by 2005 240 450 305 365 640
buyers? 2006 420 600 470 446 258
(1) Fans 2007 600 680 546 430 610
(2) Chairs
(3) Both tables and fans 2008 160 208 708 550 586
(4) Tables 2009 140 640 656 250 654
35. The number of chairs sold by shopkeeper B is 2010 290 363 880 195 483
what percent of the number of tables sold by
shopkeeper C? 39. What was the difference between the highest
(1) 19.15% number of flights cancelled by airlines-Q and the
(2) 21.23% lowest number of flights cancelled by airlines-T
(3) 17.13% out of all the six years?
(4) 25.36% (1) 446 (2) 456
(3) 422 (4) 442

________________________________________________________
648
Subhash Institute of Competitive Exams (SICE) Vijayawada
Vijaya Ph: 9030924949
________________________________________________________
Mathematics for Competitive Exams ( 181 )
Section III – Chapter 15: Data Interpretation

40. What was the approximate percentage increase Directions (Q. 44–46): Study the following table
in the number of flights cancelled by airlines S in carefully to answer the given Questions.
the year 2008 as compared to the previous year?
(1) 127 (2) 28 Number of pens manufactured by five different
(3) 150 (4) 45 companies during five different years
Directions (Q. 41–43): Study the following pie charts (In thousands)
carefully to answer the questions: Company P Q R S T
Degree-wise break-up of the employees working in Year
various departments of an organisation and the 2001 122 235 230 265 300
ratio of Men to Women
2002 250 130 240 240 200
Total Number of Employees = 3250
2003 260 170 190 190 155

2004 190 235 185 170 175

2005 150 200 135 225 240

44. What is the percentage increase in number of


pens manufactured by company R in 2002 as
compared to the previous year?
(1) 8.12% (2) 7.25%
(3) 5.68% (4) 4.35%
45. Number of pens manufactured by company P and
Q together in 2002 is what percent of number of
pens manufactured by company T in 2002 and
Dept. = Department 2005 together?
(1) 82.13% (2) 86.36%
Respective Ratios of Men to Women in Each Department (3) 92.25% (4) 90.12%
Department Men Women 46. What is the ratio between number of pens
Production 4 1 manufactured by company S in 2003 and 2004
together to number of pens manufactured by
HR 12 13 company P in 2004 and 2005 together?
IT 7 3 (1) 13 : 12 (2) 5 : 7
Marketing 3 2 (3) 18 : 17 (4) 17 : 18
Accounts 6 7 Direction (Q. 47–50): The table given below shows the
national income and population of ten states of India.
41. What is the number of men working in the
Study the table and select the most appropriate
Marketing department?
(1) 462 (2) 454 answers.
(3) 418 (4) 429 State GDP in GDP in Population Population
2001 (in 2011 (in in 2001 (in in 2011 (in
42. What is the respective ratio of the number of billion billion millions) millions)
women working in the HR department and the dollars) dollars)
number of men working in the IT department? Uttar Pradesh 124 150 167 199
(1) 11 : 12 (2) 17 : 29 Madhya 63 75 60 72
(3) 13 : 28 (4) 12 : 35 Pradesh

43. The number of men working in the Production Punjab 42 47 24 27


department of the organisation forms what Rajasthan 64 85 56 68
percentage of the total number of employees
Bihar 38 51 82 103
working in that department?
(1) 88 (2) 80 West Bengal 96 120 80 91
(3) 75 (4) 65 Orissa 35 40 36 41

________________________________________________________
649
Subhash Institute of Competitive Exams (SICE) Vijayawada
Vijaya Ph: 9030924949
( 182 )
________________________________________________________
Mathematics for Competitive Exams

State GDP in GDP in Population Population 52. What is the approximate percent increase in the
2001 (in 2011 (in in 2001 (in in 2011 (in number of passengers travelling in Train B on
billion billion millions) millions)
dollars) dollars) Tuesday as compared to the previous day?
Maharashtra 166 250 96 112
(1) 49 (2) 52
(3) 59 (4) 45
Gujarat 84 110 51 60
53. In which train, the number of passengers
Tamil Nadu 131 150 62 72
travelling consistently increases from Monday to
Saturday?
GDP
GDP Per capita = (1) A (2) B
Population
(3) C (4) D
47. How many states have more than 30% growth in Directions (Q. 54–57): Study the following graph
their GDP from 2001 to 2011? carefully to answer the questions.
(1) Three
(2) Four Amounts invested (in ` thousands) by three people in
(3) Five various schemes over the year
(4) Six
48. Which state has the highest percentage increase
in GDP from 2001 to 2011?
(1) Maharashtra
(2) Gujarat
(3) Rajasthan
(4) Bihar
49. Which state has the highest percentage increase
in population from 2001 to 2011?
(1) Uttar Pradesh (2) Rajasthan
(3) Maharashtra (4) Bihar
50. What is the average population (in millions) of all
the states in 2011?
(1) 81.40 (2) 72.20 54. What is the average amount invested in the year
(3) 91.60 (4) 84.50 2009 by all the three people together?
1
Directions (Q. 51–53): Study the following table (1) ` 28,350 (2) ` 48,333
3
carefully to answer the questions that follow:
1
Number of passengers (in hundreds) travelling in trains (3) ` 32,333 (4) ` 45,000
3
on six different days:
55. What is the percentage decrease in the amount
invested by C in the year 2007 from the previous
Train A B C D E
year?
Day (1) 11.3% (2) 13.5%
Monday 4.55 4.38 4.64 7.73 7.68 (3) 12.5% (4) 14.3%
Tuesday 3.54 6.34 6.90 5.82 5.59 56. What is the respective ratio of total amount
Wednesday 7.65 2.79 7.23 5.64 6.28 invested by B in the years 2008 and 2010
together to the total amount invested by C in
Thursday 6.75 8.65 7.83 9.74 8.83
those two years together?
Friday 5.78 9.50 8.91 8.16 6.54 (1) 7 : 8 (2) 14 : 15
Saturday 3.48 5.39 9.84 9.92 10.9 (3) 6 : 7 (4) 12 : 13
57. The amount invested by A in the year 2006 is
51. What is the difference between the total number
approximately what percentage of the total
of passengers travelling in Train C on Thursday
amount invested by him over all the years
and Saturday together and the number of
together?
passengers travelling in Train E on Saturday?
(1) 15% (2) 19%
(1) 667 (2) 676
(3) 21% (4) 8%
(3) 687 (4) None of these

________________________________________________________
650
Subhash Institute of Competitive Exams (SICE) Vijayawada
Vijaya Ph: 9030924949
________________________________________________________
Mathematics for Competitive Exams ( 183 )
Section III – Chapter 15: Data Interpretation

Directions (Q. 58–60): Study the following table 58. In which place did the monthly rent not increase
carefully to answer the questions that follow: consistently from the years 2005 to 2010?
(1) Virar (2) Dadar
Monthly Rent (in thousands) at five different places in six (3) Kandivali (4) Borivali
different years
59. In which year at Churchgate did the monthly rent
Place increase more than 100 percent from the
previous year?
Year Churchgate Dadar Kandivali Borivali Virar (1) 2006 (2) 2007
2005 5.3 3.8 1.5 2.7 1.1 (3) 2008 (4) 2009
2006 12.5 8.3 3.4 4.8 2.1 60. What was the difference between the monthly
2007 16.7 11.7 5.5 6.6 1.8 rent at Dadar in the year 2009 and Borivali in the
2008 20.9 13.6 9.8 12.7 3.6
year 2007?
(1) ` 7600 (2) ` 7900
2009 25.8 14.5 11.5 14.1 5.5 (3) ` 8100 (4) ` 8600
2010 30.3 20.9 15.6 15.9 7.8

________________________________________________________
651
Subhash Institute of Competitive Exams (SICE) Vijayawada
Vijaya Ph: 9030924949
( 184 )
________________________________________________________
Mathematics for Competitive Exams

ANSWERS AND EXPLANATIONS


1. (4) The percentage of runs 1450 13. (4)
Aditya scored more than Raj is = 100 55.76% 55%
2600 Whole fund = ` (11486 + 5252
200 160
100 7. (3) Average demand = + 4910 + 6000 + 29952)
160 crores
2100 3150 2600 5000 2800 3300
40
100 25% 6
160 = ` 57600
18950
= We know that the total angle =
2. (1) Sum of two least scores is 6
82 + 120 = 208 360°
Average production =
Sum of two highest scores is Whole fund = ` 57600
1450 3660 3100 4200 3700 4500
200 + 160 = 360 So ` 57600 = 360°
6
So, the difference is 360 – 208
= 3435 lakh tonnes ` 29952 = (360° / 57600) ×
= 152
29952 = 187.2°
3. (2) Total runs scored was 130 + Difference = 3435 3158.34 =
276.66 = 276.7 275 lakh 14. (2) The motorist covered 60 km
120 + 82 + 120 + 160 + 200
tonnes in one hour between 12 noon
= 812
and 1 pm, therefore, the speed
% of runs scored by Aditya is 8. (2) Required percentage = of the motorist during this time
200 28.8 = 60 km/h.
100 24.63 25% 100 = 8
812 360
15. (4) The required average can be
4. (4) 9. (4) Average central angle for calculated as the average of the
Delhi, Mumbai and Bangalore = individual speeds between 2 pm
Required percentage = 66 30 108 to 3 pm and between 3 pm to 4
= 68°
100 3 pm
Average number of clients from
3150 = = 40 km/h
Delhi, Mumbai and Bangalore = 2
= 100 = 70%
4500 68
25200 = 4760 16. (4) Let us list the speed attained
5. (1) 360
by the motorist during different
10. (1) Required difference = time periods as follows:
75 28.8
Ratio = × 25200 = 3234 Duration Speed
360
9.00 am – 10.00 am 20
11. (1) 10.00 am – 11.00 am 40
Companies having more Let the amount permitted is = `
demand than production = A A 11.00 am – 12.00 40
and D noon
So, A = (4910 × 110) / 100 =
Companies having more 491 × 11 = ` 5401 crores 12.00 noon – 1.00 pm 60
production than demand = B, C,
E and F 12. (2)
1.00 pm – 2.00 pm 80
2 1 Difference = ` (11486 9695) 2.00 pm – 3.00 pm 60
Ratio = =1:2 = `1791 crores
4 2 3.00 pm – 4.00 pm 20
Required percentage increase
6. (4)
in market borrowing Therefore, the highest speed
Percentage = attained by the motorist during
= (1791 ×100) / 29952 the journey = 80 km/h.
100
= 5.979 6%

________________________________________________________
652
Subhash Institute of Competitive Exams (SICE) Vijayawada
Vijaya Ph: 9030924949
________________________________________________________
Mathematics for Competitive Exams ( 185 )
Section III – Chapter 15: Data Interpretation

17 (1) 1200000 Number of illiterate females


Required ratio =
Percentage increase in export 1025000 = 58% of 1200 = 696
value from 1995 to 1999 = = 48 : 41
28. (1) It can be observed from the
500 150 23. (4) Total production of type P table that the ratio of literates to
100
150 vehicles = 12,00,000 illiterates is maximum for
350 Nathupur.
100 233.33% Average production of type P
150
1200000 Nathupur has the maximum
18. (2) vehicles = =
6 percentage of literates.
Rate of 1 box in 1997 2,00,000
29. (2) Number of literates in
330 10000000 It can be observed by visual 5
= `220 Pataudi = × 4921 = 3515
150 100000 inspection that in years 2012, 7
2013 and 2014, production of
Value of 160 lakh boxes in Number of literates in Nathupur
type P vehicles is more than the
1998 = 220 × 16000000
average production. 9
= ` 352 crore = × 2354 = 1926
11
24. (2) Total production of type P
19. (1) According to the question
vehicles in 2009 and 2011 = Required percentage
Difference = Boxes exported in (100 + 200) × 1000 = 3515 1926
1998 Boxes exported in 1997 3,00,000 = × 100 = 82.50
1926
= 16000000 15000000 Total production of type Q
30. (1) Number of illiterates of
= 1000000 vehicles in 2010 and 2014 =
3
(150 + 225) × 1000 = Kadipur = × 3248 = 1218
20. (3) Value of 1 box in 1995 3,75,000 8
150 10000000 Number of literates of Basai =
= `150 Required percentage
100 100000 7
300000 × 2709 = 2107
Value of 1 box in 1996 = 100 = 80% 9
375000
150 10000000
= `200 25. (4) Marks obtained by D in Required difference = 2107
75 100000 1218 = 889
English = 439.7
Value of 1 box in 1998 30. (4) Percentage decrease in
Marks obtained by A in English
400 10000000 production of type Q vehicles
= `250 = 351.6
160 100000 from 2010 to 2011 =
Required percentage 25
Value of 1 box in 1999 100
439.7
500 10000000 = 100% 125.06% 125% 150
= `250 351.6
= 16.67 16.7
200 100000
26. (4) Increase in marks obtained
Value per box is minimum in 31. (2) % of table sold by
by C in maths
1995. 40
149.9-134.3 shopkeeper A = =
= 100 11.6 % 86
21. (2) Required percentage 134.3 46.51%
150
= 100 = 54.54 54.5 Increase in marks obtained by D % of table sold by shopkeeper B
275
in maths = 88
22. (4) Total production of type P 160.5 149.9 = = 51.76%
100 7% 170
vehicles = (100 + 125 + 200 + 149.9
225 + 275 + 275) × 1000 % of table sold by shopkeeper C
= 12,00,000 Increase in marks obtained by E 47
in maths = = = 45.63%
103
Total production of type Q 203.3 160.5
vehicles = (175 + 150 + 125 + 100% 26.6 % of table sold by shopkeeper D
160.5
175 + 175 + 225) × 1000 = 51
10,25,000 27. (4) Number of illiterates in = = 44.34%
115
4
Badshahpur = 5100 1200
17

________________________________________________________
653
Subhash Institute of Competitive Exams (SICE) Vijayawada
Vijaya Ph: 9030924949
( 186 )
________________________________________________________
Mathematics for Competitive Exams

32. (3) Total articles sold by all 39. (3) Required difference = 680 – 45. (2) Number of pens
shopkeepers = (86 + 170 + 258 manufactured by company P
103 + 115) thousand and Q together in 2002 =
= 422
      
= 474000
40. (2) Required percentage =
Number of pens manufactured
33. (4) Percentage of chair sold by A 550 430
100 by company T in 2002 and
1000 430
= = 1.16% 2005 together =    
86000
120   
=  100 28%
Percentage of chair sold by B 430
9000
Percentage =   
= = 5.29% 41. (4)
170000 = 86.36%
Men working in Marketing dept.
Percentage of chair sold by C   46. (3)
=  = 429
5000
= 100% = 4.85% Number of pens manufactured
103000 42. (3) by company S in 2003 and
Percentage of chair sold by D Number of women working in 2004 together =    
8000 the HR department =   
= 100% = 6.95%
115000 13 36 Number of pens manufactured
3250 = 169
Hence, maximum chairs were 25 360 by company P in 2004 and
sold by D. 2005 together =    
Number of men working in the
  
34. (4) Total fans sold = (45 + 73 + IT department
51 + 56) thousand = 225000 Ratio =   
7 57.6
= 3250 = 364
Total tables sold = (40 + 88 + 10 360 47. (2) It can be observed from the
47 + 51) thousand = 226000 given table that the states
169 13
Tables were purchased more by Required ratio = having more than 30% growth in
buyers. 364 28 their GDP from 2001 to 2011
43. (2) are Rajasthan, Bihar,
35. (1) Required percentage Maharashtra and Gujarat.
= Ratio of Men in the Production
Note: Do not go for exact
4
department = calculation for these types of
9000
= 19.15%
5 sums. Always go for
47000 approximation as explained
Men in the Production
36. (4)Required average number of below:
department as percentage of
passengers is: the total no. of employees in Percentage growth for Bihar =
2040 1700 1275 4 13
× 100
= 1671 that dept. = 100 80% 38
3 5
37. (1) Number of males in train 44. (4) 13 13 1
is more than or which
100 34 26 38 39 3
R = 765 Number of pens manufactured
100 is equal to 33.33%
by company R in 2002 =
40      48. (1) It can be observed from the
= 765 306 table that Maharashtra is the
100 Number of pens manufactured only state whose GDP in 2011
38. (2) Required percentage is: by company R in 2001 = is more than 1.5 times of its
230 ×1000 = 230000 GDP in 2001 while for all other
1615
= 100 states GDP in 2011 is less than
1870 Percentage = 
1.5 times of its value in 2001.
100 =  4.35%
Maharashtra has the highest
growth in GDP from 2001 to
2011.

________________________________________________________
654
Subhash Institute of Competitive Exams (SICE) Vijayawada
Vijaya Ph: 9030924949
________________________________________________________
Mathematics for Competitive Exams ( 187 )
Section III – Chapter 15: Data Interpretation

49. (4) It can be observed from the No. of passengers travelling in Total investment of C in 2008
table that an increase in Train E on Saturday = 1090 and 2010 together = `40000 +
population from 2001 to 2011 `40000 = `80000
Required difference = 1767 –
is more than 20% only for
1090 = 677 70000
Rajasthan and Bihar while for Required ratio =
all other states increase in 52. (4) Difference of passengers 80000
population is less than 20%. between Tuesday and Monday = =7:8
Thus, there is no requirement to 634 – 438 = 196
calculate percentage increase 57. (1)Total investment of A for all
Percentage increase in the years = (30 + 35 + 45 + 35
for other states.
passengers on Tuesday =
+ 40 + 50)  1000
For Rajasthan, percentage 196 100
increase in population from = `235000
2001 to 2011 438
53. (3) The number of passengers Investment of A in 2006 =
68 56 `35000
= 100 21.42 consistently increases in train C
56
as is clearly seen in the given Required percentage =
For Bihar, percentage increase table. 35000
in population from 2001 to 100 14.9 15%
54. (2) Average amount invested by 235000
103 82 all the three people together in
2011 = 100 25.60 58. (1) It is clear from the table that
82 2009 in Virar, rent did not increase
Therefore, Bihar has highest 40000 50000 55000 consistently.
percentage increase in 3 59. (1) It is clear from the table that
population from 2001 to 2011. 145000 1
` 48333 in 2006, rent increased more
50. (4) Total population of all the 3 3 than 100% at Churchgate.
states in 2011 = 199 + 72 + 27 55. (3) Investment in 2006 by C = 60. (2) Required difference is:
+ 68 + 103 + 91 + 41 + 112 + 40000
60 + 72 = 845 millions = 14500 – 6600
Investment in 2007 by C =
Average population of all the 35000 = 7900
845
states in 2011 = = 84.5 Investment decreased by =
10
5000
millions x100 12.5%
40000
51. (4) No. of passengers travelling
56. (1) Total investment of B in
in Train C on Thursday and
2008 and 2010 together =
Saturday = 783 + 984 = 1767
`25000 + `45000 = `70000

________________________________________________________
655
Subhash Institute of Competitive Exams (SICE) Vijayawada
Vijaya Ph: 9030924949

You might also like